Analytic Geometry for Colleges, Universities & Schools eBook

June 6, 2016 | Author: Elvir Crncevic | Category: N/A
Share Embed Donate


Short Description

Analytic Geometry for Colleges, Universities & Schools eBook...

Description

ll

mmill

AfTRONOMY

ANALYTIC GEOMETRY FOR

COLLEGES, UNIVERSITIES, AND TECHNICAL SCHOOLS.

BY E.

VV.

NICHOLS,

PROFESSOR OF MATHEMATICS IN THE VIRGINIA MILITARY INSTITUTE.

LEACH, SHEWELL, & SANBORN, BOSTON. NEW YORK. CHICAGO.

COPYRIGHT, iwi,

BY LEACH, SIIKWELL, & SANBOKN.

C.

J.

PETERS & SON,

TYPOGRAPHERS AND ELECTROTYPERS. PRESS OF BERWICK

&

SMITH.

Nsf PEEFACE. THIS text-book

is

designed

and Technical Schools. to prepare a

make

it

work

for

The aim

for beginners,

Colleges,

of

and

the

Universities,

author has been

at the

same time to

for the requirements

of comprehensive For the methods of develop ment of the various principles he has drawn largely upon his sufficiently

the usual undergraduate course.

experience in the class-room. all

home and

authors,

In the preparation of the work

foreign,

have been freely consulted. In the first few chapters discussion of each principle.

whose works were

available,

elementary examples follow the In the subsequent chapters

sets

of examples appear at intervals throughout each chapter, and are so arranged as to partake both of the nature of a review and an extension of the At the preceding principles.

end of each chapter general examples, involving extended application of the are principles deduced,

the benefit of those

who may

a more

placed for

desire a higher course in the

subject.

The author takes pleasure cussion

of

Surfaces,"

Mathematics

in

appears as the

He

by

in calling attention to a

and Lee University, chapter in this work.

Washington

final

"Dis

A. L. Nelson, M.A., Professor of

which

takes pleasure also in acknowledging his indebtedness

i

PREFACE.

v

Prof. C. S. Venable, LL.D., University of Prof. William Cain, C.E., University

of

to

and

Virginia, to

North

Carolina,

of Pennsylvania, to Prof. E. S. Crawley, B.S., University

for assistance rendered in reading

and revising manuscript,

and for valuable suggestions given. E.

LEXINGTON, YA. January, 1893.

W. NICHOLS.

CONTENTS. PAET

I.

PLANE ANALYTIC GEOMETRY. CHAPTEK

I.

CO-ORDINATES.

PAGES

ARTS. 1-3.

4-6.

The Cartesian or Bilinear System. The Polar System. Examples

CHAPTER

Examples

....

1

4

II.

LOCI. 7.

Locus of an Equation.

8.

Variables.

9.

10-16. 17, 18.

The Equation

Constants. Examples Relationship between a Locus and

of a

...

9 10

its

Equation

Discussion and Construction of Loci. Methods of Procedure. Examples

CHAPTER

Locus

....

11

...

11

Examples

23

III.

THE STRAIGHT LINE.

21.

The Slope Equation. Examples The Symmetrical Equation. Examples The Normal Equation

22.

Perpendicular Distance of a Point from a Line.

19.

20.

25 29 32

Ex

26.

amples Equation of Line, Axes Oblique. Examples General Equation, Ax + B?y + C = Equation of Line passing through a Point. Examples Ex Equation of Line passing through Two Points.

27.

Length

23.

24.

25.

37 .

38 39

amples of

33 35

Line joining

Two

Points.

v

Examples

...

41

CONTENTS.

vi ARTS. 29. 30.

Intersection of

Two

PAGES

....... = ......

.

28.

Examples

Lines.

Ax + By + C + K(A a5 + BV + e ) Examples. Angle between Two Lines.

Ex

General

.................

amples

CHAPTER

IV.

TRANSFORMATION OF CO-ORDINATES. 31. 32. 33*

34 35

Objects

of.

............

Illustration

Examples to a Parallel System. an Oblique System. Rectangular Rectangular System to also Rectangular. Examples. System to Another System Polar a to System. From a Polar Rectangular System Examples. General System to a Rectangular System.

From One System

Examples

....... ......... CHAPTER THE

V.

CIRCLE.

Equation of Circle

38.

Generation of Circle. General Equation of

39.

Polar Equation of Circle

36, 37.

~rv

ou

.

.

Circle.

....

Concentric

Circles.

59

Ex

................. ............ Supplemental Chords ............. Tangent. Sub-tangent ............ Normal. Sub-normal ............. General Equations Tangent and Normal. Examples Tangent .............. Length amples

40.

41. 42. 43. 44.

45, 46.

of

of

Radical Axis.

Radical Centre.

Examples

.....

47.

Condition that a Straight Line touch a Circle.

48.

Equation of Tangent Chord of Contact Pole and Polar

49, 50. 51.

^7

.

............ ............... .....

Conjugate Diameters.

Examples.

CHAPTER

General Examples

.

VI.

THE PARABOLA. 52, 53.

70

Slope

Generation of Parabola. tions

Equation of Parabola.

Defini

................ ........... Parabola

54.

Construction of

55.

Latus-Rectum.

Examples

77

CONTENTS.

Vil

PAGES

ARTS. 57-59.

Polar Equation of Parabola Tangent. Sub-tangent. Construction of Tangent

60, 61.

Normal.

Sub-normal

62.

Tangents

at Extremities of

63.

ici

56.

= +

88 .

.

89

90

Latus-Rectum

91

92

64.

Examples y% Tangent Line makes Equal Angles with Focal Lines and Axis Condition that a Straight Line touch the Parabola.

95

65.

66.

67.

68. 69. 70.

71, 72. 73.

+

c$

Slope

Equation of Tangent Locus of Intersection of Tangent and Perpendicular from Focus Locus of Intersection of Perpendicular Tangents Chord of Contact Pole and Polar Conjugate Diameters Parameter of any Diameter. Equation of a Diameter

...

.

at the Extremities of a Chord. General Examples

Tangents

CHAPTER

95 96 97

97 98 98 100

Examples. 101

VII.

THE ELLIPSE. 74, 75.

76. 77.

Generation of Ellipse. Definition. Equation of Ellipse Focal Radii

78.

Construction of Ellipse

Latus-Rectum.

Polar Equation of Ellipse

81.

Supplemental Chords Tangent. Sub-tangent Tangent and Line through Centre

85.

86,87. 88.

89.

90.

91. 92. 93.

94.

108 Ill

79.

84.

106 109

80.

82,83.

.

Eccentricity.

Examples .

.

.

114 115

Point

of

Tangency and

H8

...

Methods of constructing Tangents Normal. Sub-normal. Examples Normal bisects Angle between the Focal Radii Condition that a Straight Line touch the Ellipse.

...

118 119

....

122

Slope

Equation of Tangent Locus of Intersection of Tangent and Perpendicular from Focus Locus of Intersection of Perpendicular Tangents Equation of Chord of Contact Pole and Polar Conjugate Diameters .

.

.

123

124 125 125

126 126

CONTENTS.

via ARTS. 95, 96.

97. 98. 99.

PAGK8 Equation of a Diameter. Conjugate Diameter 2 a"

+

b

-

=

a*

+

Co-ordinates of Extremities of

129

b*

.

130

.

...

Parallelogram on a pair of Conjugate Diameters Relation between Ordinates of Ellipse and Circles on

Axes 100, 101.

131

133

Construction of Ellipse. General Examples

Area

CHAPTER

of Ellipse.

Examples. 134

VIII.

THE HYPERBOLA. 102,103. 104, 105.

100, 107. 108.

109. 110. 111.

112,113. 114.

Generation of Hyperbola. Definitions. Equation of Hyperbola Focal Radii Eccentricity. Construction of Hyperbola. Latus-Rectum Relation between Ellipse and Hyperbola

....

116, 117. 118.

119.

120.

121.

122. 123. 124. 125.

126, 127.

128. 129.

130. 131.

144

146

Conjugate Hyperbola. Examples Polar Equation of Hyperbola

146

Supplemental Chords Tangent, Sub-tangent Tangent and Line through Point of Tangency and

150

149

Centre 115.

141

143

Method Normal.

151

of constructing

Tangents

151

Sub-normal.

Examples Angle between the Focal Radii Condition that a Straight Line touch the Hyperbola. Slope Equation of Tangent Locus of intersection of Tangent and Perpendicular through Focus Locus of intersection of Perpendicular Tangents Chord of Contact Pole and Polars Tangent

bisects

.

.

.

.

.

Conjugate Diameters Conjugate Diameters lie in the same Quadrant Co-ordinates of Equation of Conjugate Diameter. Extremities of Conjugate Diameter 2 = a12 a2 b2 Parallelogram on a pair of Conjugate Diameters. Ex

...

-

150

b"

amples Asymptotes Asymptotes as Axes. Vertex

152 154 155 155 155

156 156 156 157 157

158 159 160

Rhombus on

Co-ordinates of 162

CONTENTS.

i

ARTS. 132.

PAGES

Tangent Line, Asymptotes being Axes. The Point of Tangency Intercepts of a Tangent on the Asymptotes Triangle formed by a Tangent and the Asymptotes Intercepts of a Chord between Hyperbola and its

.............

133. 134.

135.

x

....

Examples.

General Examples

CHAPTER

165

.

165

...

165

.

Asymptotes.

164

IX.

GENERAL EQUATION OF THE SECOND DEGREE. 136, 137.

The General Equation.

138.

First Transformation.

139.

Second Transformation a = 0. c -

140, 141. 142. 143.

144. 145.

146.

....... .... ..........

Discussion

Signs of Constants

........... Summary ............. .............. = ........... ............ ..... General Summary. Examples .........

..."

6-<4ac

62

62

4 ac

>

.

.

.

4 ac

CHAPTER

170 171

172 173 175

175 176 177 173

X.

HIGHER PLANE CURVES. 147.

148.

149. 150. 151.

152.

Definition

EQUATIONS OF THE THIRD DEGREE. The Semi-cubic Parabola Duplication of Cube by aid of Parabola The Cissoid Duplication of Cube by aid of Cissoid The Witch

........... ...... ............ ................

188 190 1 91

193 194

EQUATIONS OF THE FOURTH DEGREE. 153. 154.

155. 156.

......... ..... The Limacon ............ The Lemniscate ........... The Conchoid

Trisection of an Angle by aid of Conchoid

196 198

JQQ

201

TRANSCENDENTAL EQUATIONS. 158.

The Curve The Curve

159.

The, Cycloid

157.

........... ........... ........

of Sines of

Tangents

203 204 206

CONTENTS.

X

PAGES

ARTS.

SPIRALS, 160.

Definition

161.

The The The The The

162. 163. 164.

165.

PART

208

Spiral of

Archimedes

208 210 212 213

Hyperbolic Spiral Parabolic Spiral Lituus

214

Examples

Logarithmic Spiral.

SOLID ANALYTIC GEOMETRY.

II.

CHAPTER

I.

CO-ORDINATES. 166.

The Tri-planar System.

167.

Projections

168, 169.

Length

of

Line

joining

170.

Angles. Examples The Polar System

171.

Relation

217 219

Examples

Two

Points.

Directional

220 222

between Systems. ordinates. Examples

CHAPTER

Transformation of Co 223

II.

THE PLANE. 226

172.

Equation of Plane

173.

Normal Equation

174. 175.

Symmetrical Equation of Plane General Equation of Plane

176.

Traces.

177.

178.

Perpendicular from Point on Plane Plane through three Points

179.

Any Equation between

227 229

of Plane .

.

Intercepts

229 .

three Variables.

230 231

232

Discussion.

233

Examples

CHAPTER

III.

THE STRAIGHT LINE. 180. 181.

Equations of a Straight Line Symmetrical Equations of a Straight Line

236 237

CONTENTS.

xi

ARTS.

PAGES

182.

To

183.

184.

Planes Line through One Point Line through Two Points.

185.

Intersecting Lines

242

Angle between Two Lines Angle between Line and Plane

243

247

191-193.

Transformation of Co-ordinates The Cone and its Sections

194, 195.

Definitions.

186, 187. 188. 189, 190.

find

where a given Line pierces the Co-ordinate 238 239

Examples

246

Equation of a Conic.

CHAPTER

239

Examples

.

.

.

250 254

IV.

DISCUSSION OF SUHFACES OF THE SECOND OKDER. General Equation of the Second Degree involving three Variables. Transformations and Discussion .

The Ellipsoid and varieties The Hyperboloid of One Sheet and varieties The Hyperboloid of Two Sheets and varieties The Paraboloid and varieties Surfaces of Revolution.

Examples

.

.

259 262

.... .

.

.

265 267

269 273

PLANE ANALYTIC GEOMETRY.

CO-ORDINATES.

PAST

L

CHAPTElt

I.

THE CARTESIAN OR BILINEAR SYSTEM.

of objects are determined by other some objects whose positions are referring assumed as known. Thus we speak of Boston as situated Here the ob west. in latitude north, and longitude is referred are the equator and the Boston which to jects meridian passing through Greenwich. Or, we speak of Bos ton as being so many miles north-east of New York. Here the 1.

THE

relative

them

positions

to

reference are the meridian of longitude through and New York itself. In the first case it will be each observed, Boston is referred to two lines which intersect other at right angles, and the position of the city is located when we know its distance and direction from each of these objects of

New York

lines.

In like manner, if we take any point such as P x (Fig. 1) in the plane of the paper, its position is fully determined when we know its distance and direction from each of the two lines which intersect each other at right angles in and

X

Y

This method of locating points is known by the that plane. name of THE CARTESIAN, or BILINEAR SYSTEM. The lines of 1

PLANE ANALYTIC GEOMETRY. X, Y, are called CO-ORDINATE AXES, and, when read separately, are distinguished as the X-AXIS and the Y-AXIS. The point 0, the intersection of the co-ordinate

reference

axes, is called

the ORIGIN OF CO-ORDINATES, or simply the

ORIGIN.

x and y which measure the distance of the from the Y-axis and the X-axis respectively, are point P!

The

lines

FIG.

1.

the distance (x ) from called the co-ordinates of the point the Y-axis being called the abscissa of the point, and the dis f tance (y ) from the X-axis being called the ordinate of the point.

Keferring to Fig. 1, we see that there is a point in each of the four angles formed by the axes which would satisfy the conditions of being distant x from the Y-axis and distant 2.

y from the X-axis.

This ambiguity vanishes when we com In the case

bine the idea of direction with these distances.

of places on the earth s surface this difficulty is overcome by using the terms north, south, east, and west. In analytic geome are used to serve the same try the algebraic symbols -f- and

purpose.

All distances measured to the right of the Y-axis

CO-ORDINA TES. are

called positive

abscissas

measured

those

;

3 to

the

left,

negative ; all distances measured above the X-axis are called all distances below, negative. With this positive ordinates the co-ordinates of the become understanding, (V, y ) point Pj. ;

of

P

2,

(-

x

f

,

y

)

;

of

P (- x f

3,

s

f

y

)

;

of

P4

-y

r

(x

,

,

;

f

).

3. The four angles which the co-ordinate axes make with each other are numbered 1, 2, 3, 4. The first angle is above

the X-axis, and to the right of the Y-axis the second angle above the X-axis, and to the left of the Y-axis the third ;

is

;

angle is below the X-axis, and to the left of the Y-axis the fourth angle is below the X-axis and to the right of the ;

Y-axis.

EXAMPLES. 1.

Locate the following points

(2.

1,

-

(0, 1),

(-

1,

;

-

2), (3,

What

(-

4, 0), (0,

-

(0, 1),

3).

are the lengths of its sides ?

Ans.

Vl3,

The ordinates of two points are each them situated with reference

the line joining

Ans.

line

0),

Locate the quadrilateral, the co-ordinates of whose ver (2, 0), (0, 3),

5.

2,

- 4).

tices are,

4.

(-

1),

Locate the triangle, the co-ordinates of whose vertices

are,

3.

:

- 1), (1, 2), (2, 3), (3, - 4). (0, 0), (3, 0), (0,

The commoji joining them

abscissa of two points situated ?

is

=

5, 5,

b

;

V 13.

how

is

to the X-axis ?

Parallel, below.

a

;

how

is

the

6. In what angles are the abscissas of points positive ? In what negative ?

In what angles are the ordinates of points negative ? 7. In what angles positive ?

PLANE ANALYTIC GEOMETRY.

4

In what angles do the co-ordinates of points have like In what angles unlike signs ? signs ? 8.

9.

The base

X-axis and the origin

its

of an equilateral triangle coincides with the is on the Y-axis at the distance 3 below

vertex

required the co-ordinates of its vertices ? Ans. (i- V12, 0), (0, 3), (-i V"l2,

;

-

ordinate

is

and how

is it

situated ?

A

Ans.

straight line passing through the an ing angle of 45 with the X-axis. 11.

0).

moves that the ratio of its abscissa to its = 1, what kind of a path will it describe, always

If a point so

10.

The extremities

of a line are the points

origin,

and mak

1,

(2, 1), (

2)

:

construct the line.

=

the ordinate of a point is 0, on which of the If the abscissa is ? co-ordinate axes must it lie ? 12.

If

=

13.

Construct the points

the line joining them 14.

Show

is

2,

(

3), (2, 3),

bisected at

that the point (m, n)

and show that

(0, 0). is

distant

Vm + 2

?i

2

from

the origin. 15. Find from similar triangles the co-ordinates of the middle point of the line joining (2, 4), (1, 1).

Ans.

($,$).

THE POLAR SYSTEM. 4. Instead of locating a point in a plane by referring it to two intersecting lines, we may adopt the second of the two methods indicated in Art. 1. The point P 1? Fig. 2, is fully

determined when we know its distance PI X (= 0) from some given point tion P! to X. If we give all values from line values from

to 360

to

(= in oo

and direc some given to r, and all r)

easily seen that the position be located.

0, it is

of every point in a plane may This method of locating a point

is

called the

POLAR SYSTEM.

CO-ORDINA TES.

5

The point is called the POLE the line X, the POLAR Axis, or INITIAL LINE the distance r, the EADIUS VECTOR ; the angle 0, the DIRECTIONAL or VECTORIAL ;

;

The POLAR CO

ANGLE.

distance r and the angle ORDINATES of a point.

0,

(r, 0),

are called the

In measuring angles in this system, it is agreed (as in trigonometry), to give the positive sign (+) to all angles meas5.

FIG.

ured round to the sign

(-)

to those

2.

left from the polar axis, and the opposite measured to the right. The radius vector

considered as positive (+) when measured from the pole toward the extremity of the arc (0), and negative (-) when measured from the pole away from the extremity of the (r)

is

arc

(0).

A

few examples will make

points clear. If r 2 inches and

=

this

method

of locating

=

45, then (2, 45) locates a point PI 2 inches from the pole, and on a line making an ano-le of -f 45 with the initial line. If r

=-2

=

inches and 45, then (- 2, 45) locates a 2 inches from the pole, and on a line making an angle of 45 with the initial line also but in this case the point is on that portion of the boundary line of the angle which has been produced backward the point

P

3

;

If

r

=2

inches and 9

= - 45,

through then (2,

pole.

- 45)

locates a

PLANE ANALYTIC GEOMETRY.

6

below inches from the pole, and out on a line lying point P 4 2 it. with 45 of the initial line, and making an angle 45 ) 2, 45, then ( 2 inches and 6 If r the to pole), locates a point P 2 directly opposite (with respect the point

P4

,

-

-

=-

=-

- 45).

(2,

in analytic geometry oi express 6. While the usual method it and seconds (, minutes, in is degrees, ing an angle of terms in convenient to express angles frequently becomes radius of the is equal in length to the arc whose the angle UNIT. CIRCULAR is called the measuring circle. This angle ot centre the at that angles from ,

We

same

know

"),

geometry

each other as the arcs included between and ff be two central angles, we hence, if

circle are to

their sides;

have,

arc

_.

7

& Let

ff

= unit

angle

;

~"arc

then arc

=r

(radius

of

measuring

circle).

_ Hence

= 360,

Hence,

r

circular unit .

If

arc

-.

r 6

= arc

common r

X

360

X

circular unit.

measure, then arc

=2

TT

r

X

= 2irr.

circular unit.

Therefore the equation,

= 2 X circular

360

TT

unit,

...

(1)

between the two units of measure. expresses the relationship

j EXAMPLES. an What is the value in circular measure of From (1) Art. 6, we have,

1.

= 30 X 12 = 2 circular unit. 30 = - circular unit. TT

360 ..

angle of 30 ?

6

CO-ORDINA TES. 2.

What

7

are the values in circular measure of the following

angles ?

1, 45, 60, 3.

What 3

90,,

120, 180, 225, 270, 360.

are the values in degrees of the following angles?

^

2

4"

4

8

*

* 4"

4.

What

5.

Locate the following points

is

,

8

8"

6

the unit of circular measure ?

Ans.

(2,

(-

40),

4,

57, 17

,

45.".

:

90), (3,

-

135).,

^3,

(-

-

1,

180),

3

- 1, - *), 6.

2,

-

Locate the triangle whose vertices are, / 3 \ \ 5 1 -7T \, M, -TT 4 -

7. The base of an equilateral triangle (= a) coincides with the initial line, and one of its vertices is at the pole re quired the polar co-ordinates of the other two vertices. ;

(a>

8.

The polar

co-ordinates

of

a point

are ( 2, \

three other ways of locating the

same

point,

~\ 4

()).

Give

/

using polar

co-ordinates.

Ans.

9.

a.,

/

-

i

2

Construct the line the co-ordinates of whose extremities

PLANE ANALYTIC GEOMETRY.

8 10.

How

being ( 3,

is

-\

the line, the co-ordinates of two of f 3,

\

its

points

situated with reference to the initial

Ans.

Parallel.

Find the rectangular co-ordinates of the following points 11-

=

13

4,

\

:

LOCL

CHAPTER

II.

LOCI.

THE Locus OF AN EQUATION

7.

generatrix as

it

moves

is

the

path described by

its

in obedience to the laiv expressed in the

equation.

The EQUATION OF A Locus the

law subject

to

the algebraic expression of

is

which the generatrix moves in describing that

locus.

If we take any point P 3 equally distant from the X-axis and the Y-axis, and impose the condition that it shall so move ,

FIG.

that the ratio of

its

ordinate to

3.

its

abscissa shall always be

equal to 1, it will evidently describe the line algebraic expression of this law is It

x

and

is

The

=

1,

or y

P

3

Pi-

= x,

called the Equation of the Locus. line is called the Locus of the Equation.

P^

The

Again

:

PLANE ANALYTIC GEOMETRY.

10

we take the point P 4 equally make it so move that the ratio of

if

,

distant from the axes, and ordinate to its abscissa

its

at any point of its path shall be equal to 1, it will describe In this case the equation of the locus is the line P 4 P 2 .

y~

=

1,

x

and the 8. first,

line

P4 P 2

or y

=

x,

the locus of this equation.

is

be observed in either of the above cases (the for example), that while the point P 3 moves over the

It will

line P 8 PU its ordinate and abscissa while always equal are yet in a constant state of change, and pass through all values For this reason y and x are from oo, through 0, to -|- GO.

called the

VARIABLE

or

GENERAL CO-ORDINATES

of the line.

we

consider the point at any particular position in its x as at P, its co-ordinates ( ?/) are constant in path, of to this the point, and to and value, position correspond If

,

this position alone.

The

variable co-ordinates are represented

by x and

of the moving y, and the particular co-ordinates point for any definite position of its path by these letters with a dash or subscript or by the first letters of the Thus (x ?/), (x lt yi), (a, ), (2, 2) alphabet, or by numbers. correspond to some particular position of the moving point. ;

,

EXAMPLES. 1.

Express in language the law of which y

algebraic expression. Ans. That a point shall so

move

= 3x

-f-

2

is

the

in a plane that its ordinate

shall always be equal to 3 times its abscissa plus 2. 2.

A

3.

The sum

+

a quantity a is point so moves that its ordinate b to its abscissa a ; required the quantity always equal of the law. expression algebraic b. Ans. y -\- a \x a

of the squares of the ordinate and abscissa of a 2 what is the is always constant, and

=

moving point

equation of

its

path

;

?

Ans.

x2

+y

2

==

a2

.

LOCI.

11

Give in language the laws of which the following are

4.

the algebraic expressions

:

^-2,*= -6.

2y--.|. y

2

=

4

4 z2

z.

2 x 2 -f 3

2 ?/

= 6. if

= 2px.

a^f

5 y2

+

6

2

z2

= 18. =a6 2

2

.

9. As the relationship between a locus and its equation constitutes the fundamental conception of Analytic Geometry,

important that it should be clearly understood before We have entering upon the treatment of the subject proper. it

is

been accustomed in algebra to treat every equation of the x as indeterminate. Here we have found that this form y

=

equation admits of a geometric interpretation ; i.e., that it repre sents a straight line passing through the origin of co-ordi nates and making an angle of 45 with the X-axis. We shall find, as

we

dental,

which does not involve more than three variable quan

tities, is

proceed, that every equation, algebraic or transcen

susceptible of a geometric interpretation.

We

shall

geometric forms can be expressed alge braically, and that all the properties of these forms may be find, conversely, that

deduced from their algebraic equivalents. Let us now assume the equations of several loci, and let us locate and discuss the geometric forms which they represent. 10. Locate the geometric figure whose algebraic &wwdent is

y

We its

know

= 3x+2.

that the point where this locus cuts the Y-axis has in the equa 0. If, therefore, we make x

abscissa x

=

=

we

shall find the ordinate of this point. Making the substitution we find y 2. Similarly, the point where the

tion,

=

locus cuts the X-axis has

for the value of its ordinate.

Mak-

t+T*

PLANE ANALYTIC GEOMETRY.

12

=

=

in the equation, we find x ing y the axes and marking on them the points

we

will have

two points of the required

successively equal to 1, 2, 3,

-

1,

-

2,

- 3,

f.

locus.

Drawing now

Now make

x

etc.

in the equation, and

find the corresponding values of y. the result thus let us tabulate convenience

For

:

Values of x

Corresponding

_

2 3

"

"

-

1

_2

Values of y 5 8 11

1

FIG.

1

4

"

4.

a line through them we Locating these points and tracing have the required locus. This locus appears to be a straight

LOCI.

13

We

line and it is, as we shall see hereafter. shall see also that every equation of the first degree between two variables The distances Oa and Ob represents some straight line.

which the

line cuts off on the co-ordinate axes are called INTERCEPTS. In locating straight lines it is usually sufficient to determine these distances, as the line drawn through their extremities will be the locus of the equation from which their

values were obtained.

EXAMPLES. 1.

Locate the geometric equivalent of

I,-..-l_2* Solving with respect to y

y=-2x + The extremities

we

in order to simplify,

have,

2.

of the intercepts are (0,2),

(1,0).

Locating these points, and drawing a straight line through them, we have the required locus. Construct the loci of the following equations

5.

6.

12.

y

2y Is

=

cx

-cl

= 3x.

10.

i-

11.

X

the point (2, 1) 2x 3 ? Is y

=

-

-y=-3 y

:

_2x.

on the line whose (6, 9) ?

NOTE. If a point is on a line, the equation of the line.

its

Is (5, 4) ?

equation Is (0,

co-ordinates

must

is

- 3) ? satisfy

PLANE ANALYTIC GEOMETRY.

14 13.

Which

of the following points are on the locus of the

2 equation 3 x

(2, 1),

14.

+2y = 6? 2

(V2,

(0,V3),

0),

=

2x

1,

Construct the lines y

show by similar

11.

-2x = 3y

0), (2,

V3)

line

6.

Construct the polygon, the equations of whose sides are

y 16.

(- V2,

1, 3),

Write six points which are on the - y

15.

(-

Discuss

y

=

= sx

a;,

= 5.

y

and y

-f-

=

saj

+ 4,

and

triangles that they are parallel.

and x*

construct the equation

+ y* =

Solving with respect to

y =

=t

:

16.

y,

we

Vie - X

have, 2-

The double sign before the radical shows us that for every we assume for x there will be two values for y, equal

value

and with contrary

This is equivalent to saying that signs. for every point the locus has above the X-axis there is a cor responding point below that axis. Hence the locus is symmet

with respect to the X-axis. Had we solved the equation with respect to x a similar course of reasoning would have shown us that the locus is also symmetrical with respect to the Y-axis. Looking under the radical we see that any value of x less than 4 (positive or negative) will always give two real values for y ; that x 0, and that any J- 4 will give y = value of x greater than -j- 4 will give imaginary values for y. Hence the locus does not extend to the right of the Y-axis x 4. farther than x -f- 4, nor to the left farther than rical

=

=

Making

= 0, = 0, y

x

we have y "

"

x

= =

_j_

4

-j-

4.

LOCI.

15

Drawing the axes and constructing the points, - 4), (4, 0), ( 4, 0), we have four (0, 4), (0, points

of

the locus

i.e.,

;

B,

B

,

A, A!. Y B

FIG.

Values of x

5.

Values of y

Corresponding

+ 3.8 and 3.8 + 3.4 and - 3.4 + 2.6 and - 2.6

1

2 3

4

-1 -2 -3 -4

+ 3.8 and - 3.8 + 3.4 and 3.4 + 2.6 and - 2.6

Constructing these points and tracing the curve,

we

find

it

to be a circle.

This

might readily have been inferred from the form

of the equation, for

we know

that the

sum

of the squares

PLANE ANALYTIC GEOMETRY.

16 the

(OC) and ordinate (CP^ of any point Pj equal to the square of the radius (OPj). We might, therefore, have constructed the locus by taking the origin as centre, and describing a circle with 4 as a radius. NOTE, x for any assumed value of y, or y -JL _j_ 0, of

abscissa

in the circle

is

=

=

any assumed value of x always indicates a tangency. Re ferring to the figure we see that as x increases the values of y decrease and become -j- when x = 4. Drawing the line for

represented by the equation x to the curve.

We

= 4,

we find that it is tangent we proceed that any two

shall see also as

coincident values of either variable arising from an assumed or given value of the other indicates a point of tangency. 12.

Construct and discuss the equation 9 x2 16 y 2 144.

=

+

Solving with respect to

we have

?/,

-

144

9 x2

16

= y =

x

X

"

Drawing the axes and laying four points of the locus

Values of x 1

2 3 4 1

3 4

;

= =

gives y

i.e.,

B,

off

B

,

-j-

;

these distances,

A,

Corresponding "

3

4-4:.

A

.

Values of y

4 2.9 and - 2.9

"

+2.6

((

_i_

"

we have

Fig. 6.

2

+2.9

"

-r-

2

2.6

"

2

"

2.9

"

"

-

2

"

-J-

Locating these points and tracing the curve through them, the required locus. Referring to the value of y we see from the double sign that the curve is symmetrical with

we have

respect to the X-axis.

The form

of the equation (containing

LOCI.

17

of the variables), shows that the locus only the second powers with Looking is also respect to the Y-axis.

symmetrical

FIG.

6.

under the radical we see that any value of x between the 4 will give two real values for y ; and that 4 and limits will give imaginary values for any value beyond these limits Hence the locus is entirely included between these limits. y. with which we shall have more to do hereafter, This

+

is

curve, called the ELLIPSE.

13.

Discuss and construct the equation 2 2/

Solving,

=

we have

is symmetrical with respect to the of the as and equation contains only the first power X-axis, As to the Y-axis. with not it is that respect symmetrical x, will always give real values for y, every positive value of x the locus must extend infinitely in the direction of the posi tive abscissae and as any negative value of x will render y

We

see that the locus

;

PLANE ANALYTIC GEOMETRY.

18

imaginary, the curve can have no point to the left of the hence the curve Making x 0, we find y -j- 0; passes through the origin, and is tangent to the Y-axis.

=

Y-axis.

Making y

= 0,

we

=

find

x

=

;

hence the curve cuts the

X-axis at the origin.

Values of x

Corresponding

1

+

"

2 3 4

From is

Values of y 2 and

+2.8

+ 3.4 +4

these data

we

-2 - 3.4 -4 -2.8

easily see that the locus of the equation

represented by the figure below.

FK;.

This curve 14.

is

called the

7.

PARABOLA.

Discuss and construct the equation 4 z2 9 ?2 36

=

Hence

We see from the form of the equation that the locus must be symmetrical with respect to both axes. Looking under

19

LOCI.

the radical, we see that any value of x numerically less than 3 will render y imaginary. Hence there is no _j_ 3 or We see also that point of the locus within these limits.

+

any value of x greater than values for

The

y.

3 or 3 will always give real locus therefore extends infinitely in the

direction of both the positive 3. limits x

=i Making x = 0,

we

find

y

and negative

=

-J-

2

V

1

abscissae

;

from the

hence, the curve

does not cut the Y-axis.

Making y

= 0,

we

find

the X-axis in two points

Value of 4

x.

x

=

(3, 0), (

-j-

3

;

hence, the

Corresponding.

Values of y

5

+ + 2.6 + 3.4 + 1.7 + 2.6

-6

4-3.4

1.7

5 6

-4

FIG.

This curve

is

called the

curve cuts

3, 0).

8.

HYPERBOLA.

and "

"

"

- 1.7 - 2.6 - 3.4 - 1.7 -2.6

"

- 3.4

PLANE ANALYTIC GEOMETRY.

20 15.

We

have in the preceding examples confined ourselves

RECTANGULAR equations of equations whose loci were referred to rectangular Let us now assume the POLAR equation axes. to the construction of the loci of

;

i.e.,

=

r

and discuss and construct

Assuming values

for

-

6 (1

cos 0)

it.

6,

we

find their cosines

convenient table of Natural Cosines. values,

we

Substituting these

find the corresponding values of

Values of 6

Values of cos 1.

.86

6

.50

6 6

160

- .50 - .94

180

-

200 240

1.

-

.94 .50

270

(1 (1 (1

6 (1

30

90

r.

Values of r

60

120

from some

1

)

.86) .50) )

+ .50) + .94) 6 (1 + 1 6 (1 + .94) 6 (1 + .50) 6 (1 6

(1

6 (1

)

)

300

.50

6 (1 -- 50)

330

.86

6 (1

-

.86)

= = .84 = 3. = 6. = 9. = 11.64 = 12. = 11.64 = 9. = 6. = 3. = .84

as the the initial line OX, and assume any point a series of lines, making the draw this point Through pole. assumed angles with the line OX, and lay off on them the corresponding values of r. Through these points, tra the have we required locus. cing a smooth curve,

Draw

LOCI.

FIG.

This curve, from 16. Discuss

and

its

A

9.

heart-like shape,

is

called the CARDIOID.

construct the transcendental equation

y NOTE.

21

= log x.

transcendental .equation

is one whose decree power of analysis to express. Passing to equivalent numbers we have = x, when 2 is the base of the system of logarithms selected. As the base of a system of logarithms can never be nega tive, we see from the equation that no negative value of x can Hence the locus has none of its satisfy it. to

ti-anscends the

&

the left points the other hand, as every positive value of x will give real values for y, we see that the curve extends the direction of the infinitely of the Y-axis.

m

If

y = 0,

On

positive abscissae

then

PLANE ANALYTIC GEOMETRY.

22 If

x

=

0,

then 2*

The

=

.-.

y

= log

.-.

y

=

co.

at a unit locus, therefore, cuts the X-axis

s

distance on

the positive side, and continually approaches the Y-axis with out ever meeting it. It is further evident that whatever be the base of the system of logarithms, these conditions must x. hold true for all loci whose equations are of the form a?

=

Values of x 1

2

Corresponding

Values of y

"

1

"

4

"

8

"

2

3

-

.5

1

2

"

.25

traced through them will Locating these points, the curve

be the required locus.

FIG.

10.

This curve is called the LOGARITHMIC Curve, being taken from its equation.

its

name

LOCI.

23

The preceding examples explain the method employed

17.

in constructing the locus of any equation. While it is true that this method is at best be made approximate, yet it

may

sufficiently accurate for all practical purposes by assuming for one of the variables values which differ from each other

by very small quantities. It frequently happens (as in the case of the circle) that we may employ other methods which are entirely accurate. In the discussion of an equation the first step, usually, it with respect to one of the variables which enter

18. is

to solve

into

it.

The question

of which variable to select

is

immate

rial in principle,

yet considerations of simplicity and conven ience render it often times of The sole great importance. difficulty, in the discussion of almost all the higher forms of If this difficulty can equations, consists in resolving them. be overcome, there will be no trouble in tracing the locus and discussing it. If, as frequently happens, no trouble arises in the solution of the equation with respect to one of the vari

then that one should be selected as the dependent variable, and its value found in terms of the other. If it is equally convenient to solve the equation with to either ables,

respect

of the variables

selected

which enter into

it,

whose value on inspection

then that one should be will afford the

discussion.

EXAMPLES. Construct the loci of the following equations 1.

2.

+ l-0.

2jr-4 y

-., 2

3.

2y

4.

4aj 2

10.

= 10.

+ 5s -9 =-36. 2

5.

fi+4

6.

**+y-

7.

r

8.

35

2

a

?/

:

= ^ cos = logy. 2

simpler

PLANE ANALYTIC GEOMETRY.

24

Construct the loci of the following 9 10. 11.

si.y^O. x + 2 ax + a = 0. x - a* = 0. 2

2

2

12

yi_9 = 0.

13.

7/

2

-

2 xy

+ x - 0. 2

:

14.

x*-x-6 = 0.

15.

or -j-

2

2

x

6 ==

+4x - 5

0.

-

16.

x

17.

x*-7x + l2 = 0.

18.

x2

0.

+ 7 z + 10 =

Factor the first member: equate each factor NOTE. and construct separately.

0.

to 0,

THE STRAIGHT LINE.

CHAPTER

25

III.

THE STRAIGHT

LINE.

19. To find the equation of a straight line, given the angle which the line makes with the X-axis, and its intercept on the Y-axis.

FIG.

11.

Let C S be the line whose equation we wish to determine. Let SAX and OB b. Take any point P on the line

= PM

=

to OY and BN to OX. Then (OM, MP) = (x, y) are the co-ordinates of P. From the figure PM = PN + OB = BNtan PEN +

and draw

||

||

BN = OM = x, ..

and tan PBN = tan SAX = tan Substituting and letting tan a = s, we have, (I)

.

b,

but

PLANE ANALYTIC GEOMETRY.

26

Since equation (1) is true for any point of the line SC, it true for every point of that line hence it is the equation of the line. Equation (1) is called the SLOPE EQUATION OF THE is

;

STRAIGHT LINE

COROLLARY

If

= tan b = in

=

sx

s

;

1.

y

(

.

(1),

.

.

called the slope.

is

)

we

have,

(2)

a line which passes through the

for the slope equation of origin.

COR.

=

If s

2.

y which

is,

as

it

we have

in (1),

=b

ought to

be, the

equation of a line parallel to

the X-axis.

COR.

3.

If

=

s

oo,

=

then

90, and the

line

becomes

parallel to the Y-axis.

Let the student show by an independent process that the equation of the line will be of the form x = a.

SCHOLIUM. We have represented by the angle which the makes with the X-axis. As this angle may be either acute or obtuse, s, its tangent, may be either positive or nega

line

tive.

The

line

may

also cut- the Y-axis either above or below

the origin; hence, b, its Y-intercept, From these considerations negative.

y

=-

sx

y

-{-

it

appears that

+b

represents a line crossing the == sx

be either positive or

may

first

angle

;

b

represents a line crossing the second angle

y =

sx

b

represents a line crossing the third angle

y

= sx

;

b

represents a line crossing the fourth angle.

;

THE STRAIGHT

LINE.

27

EXAMPLES.

The equation of a and intercepts.

1.

slope

line

Solving with respect to

Comparing with

=

we

y,

22 1

=

y

x H

,

(1)

Art.

Making y

Y-intercept.

is

2y

+x=3

;

required

its

have,

3 .

19,

we

=

1

and

2

=

= 3 = X-intercept.

find s

in

the

equation,

b

=

^

2

we have

=

3. Construct the line 2 y -\- x The points in which the line cuts the axes are

2.

O,

?V

and

(3,

3).

-*/

Laying these points

off

on the axes, and tracing a straight

through them, we have the required

line

thus

~F-+fLay

off

OB =

b

=

|

;

draw

BN OX and make it = 2, also NP OY and make it = + 1. ||

||

The

line

through

P and B

the required locus. T^I^T

-*

For

=

locus.

solving the equation with respect to

:

tan

BAX. .-.tan

BAX

is

y,

we

Or otherwise have,

PLANE ANALYTIC GEOMETRY.

28 3.

=

x

Construct the line 2 y

we

Solving with respect to y,

have,

Lay ||

to

also

A

3.

off

OX NP

||

BO = 5 = |

and make it = 2 to OY and make

straight line

BK

Draw

.

draw

;

it

=

1.

P and B

through

will be the required locus.

= - = tan PBN = tan

For

BAX =

Hence, in general,

s.

Y

to the left of

BN

is

laid off to the right or

according as the coefficient of

x

is fiositive

or negative.

Give the slope and intercepts of each of the following and construct

lines

:

4.

2

Ans. 5.

s

=

,

&

=

*

= - 2, a = - -

1,

^^

x-

6.

.

* ==

- 12, 8.

y

~ 3

+2x = 1 -

.

y.

9.

_

a and 6 in the answers above denote the X-intercept NOTE. and the Y-intercept, respectively.

THE STRAIGHT What

14.

29

angle does each of the following lines cross ? 10.

y

= 3x +

11.

y

= - x + 2.

12.

1.

y

=2

a;

1.

\<^

Construct the figure the equation of whose sides are

2y 15.

LINE.

+ s-l = 0, 3^=2x + 2, y = - x -

I.

Construct the quadriiftfc&rai the equations of whose sides

are

x

= 3,

y

=

x

+

y

l,

= 2,

x

= 0.

20. To find the equation of a straight line in terms of

its

intercepts.

Y

FIG.

Let S C be the

line.

Then OB

OA =

12.

a

= b = Y-intercept, = X-intercept.

The slope equation of Art. 19. equation (1), y

a line

=

sor

and

we have determined

+

/>.

to be

PLANE ANALYTIC GEOMETRY

30

From

the right angled triangle

tan

GAB =

tan

AOB, we

BAX =

-

s

have,

= OB OA

.-.-.--*. a

Substituting in the slope equation,

This

is

called the

we

have,

SYMMETRICAL EQUATION

of the straight

line.

COR.

1.

+ and b +, then we have, -= 1, for a line crossing the

If a

ni

rvt

\-

first

j-

and

If a

b

+, then

---f-^ = lisa line crossing the a

If

and

a

b

,

b

a

-f-

and

--

^

a

b

b

,

=1

is

a line crossing the third angle.

then

----- *-

a

second angle.

then

-

If

angle.

=1

is

b

a line crossing the fourth angle.

EXAMPLES. 1.

Construct

NOTE. Y-axis.

^

=

1.

Lay off 3 units on the X-axis and Join their extremities by a straight

2 units on the line.

Across which angles do the following lines pass

?

THE STRAIGHT

LINE.

31

Give the intercepts of each, and construct.

+ |=1.

2-

|

/v,

q

\

Q o

7/ if

=

4.

i*^ *

1

^ ^

57^ =

n

3

"Q

y

r>

-i -

1

^^

AXV-~)C7^-

z--<r^-

-

K"

Write the slope equations construct

-f~V=l.

the

of

following lines, and

:

=

x

6.

o

r-IS-7. 3

.

s.

y 2

T ?.6 = _ +

i.

y-ix-2. O .

^

^

-J

O J

5 10.

Write y

=

e

+ 6 in a symmetrical ."

7-

;

=

form.

1 1.

/-t

A O*^"

/>

J

Given the following equations of straight their slope and symmetrical forms

lines, to write

:

11.

+ 3z-7 = z + 2. ^^ = ~ 3

2y

23

13.

a?

12.

.

14

2? x

=

43

~

V

_2x-l

PLANE ANALYTIC GEOMETRY.

32

To find the equation of a straight line in terms of the perpendicular to it from the origin and the directional cosines 21.

of the perpendicular.

The Directional cosines of a line are the cosines it makes with the co-ordinate axes.

NOTE.

the angles which

FIG

Let OS Let

be-

the

line.

AOP = triangles AOP and BOP, we OA = -OP- OB = OP

OP = p, BOP

From

the

13.

==

.

7,

have

,

COS 7

COS

that

a

is,

=

P

7.

*

,

b

=

P

^

cos w

cos 7

Substituting these values in the symmetrical equation, Art. 20. (1),

_

a

+ y-b =

x cos

which

is

1,

we

have, after reducing,

+ y cos 7 = p

the required equation.

.

.

.

(1)

of

THE STRAIGHT Since

=

y

90

,

This form

and

COR.

and the COR.

.

.

NORMAL EQUATION

(1)

of the straight line.

becomes parallel to the Y-axis.

line

= 90,

If

2.

.

= 0, then x =p

If

1.

;

more frequently met with than that given in

is

called the

is

= sin hence + y sin = p (2)

cos y

cos

cc

33

LINE.

then

y*=p and the

If x

22.

becomes parallel to the X-axis.

line

+ y sin = p be the equation of a given line, + y sin a = p -- d is the equation of a parallel

cos

then x cos

For the perpendiculars p and p j- d coincide in direction line. hence the lines since they have the same directional cosines to which they are perpendicular are parallel. ;

COR.

Since

1.

d

p it

is

evident that d

therefore, (x

the origin

,

p

is

y

)

-J-

-p = is

the distance between the lines.

_j_

If,

be a point on the line whose distance from d,

we have

+ y sin = p = x cos d y

x cos u .-.

d

"

-\-

-J-

d.

sin

*

p

.

.

.

(1)

a point (x y ) from the line found -\- y by transposing the constant term to the first member, and substituting for x and y the co Let us. for example, find the ordinates x y of the point. distance of the point ( y~3, 9) from the line x cos 30 y sin

Hence the distance sin a

x cos a

=p

of

,

is

,

+

30

=

5.

From

(1)

d

= V3 cos 30

-f 9 sin 30

-

5

PLANE ANALYTIC GEOMETRY.

34

From .*.

Fig.

p

sin= / =

,

b

-2

_L

7

t=

<2

/

(

y

i

|-

a

i

1

^b

+6

Vtt 2

2

the expression for the distance of the point (x

line

-

J/

-

a

=

Hence is

=

13 we have cos

whose equation

is

of the form -

a

+b=

r

?/)

,

from a

1.

Let the student show that the expression for d becomes

VA + B 2

when the equation

of the line

See Art. 24, Equation

2

given in

is

general form.

its

(1).

EXAMPLES. 1.

line

The perpendicular Kit makes an arfgle

= 5 and

equation of the

fall

from the origin on a straight with X-axis required the

of 30

;

line.

V3

Ans.

sc

+ = //

2. The perpendicular from the origin on a straight makes an angle of 45 with the X-axis and its length =

required the equation of the

line.

Ans. 3

What

is

the distance of the point

(2.

Ans.

.

Find the distance of the point from the following cases

x -\-y

4) from the

x

From

(2,

5.

From

(3, 0)

to

6.

From

(0, 1)

to 2

7.

From

(a, c) to

5) to

-

=

-

1.

?/

= - x = 2. 1.

|

y

=

SJT

+

5

= 2. line

line in each of the

| |

line

V2

i.

:

4.

10.

b.

^^

THE STRAIGHT LINE.

35

23. To find the equation of a straight line referred to oblique axes, given the angle between the axes, the angle which the line makes with the X-axis and its Y-intercept.

NOTE.

Oblique axes are those which intersect at oblique

angles.

Y

1

M N>

FIG. H. %

it

Let CS be the line whose equation we wish to determine, being any line in the plane YOX. Let YOX = ft SAX = OB = b. Take any point P on the line and draw PM to OY and ON to SC ,

From

NP = OB = b.

== ,

the figure

y

From

;

||

||

PM = y, OM = a, NOX

then,

triangle

= MN + NP = MN + b

ONM, we

MN

sin

OM

sin

.

.

.

(1)

have,

NOM MNO sin

sin

Substituting the value of (1),

we

drawn from

.M"N"

this equation in

have,

y

Sin

= sin

(/?

"

-

x )

+b

...

(2)

PLANE ANALYTIC GEOMETRY.

36

This equation expresses the relationship between the co Bnt as the point ordinates of at least one point on the line. selected was any point, the above relation holds good for every point, and is, therefore, the algebraic expression of the law which governed the motion of the moving point in de It is therefore the equation of the line.

scribing the line. COR. 1. If b

= ?/

0,

then sin

=

-

sin (p

x

.

.

.

(3)

a)

the general equation of a line referred to oblique axes passing through the origin. is

COR.

2.

If b

= y

=

and

=o

.

.

.

then

0,

(4)

the equation of the X-axis.

COR.

3.

If

=

ft

and

=

x

.

=

/3 .

.

;

y

But tan This 19

is

<t

=s

5.

.

= tan n x y = sx

if

-J-

-\-

the axes are

made

rectangular,

ft.

ft-

See Art.

the slope equation heretofore deduced.

(1).

Cor.

.

then

(5)

the equation of the Y-axis. COR. 4. If (3 = 90 i.e.,

then

,

If

/3

= 90 and = 0, then y = sx. See Art. 19, Cor. ft

1.

EXAMPLES. makes an two units

Find the equation of the straight line which 1. angle of 30 with the X-axis and cuts the Y-axis distant from the origin, the axes

making an angle

with each other. Ans. 2.

had been assumed rectangular what would have been the equation ?

If the axes

ple above,

Ans.

y

of

=x

in the

=

-JL-

60

-f-

2.

exam

+ 2.

THE STRAIGHT LINE.

37

The

co-ordinate axes are inclined to each other at an of angle 30, and a line passing through the origin is inclined to the X-axis at an angle of 120, required the equation of the 3.

line.

Ans.

24. is

y y

= f\

,/q vo

Every equation of the first degree between two variables

the equation of a straight line.

Every equation of the first degree between two variables can be placed under the form

Ax in

y,

+ By + C

-

.

which A, B, and C may be either Suppose A, B, and C are not zero.

we

and whose slope

from which If

(1)

finite or zero.

Solving with respect to

with

(2)

(1) Art.

the equation of a straight line -

.

have,

Comparing equation is

.

A = 0,

it

then

s

=

was derived ?/

=

is

A

;

.19,

we

see that

whose Y-intercept

hence

(1),

the

b

it

=

equation

the equation of a straight

line.

*

B

the equation of a line parallel to the X-axis. If

B

= 0,

then x

=

A

the equation of a line parallel to the Y-axis. If

C

= 0,

then y

=-

A B

X)

the equation of a line passing through the origin. Hence, for all values of A, B, C equation (1) is the equa tion of a straight line. ^

J

PLANE ANALYTIC GEOMETRY.

38 25.

To find the equation of a straight

line

passing through

a given point.

Let (V, y

)

be the given point. is to be straight,

its

Since the line

y = sx + b

which

in

s

and

...

equation must be

(1)

be determined.

b are to

of a line expresses the relationship which of every point on it; hence co-ordinates the exists between when the co-ordinates of any satisfied be must its

Now, the equation

equation for the general co-ordinates x and point on it are substituted the We equation of condition. have, therefore, y.

+

=sx

y

But a straight

...

b

(2)

cannot in general be made to pass cut off a given distance (b) on

line

through a given point (x /), the Y-axis, and make a given angle (tan. = s) with the X-axis. We must therefore eliminate one of these requirements. By ,

subtracting (2) from

y

which

is

COR.

-

(1),

we have,

=s

y

(

-

x

x

(

}

3)

the required equation.

1.

If

x

=

y

0,

-

then

= SX

y

.

.

.

(4)

on the the equation of a line passing through a point Y-axis. COR. 2. If y 0, then

is

=

y is

=s

(x

x

.

.

.

(5)

the X-axis. the equation of a line passing through a point on

COR.

3.

If

x

= y

and y

= 0,

then

= sx

the equation (heretofore through the origin. is

)

determined), of a line passing

THE STRAIGHT

LINE.

39

EXAMPLES. 1.

Write the equation of several lines which pass through

the point 2.

is the equation of the line which passes through and makes an angle whose tangent is 2 with the

2),

(1,

(2, 3).

What

X-axis ?

Ans. y

A

3.

straight line passes through ( with the X-axis. What

angle of 45

1, is its

3),

=

2 x

4.

and makes an

equation ? Ans. y

=x

2.

Required the equations of the two lines which contain and 60, respectively (a, b), and make angles of 30

4.

the point

with X-axis. Ans.

y 26.

y

-b = ^.(x-a).

^

,

(x",

Since the line

which

s

and

we

line passing

through

be the given points.

straight its equation

= sx + b

b are to

Since the line y"),

y")

is

y

(x",

V3

To find the equation of a straight

two given points. Let (x r y ),

in

-b = ?L=JL

y

.

.

.

must be

(1)

be determined.

required to pass through the points (x have the equations of condition.

y y"

is

= =

8X #t"

+ +

b

*

.

.

.

,

y ),

(2)

...

(3)

As a straight line cannot, in general, be made to fulfil more than two conditions, we must eliminate two of the four con ditions expressed in the three equations above. Subtracting (2) from (1), and then (3) from (2), we have, y y

- y = s (x - x - = s (x y"

f

) x")

PLANE ANALYTIC GEOMETRY.

40

member by member, we

Dividing these,

y

y

_

x

tf-y"

Hence

y

-

y

=

-x"

^^

-

(x

x

.

.

)

(4)

.

OC

3C,

is

have,

~ xx

the required equation. r then COR. 1. If y

= -y = y y",

which

is,

COR.

tf>

it

=

2.

is

=

or y

should be, the equation of a line then If x

as

to the X-axis.

||

x",

x

x

which

>

0,

or x

the equation of a line

||

=

x

1

,

to the Y-axis.

EXAMPLES. 2, 8) required both Given the two points ( 1, 6). ( line the passing through the slope and symmetrical equation of 1.

;

them. Ans.

y

vertices of a triangle are required the equations of its sides. 2.

The

=(-

+ 4,

2 x

(-

2, 1),

| 3,

+|=

-4)

1.

(2, 0)

= 5 X + 11 oy = 8 -\4:X (y + x = 2. (

Ans.

y

Write the equations of the lines passing through the points 3.

Ans. 4.

?/

(1,4), (0,0)

Ans. 5.

6.

(-2,3),(-3,-l)

y

(0,2),(3,-1) Ans.

= 4x-f-ll. 7.

7 y

2.

+ 2 x = 24.

(-3,0)

Ans. y 8.

+ =

(2,0),

= 0.

(-l,-3), (-2,4) Ans.

:

(-2,4)

Ans.

= 4 x. x

(5,2),

;

y

+ 7 x + 10 = 0.

THE STRAIGHT LINE. 27.

To find the length of a

line joining

FIG.

Let (x

f

,

i/),

(x",

y"}

41

two given points.

15.

be the co-ordinates of the given points

L = FT" = required length. Draw and P A to OY, and PC to OX. We see from the figure that L is the hypothenuse of a

P,

P".

P"B

||

||

right

angled triangle whose sides are

PC P"C

Hence, PT"

= AB = OB - OA = - x - EC = if - y = x"

P"B

=L=

1.

If

L

~

- xY

fa"

x and y the origin, and we have Con.

2 V*"

for the distance of a point

,

and

.

= 0,

+

2 y"

+

(y

r::~y lY

the point

...

from the

P

.

-

-

(l)

coincides with

(2)

origin.

EXAMPLES. Given the points (2, 0), ( 2, 3) required the distance between them also the equation of the line passing through them. Ans. L = 5, 47/-f3;r =6. 1.

;

;

PLANE ANALYTIC GEOMETRY.

42 2.

what

The

vertices of a triangle are are the lengths of its sides ?

(2, 1)

1,

(

2)

Give the distances between the following points 0)

(2, 3), (1,

(4,

-

5), (6,

(-

(0, 2),

Ans. 11.

What

is

Ans. Area 28.

To

;

(-

2,-l),

9.

(a,

ft),

10.

(-

2, 3),

:

(0, 1)

;

(2,

0)

d)

(c,

V5.

(-

a, 6).

2.

the expression for the area of a triangle whose (V, y

),

(a",

-

= \ \x ./iTicZ

8.

0)_

0)

(0, 0), (2,

vertices are

0)

V20. 1,

Ans. 6.

(-

3,

-_!)

^ws. 5.

3,

VlO.

Ans. 4.

7.

2)

-

V8, VlOJ V2fr

Ans.

3.

(

(y"

y

")

(*

y")>

+

",

y

^Ae intersection

? ")

- /) +

f *"

(y"

xf

1

"

(y

y")]-

of two lines given by their

equations.

Let

y y

= sx =s +6

-f- b,

and

a:

be the equations of the given lines. Since each of these equations is satisfied for the co-ordinates of every point on the locus it represents, they must -ttt-tke samer-tim* be satisfied for the co-ordinates of their point of Hence, for the intersection, as this point is common to both. So the equations are simultaneous. co-ordinates of this

treating them,

we

pointfind b

s b

b

sb

for the co-ordinates of the required point.

EXAMPLES. 1.

Find the intersection of y

= 2x + 1

and 2 y Ans.

= x - 4. (-2, -3).

THE STRAIGHT LINE. 2.

The equations

43

of the sides of a triangle are

required the co-ordinates of

its vertices.

(MH-i, \o oj

Ans.

7

\

Write the equation of the

= ]

which shall pass through and 3 y x 8=0, and make an angle with the X-axis whose tangent is 4. 3.

the intersection of 2 y

3x

-f-

line

-f-

2

=

Ans. 4.

What the

=

x -f

2,

y

= 3 x -f 2, Ans.

5.

x2

The equation 2

-f-

?/

=

= 4 x -|- 10.

are the equations of the diagonals of the quadri equations of whose sides are y x-\-l =Q,

lateral

y

y

10

is

y

+2x+2 =

and y

23 y

9

;c

?

+ 2 = 0, 3y

A*

A

and

a;

=

6.

whose equation is required the length of the chord.

of a chord of the circle

=x

-f-

2

;

Ans. 29. If

30 x

+ By -f C = +By+C =

.

.

.

.

.

L

(1) .

(2)

be the equations of two straight lines, then

Ax -f By + C

+K

(A a;

+ B y +C = )

...

(3)

(K being any constant quantity) is the equation of a straight line which passes through the intersection of the lines repre sented by (1) and (2). It is the equation of a straight line because

it

variables.

an equation of the first degree between two See Art. 24. It is also the equation of a straight

is

which passes through the intersection of (1) and (2). it is obviously satisfied for the values of x and y which simultaneously satisfy (1) and (2). Let us apply this principle to find the equation of the line which contains the point (2, 3) and which passes through the intersection of y = 2 x -\- 1 and 2 y + a; = 2. line

since

PLANE ANALYTIC GEOMETRY.

44

From

(3)

we have y

2a

l

+ K(2y +

sc

2)=0

for the equation of a line which passes through the intersec But by hypotheses the point (2, 3) is tion of the given lines.

on

this line

;

hence 3

4-1 + K K

Substituting this value for

we

(6

+ 2-2) =0

have,

o

x

y

or,

1 ==

Let the student verify this result for the required equation. by finding the intersection of the two lines and then finding the equation of the line passing through the two points. 30.

To find the angle between

given by their

lines

tivo

equations. M,

Yrf

FIG.

Let

y y

16.

= sx +- and = sx+ V />.

be the equations of s

1

SC and MN,

= tan

and

s

respectively

=

tan

.

;

then

THE STRAIGHT LINE. From

45

the figures

= .

,

(f

(f

=

-\-

a

u

a.

From trigonometry, tan

.-.

cp

)

(

= tan

= tan -

1

-

.

.

.

1 -f ss

COR.

1.

If s

= <p

tan a

s

,

(2).

then

= tan -1

= 0.

.. qp

the lines are parallel.

COR.

2.

If 1

+ = ss

(f

/.

= tan

substituting;

Or,

.*,

cp

then

0,

= tan -

1

oo

.-. g>

= 90

the lines are perpendicular.

These results may be obtained geometrically.

SCHOL.

If the lines are parallel, then, Fig. 16,

If they are perpendicular

= 90 .-.

= s = tan

tan a

.-.

1

+ (90

+ ss = 1 tan 1-1 = 0. -{-

a

-f

)

tan

= -

cot

=

.

tan a

=

1 4- tan

=

|

tan

EXAMPLES. 1.

What

and 2y

is

the angle formed by the lines y

x

1

+ 2cc + l=0?

y

=

= 90.

PLANE ANALYTIC GEOMETRY.

46

2. Required the angle formed by the 8 = 0. 6x and 2 y

+

3.

(2,

Ans.

3x y Perpendicular to 2 ?/

5

(b)

= 0.

= 0.

3x

-

5=0. 2 y

= 8,

(5)

3y

-j-

2x

= 1.

Given the equations of the sides of a triangle a -f- 2 and y = 3 required. l, y = 2#

+ y=

(a)

q>

Required the equation of the line which passes through 1) and is

Ans. (a) 3 x 4.

=

2

y -\-ox

+

(a) Parallel to 2 (b)

lines

;

The angles of the triangle. The equations of the perpendiculars from

vertices to

sides. (c) 5.

The lengths

What

relation exists

y y y y 6.

What

of the perpendiculars.

between the following

lines

:

= SX + = SX 3. = SX -\- 6. = SX + m. b.

relation exists between the following:

y y

= sx -[= sx

b.

-j- c.

Find the co-ordinates of the point in which a perpen 1 = 0. 2 a; dicular through ( 2, 3) intersects y 7.

+

,. 8.

Find the length of the perpendicular

origin on the line

2y-\-x

(,r

let fall

from the

L

= - V80.

= 4.

Ans.

o

=

be the equations of three straight lines, and /, m, and n be three constants which render the equation

B ty-j-C"

THE STRAIGHT I

+ By + C) + m

(Ax

C")

+

LINE.

47

+

+ Wy -f

+

By n (Af x CO (Afx an identity, then the three lines meet in a point.

=

Find the equation of the bisector of the angle between Ax + By + C = and Afx -f B y -f C = 0. Ax + By = __ (A x +j^y + CQ

10.

the two lines

+_C "

VA + B 2

VA + B

2

2

2

GENERAL EXAMPLES.

A

1.

straight line makes an angle of 45 with the X-axis off a distance 2 on the Y-axis what is its equation

=

and cuts

when

;

the axes are inclined to each other at an angle of 75 ? Ans. y -Y/2 x -j- 2.

=

2.

Prove 3

y 3.

a;

-f-

that

f

If

the

lines

y

=x (1,

-{- 1,

3 intersect in the point

(x

y

,

)

and

tremities of a line,

ordinates of

its

(x",

y"}

show that

y

=2x+2

and

0).

are the co-ordinates of the ex I

t~,

S-

1_2.

a rf the co j

middle point.

of the sides of a triangle are y = x -f- 1, x 1 required the equations of the sides of the triangle formed by joining the middle points of the sides 4.

a;

The equations

= 4,

y

=

;

of the given triangle.

(y Ans.

lyo ^

5.

=

=x ,

JL

_

x

+4 4

q O.

Prove that the perpendiculars erected at the middle

points of the sides of a triangle meet in a

NOTE.

Take the origin

common

at one of the vertices

point.

and make

the X-axis coincide with one of the sides. Find the equations of the sides and then find the equations of the perpendiculars ;

at the

middle points of the

sides. The point of intersection of any two of these perpendiculars ought to satisfy the equa tion of the third.

PLANE ANALYTIC GEOMETRY.

48 6.

Prove that the perpendiculars from the vertices of a meet in a point.

triangle to the sides opposite 7.

Prove that the line joining the middle points of two of is parallel to the third side and is equal

the sides of a triangle to one-half of it.

8. The co-ordinates of two of the opposite vertices of a square are (2, 1) and (4, 3) required the co-ordinates of the other two vertices and the equations of the sides. Ans. (4, 1), (2, 3) y 4. 2, x 1, y 3, x ;

=

;

9.

=

=

=

Prove that the diagonals of a parallelogram bisect each

other.

Prove that the diagonals of a rhombus bisect each other

10.

at right angles. 11.

Prove that the diagonals of a rectangle are equal.

12.

Prove that the diagonals of a square are equal and

bi

sect each other at right angles.

The distance between the points (x, y) and the algebraic expression of the fact. give 13.

Ans. (x

The points

14.

-

+

1)2

A

15.

3) are equi-distant

points

(x",

at an angle

L

y

2), (

What

16.

(

_

2

2)

= (x -

2

2)

+

(y

-

circle circumscribes the triangle

(3, 4), (1,

Ans.

+

2

I)

=4

- 2) = 4 2

(y

;

2 .

from the point

Express the fact algebraically.

(x, y).

(x

(1, 2), (2,

-

(1, 2) is

=

is

y"),

(3

1,

2)

;

a

3)

;

x

or,

whose

+ y = 4.

vertices are

required the co-ordinates of its centre. Ans. (2, 1).

the expression for the distance between the inclined (x ?/ ), the co-ordinate axes being ,

?

- xj + 2

V<V

(y"

- yj + 2

-x (x"

)

(y"

-y

}

cos

THE STRAIGHT LINE.

49

17. Given the perpendicular distance (p) of a straight line from the origin and the angle () which the perpendicular makes with the X-axis required the polar equation of the line. ;

Ans. cos (0 18.

origin on the line 19.

)

Required the length of the perpendicular from the

What

the point

is

|

+|=

2 4

AnSt

the equation of the line which passes through and makes an angle of 45 with the line

2),

(1,

1.

whose equation

is

y

-\-

2x

=

1 ?

Ans.

y

=

- x

+_

.

of two lines passes through the points (1, 2), the other passes through the point (1, 3), and makes an angle of 45 with the first line required the 20.

One

4,

(

3),

;

equations of the lines. Ans.

= 1. x + 1, and y = 3, or x y in the normal equation of a line, through 21. If p = what point does the line pass, and what does its equation Ans. (0, 0) y = s x. become ? ;

22.

r sin

Required the perpendicular distance of the point (r cos $, Ans. r from the line x cos y sin = p. p.

+

0),

Given the base of a triangle the squares of its sides == 4 c 2 the vertex is a straight line. 23.

of

.

= 2 a, Show

and the difference that the locus of

24. What are the equations of the lines which pass through the origin, and divide the line joining the points (0, 1), (1, 0), into three equal parts. 2 y x. Ans. 2 x ?/,

=

=

25.

If (x

,

y ) and

show that the point

(x",

(

y")

be the co-ordinates of two points,

+ nx m + "/^divides the m+n m n

mx

"

y"

^

ii

ne

-\-

joining

m

:

n.

them

into

two parts which bear to each the

ratio

PLANE ANALYTIC GEOMETRY.

50

CHAPTER

IV.

TRANSFORMATION OF CO-ORDINATES. IT frequently happens that the discussion of an equa and the deduction of the properties of the locus it represents are greatly simplified by changing the position of the axes to which the locus is the referred, thus 31.

tion,

simplifying

equation, or reducing

some desired form. The operation accomplished is termed the TRANSFORMATION it

to

by which this is OF CO-ORDINATES.

FIG.

The equation

of the line PC, Fig. 17,

y

when axes

= sx

and

X

its

y

is

-f b

referred to the axes

Y

17.

Y

and X.

If

we

refer

equation takes the simpler form

= sx

.

it

to the

TRANSFORMATION OF CO-ORDINATES. we

If

refer

it

to

Y"

and

X",

51

the equation assumes the yet

simpler form y"

appears that the position of the axes materially

it

Hence,

affects the

- 0.

form of the equation of a locus referred to them.

The equation

NOTE.

of a locus

which

referred to rec

is

of tangular co-ordinates is called the KECTANGULAR EQUATION the locus when referred to polar co-ordinates, the equation is called the POLAR EQUATION of the locus. ;

To find the equation of transformation from one system

32.

of co-ordinates to a parallel system, the origin being changed.

O

FIG.

18.

X

and Y as axes, be any plane locus referred to be any point on that locus. Draw PB to OY then from the figure, we have, of P when referred (OB, BP) (x, y) for the co-ordinates

CM

Let

and

let

P

;

||

=

to

X

and

Y

;

= (# ,/) for the

(O A, AP) to

X

Y DO

and

(OD, origin.

co-ordinates of

P when

referred

;

)

(a,

b)

for the

co-ordinates

of

,

the

new

PLANE ANALYTIC GEOMETRY.

52

From

OB = OD

the figure

x

hence

=

a

+

-f

DB

x and y

=b

BP = BA

and

;

-\-

+ AP

;

y

are the desired equations. As these equations express the relations between x, a, x y, on the locus they express the relations b, and y for any point f

,

Hence, since the

between the quantities for every point.

CM

Q expresses the relationship between equation of th locus the co-ordinates of every point on it if we substitute for

x and y in that equation their values in terms of x and y the resulting or transformed equation will express the rela tionship between the x and y co-ordinates for every point on it.

EXAMPLES. 1.

What

origin

is

=

does the equation y

removed

+1

3x

become when the

to (2, 3) ?

Ans. y

x

Construct the locus of the equation 2 y fer the origin to (1, 2) and re-construct. 2.

3.

The equation

of a curve

is

2 ?/

2 -f x

(2,

-

origin

2) ?

What

does the equation

become when the origin

is

2 ?/

- 2 x* -

removed

to (

,

9

The equation

of a circle

is

x*

+

2 ?/

4.

8

=

;

taken at

+ y = 16. + 6x - 3 =

2y 1

2

? )

Ans. 2 5.

is

-\-

Trans

x2

Ans. 4.

= 2. 4 x

4 y

-\-

what does the equation become when the

= 3x

2 ?/

=a

2

4 x2

when

=

-

1.

referred

does this rectangular axes through the centre. What left-hand the at equation become when the origin is taken

to

extremity of the horizontal diameter

?

TRANSFORMATION OF CO-ORDINATES. 33.

53

To find the equations of transformation from a rectangu-

lar system to

an

oblique system, the origin being changed.

B

L

D FJG.

19.

Let P be any point on the locus CM. Let O Y O X be the new axes, making the angles y and with the X-axis. Draw PA to the Y -axis also the lines ,

;

||

O D, AL, PB AF, O K to ||

From

the figure,

we

to the Y-axis, the X-axis. ||

and

have,

OB = OD PB = DO

+ O N + AF, + AN + PF.

and

OB = x, OD = a, O N == x cos 6, AF = PB = y, DO = b, AN = x sin 0, PF =

But

hence, substituting,

we

cos g>,

sin <p

;

have,

=a+x = b x y x

+ y cos + y sin

cos sin

-}-

)

,^

q>

<p

)

for the required equations.

COR.

1.

If a

= 0,

have,

x

y

and

b

= x! cos = x sin B

= 0, -f-

-f

y cos y sin

coincides with 0, and

<p

go

)

...

we

(2)

.

for the equations of transformation from a rectangular system to an oblique system, the ori-gin remaining the same.

PLANE ANALYTIC GEOMETRY.

54

= 90 + 6, coincides with 2. If a = 0, b = 0, and and the new axes X and Y are rectangular. Making these substitutions, and recollecting that COR.

we

g>

cos

(p

sin

(f

= cos (90 + 0) = sin = sin (90 + 0) = cos

0,

and

9,

have,

=x y = a

x

y sin 9 \

cos

+ ?/ cos

sin

,^

j

one rectangular system /or &e equations of transformation from the to another rectangular system, origin remaining the same. If we rind the values of x and y in equations (2) NOTE. in terms of x and y we obtain the equations of transforma tion from an oblique system to a rectangular system, the

origin remaining the same.

EXAMPLES.

=

16 become when the does the equation x* if ? 45 of an angle axes are turned through Ans. The equation is unchanged. 2.

tion

+

What

1.

1 required the equa The equation of a line is y = x axes to referred when making angles of line same the of ;

45 and 135

with the old axis of

x.

Ans.

3.

when

y

=

-\A-

does the equation of the line in Example 2 become an referred to the old Y-axis and a new X-axis, making

What

angle of 30

with the old X-axis. Ans.

2 y

= (V3 -

1)

x

-

2.

TRANSFORMATION OF CO-ORDINATES. 34.

To find the equations of transformation from a rectangu

lar system to a polar system, the origin

FIG.

Let angle

55

(a, b) g>

be the pole and

with the X-axis.

point on

it.

From

non-coincident.

20.

O

S the

initial line,

CM

Let

the figure,

and pole

we

making an

be any locus and have,

P

any

OB = OD BP = DO But

OB = x, BP = y,

+ O F, + FP. = OD a, O F = O P cos PO F = r cos (B + DO = FP = O P sin PO F = r sin (0 +

g>)

b,

<p)

hence, substituting,

we

=a = b y x

have,

-f-

-(-

;

r cos

qp))

+?

r sin (B

)

)

for the required equations.

COR.

1.

If the initial line

usual to so take

it)

x y

cp

= 0,

O

S

is

parallel to the X-axis

(it is

and

= a + r cos = b + r sin

^

) )

become the equations of transformation. COE. line

2.

If the pole is taken at the origin 0,

made coincident with

the X-axis a

and the

initial

= 0, b = 0, and = 0. g>

PLANE ANALYTIC GEOMETRY.

56

Hence, in this case,

s-rcosfl) (3)

_

= r sin 6

y

)

will be the required equations of transformation.

35.

system

1.

To find the equations of transformation from a polar to a rectangular system.

When

From

and

the pole

initial line coincides

equations

=

2

adding

r

by division

tan

origin are coincident,

and when the

with the X-axis. 2

cc

we

Art. 34,

(3),

+y

=

2 ;

have, by squaring and

and,

.

x

for the required equations.

We

have, also, from the same

equations,

Vx +

r

2.

When

2

the pole

y

and

Vx + y

r

2

2

2

origin are non-coincident,

and when

the initial line is parallel to the X-axis.

From

a simi equations (2) of the same article, we have, by

lar process,

= (x- ay + tan. 6 = y^~~-

r*

x cos

=x

sin0

=_y

,,

a

=

a

;

(y

-

also

a

x

-

2

b)

b

V(* for the required equations.

-

ay

+

(y

-

W

TRANSFORMATION OF CO-ORDINATES.

57

EXAMPLES.

The rectangular equation of the circle is x + y = a what is its polar equation when the origin and pole are coin 2

2

2

1.

cident and the initial line coincides with the X-axis ? Ans.

The equation

2.

of a curve

is (x

2

+y

2

2

)

=a

2

2

= a.

r

y

(x

;

2

)

re

;

and initial being taken as quired its polar equation, the pole in the previous example. a 2 cos 2 0. Ans. r 2

=

Deduce the rectangular equation of the following curves, assuming the origin at the pole and the initial line coincident with the X-axis.

= a tan sec ^ws. o^ = a? y. sec 4. r = a tan = a y. 2

r

3.

;c

3

r2

=a

6.

2

sin 2 2

Ans.

2

2

2

5.

(x

+

a (cos 2 Ans. x 2 y r ==

+

2

2

2 ?/

)

=2a

scy.

sin 0)

=a

y}

(x

GENERAL EXAMPLES. Construct each of the following the origin to the point indicated, the to the old, and reconstruct

straight lines, transfer

new axes being

parallel

:

1.

2. 3. 4.

= 3 x + 1 to (1, 2). to (- 1, 2). 2 y - x - 2 = _ i = to 4 (- 2, - 1). y y + x _f_i = o to (0, 2).

y

_|_

What do referred to

5.

6. 7.

8.

= sx + b to d). - 2). y + 2 = to (2, = mx to w). y -4o; + c = 0to(d,0).

y

(c,

a;

?/

(Z,

the equations of the following curves become when a parallel (rectangular) system of co-ordinates

passing through the indicated points ? 9.

10

"

11.

3 x2

?

+2y = 2

= 4 *(1

9 y2

6, (

,

0).

>>-

- 4x = 2

V2

12.

36

(3, 0).

/_ 2

*

PLANE ANALYTIC GEOMETRY.

58

=

What

13.

X-axis

is

Y-axis

is

does the equation x 2 -f- y 2 4 become when the turned to the left through an angle of 30 and the turned to the right through the same angle ?

14. What does the equation x axes are turned through an angle of 2

15.

What

2

,y

=a 45

become when the

?

=

the polar equation of the curve y 2

is

pole and origin being coincident, and the ing with the X-axis ? 16.

2

The polar equation

of a curve

r

is

2 px, the

initial line coincid

=a

(1

-f-

2 cos 0)

;

required rectangular equation, the origin and pole being coincident and the X-axis coinciding with the initial line. its

2

Ans.

-f

(x

y

2

2 ax) 2

=

a 2 (x 2

-f

y

2

).

Required the rectangular equation of the following curves, the pole, origin, initial line, and X-axis being related as in

Example 17.

r

2

16..

*

=

20.

r

=

asec 2 ?-. 2

cos 2

Ans. x 2 18.

19.

2

y

= a sin r = aO.

r

0.

=a

2 .

21.

22.

r r

2

= a sin 2 0. -2r (cos + V3 sin

Find the polar equations of the equations are 23. 24.

2

2

whose rectangular

:

= y (2 a - x). 4a x = y (2 a x). xs

loci

2

25. 26.

= a x - x\ x* -f y* = a*. 2

a*y

2

4

= 5. 6>)

THE CIRCLE.

CHAPTEE

59

V.

THE CIRCLE. 36.

THE

generated by a point moving in remain at the same distance from a will be observed that the circle as here de

circle is a curve

the same plane so fixed point.

fined

is

It

as to

the same as the circumference as defined in plane

geometry. 37.

Given the centre of a

circle

and

radius

its

to

deduce

its

equation. Y

Let C

(^ $)

be the centre of the

point on the curve.

OX

;

then

circle,

PM

to ||

and

OY

let

P

and

be any

CN

^^

= (*(, & MP) = (x, y)

(OA, AC)

(OM,

Draw CA and )

are the co-ordinates of the centre C.

are the co-ordinates of the point P.

||

to

PLANE ANALYTIC GEOMETRY.

60

=

Let CF

From

a.

CN ON NP

But

2

have,

+ NP = CP ... (1) = (OM - OA) = -^, = (MP - AC) = (y -^) and =a 2

2

;

2

2

(a;

2

2

2

,

CP 2

2

Substituting these values in 2

(x

we

the figure,

-1^)

+

we

(1),

have,

-$) = a 2

(y

.

.

.

for the required, equation. For equation (2) relation existing between the co-ordinates of

the circle

(2)

expresses the

any point (P) on between the co

hence it expresses the relation ordinates of every point. It is, therefore, the equation of the ;

circle.

If in (2)

we make x a*

+ ,f CC

or,

symmetrically,

= =a

2

.

2

- 4-

= 0,

and y .

.

=1

?/

have,

*

(3)

.

.

.

(4)

L

CL

we

*

for the equation of the circle when referred to rectangular axes passing through the centre.

Let the student discuss and construct equation

(3).

See

Art. 11.

COR. factor,

1.

we

If

we transpose x z

have, 2 ?/

i.e.,

=

(a

+ x)

in (3) to the second

x)

(a

in the circle the ordinate

is

member and

;

a mean proportional between

the segments into which it divides the diameter. COR. 2. If we take L, Fig. 21, as the origin of co-ordinates, and the diameter as the X-axis, we have,

LC

H = x = a and y = 0.

These values of x and y (x

-

2

a)

+

+

in (2) give 2 ?/

=a

2 ,

=

2 2 2 ax reduction, x y (5) for the equation of the circle when referred to rectangular

or, after

.

.

.

THE CIRCLE.

61

axes taken at the left hand extremity of the horizontal diameter. 38.

Every equation of the second degree between two varia which the coefficients of the second powers of the variables are equal and the term in xy is missing, is the equa bles,

in

tion

of a

circle.

The most general equation these conditions obtain

ax 2

of the second degree in

which

is

+ ai/ + cx+dy+f=Q.

.

.

.

(1)

Dividing through by a and re-arranging, we have, a If to both

members we now add c

2

4 a2 the equation

may

+

d*

4 a2

be put under the form

4 a2 this with (2) of the preceding article,

Comparing that

it is

we

see

the equation of a circle in which c ~a

d ~2~o

are the co-ordinates of the centre and

=

COR. 1. If ax 2 -{be the equation -f ex dy -f- m of another circle, it must be concentric with the circle repre sented by (1) for the co-ordinates of the centre are the same. ay"-

+

;

Hence, when the equations of

circles

have the variables in

PLANE ANALYTIC GEOMETRY.

62

their terms affected with equal coefficients, each to each, the

Thus

circles are concentric.

are the equations of concentric circles.

EXAMPLES.

What

is

of the

the equation

circle

when

the origin

is

taken.

What

1.

At D,

Fig. 21 ?

Ans.

x2

2.

At K,

Fig. 21 ?

Ans.

x2

3.

At H,

Fig. 21 ?

Ans.

x2

+ f - 2 ay = 0. + if + 2 ay = 0. + if + 2 ax = 0.

are the co-ordinates of the centres, and the values of

the radii of the following circles ? 4.

Ans. 5.

a; a-t-

2 ,*

7.

+2

2 z2

X2

2

,

?/

=

2, 3),

a

= 4.

a

= 2.

a

= 3.

4) a

= 5.

+ 4a;-6y-3=0. (-

^tns.

6

a

(1, 1),

2

= 0.

8x

?/

Ans.

(2,0),

_ 6 x = o. (3, 0),

8.

xz

+

2

?/

4 x -f 8

5

?/

= 0. .

9.

x 2 H-

10.

x2

11.

x2 2

12.

x

13.

x2

2 ?/

mz

+^ +G= 2/

+ = m. 4* = + =c +^ cz + =/ 2

?/

2

?/

2

2

2

.

2/

2

7/

wy.

(2,

THE CIRCLE. Write the equations of the

circles

63 whose

and whose

radii

centres are 14.

a

= 3,

18.

(0, 1).

x*+y -2y = 8. 15. a -2, (1, - 2). 19. Ans. x + if -2x + 4 y + 1 =0. 20. 16. a = 5, (- 2, - 2). + y -f 4 + 4 = 17. 17. a = 4, (0, 0). 21. = 16.

a

= m,

a

=

a

= 5,

(b, c).

2

Ans.

b, (c,

-

d).

2

^4rcs.

z

2

ic

The radius

of a circle -\-

y

5

is

;

what

is its

\

24.

(2, i).

equation

x 2 -f

2

common

centre the point

Find the equation of a

(2,

circle passing

4

?/

Write the equations of two concentric

have for their

A;,

if it is

4ic=2?

2

Ans.

v

=

?/

concentric with x 2

23.

&).

2

2

cc

22.

(7,

a;

?/

cc

circles

1).

= 21. which

,

through three

given points. 39.

To deduce the polar equation of the

circle.

B FIG.

The equation

of the circle (X

-XY+

22.

when

(y

referred to

_ y Y = a\

OY,

OX

is

PLANE ANALYTIC GEOMETRY,

64

To deduce the curve, then

polar equation let

P

be any point of the

(OA, AP) = (x, y) - (* y ) (OB, BO ) = (r, 0) (OP, POA) ,

= (/, = r cos

(00 O OB) ,

From

A=

the figure,

OB = hence, substituting, (

a;

we

=r

cos

,

)

AP = y = r sin BO = y = r sin

2

)

+

(r sin

- / sin

Squaring and collecting, we have,

+ sin 0) + r (cos + sin =a sin r + r - 2 r/ cos (0 -

+ sin

2

ff

)

2

)

=

a

2 .

- 2 r/ (cos

cos

)

2

&e polar equation of the

is

:

2

2

i.e.,

2

2

2

2

^(cos

0,

0,

have,

r cos

r C os 6

x

)

=

a*

.

.

.

(1)

circle.

This equation might have been obtained directly from the triangle*

COR. tre

OO P.

1.

If

= 0, the initial line OX passes through the cen

and the equation becomes r2

COR.

2.

Tf

+/

= 0,

a

2 rr cos

and /

= a,

=a

2 .

the pole lies on the circum

ference and the equation becomes

= 2 a cos = 0, and / = 0,

r

COR.

3.

If

0.

the pole

is

at the centre

and

the equation becomes r

40.

= a.

To show that the supplemental chords of the

each other. perpendicular The supplemental chords of a

circle are

to

circle are those chords

diameter and pass through the extremities of any other on the circumference.

which

intersect each

65

FIG.

23.

Let PB, PA be a pair of supplemental chords. prove that they are at right angles to each other. The equation of a line through B ( a, o) is

For a

line

A

through y = s

Multiplying these, if

=

(a, o),

We

wish to

we have

a).

(x

member by member, we have a 2) ... (a) ss (x*

for an equation which expresses the relation between the co-ordinates of the point of intersection of the lines.

Since the lines must not only intersect, but intersect on the circle

whose equation

this equation

above

;

is

must subsist

hence, dividing,

same time with equation

at the

(a)

we have

"1

or,

1

+ ss =

...

(1)

Hence the supplemental chords

of a circle are perpendicular

to each other.

Let the student discuss the proposition for a pair of chords passing through the extremities of the vertical diameter.

PLANE ANALYTIC GEOMETRY.

66

To deduce the equation of the tangent

41.

FIG.

to the circle.

24.

Let CS be any line cutting the Its equation is

circle in the points

P

r

(#

,

y

),

P"

(x",

y").

r

x

(x

Since the points (a/, y ), the equations of condition /**

(x",

I

/"I

~~

J

*

*

(Art. 26, (4)

are on the circle,

y")

2

/2

i

).

*

).

we have

\

\

/

These three equations must subsist at the same time hence, subtracting (2) from (1) and factoring, we have, ;

f

(x

+ y x

x")

y

-

(*

x")

+

y,

_|_

X

y

+

y"

^

.x"

+

(y

l

y")

(y

Substituting in the equation of the secant line

If

point

we now revolve the secant

P

when the

will approach

secant

P

;/

CS becomes

line

-

y")

-

;

"

it

becomes

upward about

P"

the

will finally coincide with it tangent to the curve. But when

and

THE CIRCLE. P

coincides with

in (3)

we

P",

x"

and

?/

=

?/";

hence, substituting

have,

-

y or, after

=

x

67

y"

=-

^

(*

-

(*)

*")>

reduction,

+ yy _;..;.. "

a-x"

or,

(5)

symmetrically,

for the equation of the tangent.

SCHOL. The SUB-TANGENT for a given point of a curve is the distance from the foot of the ordinate of the point of tangency to the point in in Fig. 24,

which the tangent intersects the X-axis

AT

its

value

we

have,

P".

make y

OT = But

thus,

;

the sub-tangent for the point To find in the equation of the tangent (5) and

is

x

=

.

AT = OT - OA =

-

x"

x"

= an ~ x = V,//2 -2

sub-tangent

"2

.

To deduce the equation of the normal to the The normal to a curve at a given point is a dicular to the tangent drawn at that point. 42.

The equation 24, is

of

any

-

y

y"

through the point

line

= s (x -

x")

...

circle.

line

perpen

P"

(x",

y")

Fig.

(1)

In order that this line shall be perpendicular to the tangent P"T, we must have 1

But

Art. 41, (4)

-f s

88

=

= 0.

~ y"

;

hence,

we must have

s

=V x"

.

PLANE ANALYTIC GEOMETRY.

68

Therefore, substituting in

y

y"

we

(1),

-*(*-

have, (2)

*")

;

*

or, after

reduction,

yX

-X

"

y"

=

...

(3)

for the equation of the normal. see from the form of this equation that the the circle passes through the centre.

We

normal to

SCHOL. The SUB-NORMAL for a given point on a curve is the distance from the foot of the ordinate of the point to the In the circle, point in which the normal intersects the X-axis.

we

see

from Fig. 24 that the Sub-normal =

By methods

43.

the last two articles, (x

to be

-x

x" .

precisely analogous to those developed in we may prove the equation of the tangent

- x )* + (y- y Y = *

- x ) + (y - y ) ) and that of the normal to be - x ) -(x(y -y") (x

r

(x"

(y"

- y } = a*

-y Let the student deduce these equations. x")

(x"

(y"

)

=

.

.

.

...

(1)

(2)

EXAMPLES. 1.

dy

What is

+f=

0,

+

+

+

the polar equation of the circle ax 2 ex ay* the origin being taken as the pole and the X-axis

as the initial line ?

Ans.

r2

+ (- cos \a

2.

x2

What

is

+ f = 25

equation of (3,

- sin a

\r J

What

+ /a = 0.

the

tangent to the circle of the subAns. 3 x 4y 25 J^.

The value

4) ?

+

tangent ? 3.

the

at the point

-f

=

;

the equation of the normal to the circle 37 at the point (1, 6) ? What is the value of the

x 2 -f if = sub-normal

is

? --

Ans. y

= 6x

;

1.

THE CIRCLE. What

4.

x2

circle

are the equations of the tangent and normal to the 20 at the point whose abscissa is 2 and ordi-

if

-f-

69

=

Give also the values of the sub-tangent and sub-normal for this point. nate negative ?

2

Ans.

cc

- 4 = 20 =8 ?/

Sub-tangent

2

;

;

?/

+4 = = 2. cc

;

sub-normal

Give the equations of the tangents and normals, and the values of the sub-tangents and sub-normals, to the following circles 5. 6.

7.

:

+ ?f = 12, at (2, + V8). = 25, at (3, - 4)! x + = 20, at (2, ordinate +). x + x -f if = 32, at (abscissa +, 4). x + y = a at y = m, at (1, ordinate +). = &, at (2, ordinate x + = 18, at (m. ordinate -f). = 45 and the Given the circle x + x* z

2

?/

2

2

?/

2

8.

9.

2

2

2

2

2

,

10.

a?

-|-

2

11.

(6, c).

2

-j- ?/

12.

cc

2

).

2

7/

2

13.

+ =

2

line 2 y x 2 required the equations of the tangents to the circle which are parallel to the line. ?/

Ans

What

14.

x~

+

2 2/

f3*

{3*

;

+ 6y = 45. + 6y=-45.

are the equations of the tangents to the circle

= 45 which are perpendicular to the line ,

2y

-f-

^

=2?

(3y- 6x^45.

- 3 y = 45. x + y = 9

16 x

The point

1 2 (3, 6) lies outside of the circle the of the required equations tangents to the circle which pass through this point. X 3 Ans. A 4 3x 15. y

16.

;

=

=

PLANE ANALYTIC GEOMETRY.

70

What

17.

_

(x

2

2)

equation of the tangent to the circle the point (4, 4) ? Ans. 2 x -f y 12.

the

= 5 at

2

3)

(y

=

The equation

18.

the

+

is

x2

circle

-\-

y*

of one of

=9

is

y

=

two supplementary chords of x -|- 2, what is the equation

of the other ?

2

?/

-f 3

.x

== 9.

Find the equations of the lines which touch the circle 2 2 = r* and which are parallel to y = sx -f- c. a) -\- (y b)

19.

(x

x 2 + ?/ 2 4 x + 4 ?/ = 9 of the normal at the the point whose required equation abscissa == 3, and whose ordinate is positive. Ans. 4 x y = 10.

The equation

20.

of a circle

is

;

44. To find the length of that portion of the tangent lying between any point on it and the point of tangency.

Let (xu this point

be the point on the tangent. The distance of 1/1) from the centre of the circle whose equation is

x

(x V(a>!

Y

-f-

y

(y

-xY+

Y

<*?

- y y.

(y,

is

evidently

See Art. 27,

(1).

But

this distance is the hypothenuse of a right angled tri whose sides are the radius a and the required distance angle

d along the tangent d*

COR.

1.

If

x

= d2

as

it

;

=

hence (x,

-xy+

and y

= 0,

(2/1

then

= x^ + ^ - a

- yY - ^ (1)

...

(i)

becomes

2 .

.

.

(2)

ought.

45.

To deduce the equation of the radical axis of two given

circles.

The RADICAL AXIS OF TWO CIRCLES is the locus of a point from which tangents drawn to the two circles are equal.

THE CIRCLE.

71

T

FIG.

Let

x

(x \

Y

+ (v + (y

/

x")

y/

\*/

I

2

(x

25.

2

} /

y")

=a =b

2 ,

2

be the given

Let P (XD yi) be any point on the radical axis the preceding article, we have,

d2 = d2 = .:

+

6/1

2

b

y")

(x"

,

x

)

(y"

Calling, for brevity,

w iH

2

(

)

2

(

y")

2

)

1

y)

-f (y t

hence, reducing,

- x x,+2

-f-

(#15 2/i)

+ Vl - y + yi 2

x")

( Xl 2

2

-

(x,

by definition 2

-x )

(x,

=

y ) y, 2 a2 b

we

then from

;

a2 b2

=

a2

2

( Xl

x")

have, 2

x"

2

circles.

-

x

2

+

-y

2 y"

2

.

the second

member m, we

see that

satisfy the equation. 2

(x"

-x )x+2

(y"

y

)

y

=m

.

.

.

(1)

But (xu 1/1) is any point on the radical axis hence every It is, therefore, the re point on that axis will satisfy (1). ;

quired equation.

COR. then, is

1.

If c

=

and

c

c

=

c

=

be the equation of two circles,

the equation of their radical axis.

PLANE ANALYTIC GEOMETRY.

72

2. From the method of deducing (1) it is easily seen the two circles intersect, the co-ordinates of their points of intersection must satisfy (1) hence the radical axis of two

COR.

that

if

;

intersecting circles is the line joining their points of intersection,

PA,

Fig. 25.

Let the student prove that the radical axis of any two circles is perpendicular to the line joining their centres.

To show that the radical axes of three given a common point.

46.

circles in

tersect in

Let

c

= 0,

c

= 0,

and

c"

=

be the equations of the three circles. Taking the circles two and two we have for the equations of their radical axes c c

-c = - = _c =o

.

c"

.

.

(1)

...

(2)

"

c

.

.

.

(3)

evident that the values of x and y which simultaneously will also satisfy (3) hence the proposition. and

It is

satisfy (1)

(2)

;

The intersection of the radical axes of three given circles

THE RADICAL CENTRE

called

is

of the circles.

EXAMPLES. Find the lengths of the tangents drawn to the following circles 1.

:

(x

-

2

2)

+

(y

-

2

3)

=

16 from

(7, 2).

Ans. 2.

x>

+

(y

+ 2)

2

-

10 from

4. 5.

_ a y + if = 12 from x 2_f2/ 2_2^+4?/-2 from x + y = 25 from (6, 3).

VlO.

d=^/3.

(b, c).

(a-

2

=

(3, 0).

Ans.

3

d

(3, 1).

2

Ans.

d

=

V20.

THE CIRCLE. 6.

7>

g.

+y 2

x2

_

(3

X

2

a )2

+y

i

=

2 x

10 from

73

(5, 2).

= 3.

Ans. d

+

_

_

(^

6)2

=

4y

=

rom

c f

10 rom f

(d,

/).

(0, 0).

Give the equations of the radical axis of each of the follow ing pairs of circles

:

9.

10.

13.

+_ y 3)2 | (x (

1

x2

2 |X

_^_

* |x

_|_ (

(

x

2

^2

4y ^2

_|_

_ _

y

2 y

3)2

=

0.

_

16

+- y = 16. = I) +

^s.

o.

-4?w.

3

= 2 y.

a?

a?

=

i-

= 0.

z

2

2

1 (x

= 0. _9 _

2/

a?.

14.

Find the co-ordinates of the radical centres of each of the following systems of circles 15.

:

= 9. - 2 = 25. 2

16.

+ if - 4 x + 6 y - 3 = -4* = 12. = 6 | x = (.T + = 9. J I) + f (^ __ 2 4. 4 y = 10. ( ^2 + yj x + if - k* = -j= d. // (

J

x-

_j_

?/

_|_

y>

7.

2

2

.

7/

2

a,

18.

(

G,

^rcs.

(1,

-

3).

0.

x8 4-3^ .2

17.

-4w.

2

ft

2

2

?/

?H

.

2

-f-

//

I)-

PLANE ANALYTIC GEOMETRY.

74 47.

To find the condition that a straight

= sx + b + =

line

y

2 a2 must fulfil in order that it may touch the circle x y~ In order that the line may touch the circle the perpendicu lar let fall from the centre on the line must be equal to the .

radius of the circle.

From

p

= bcosy = p

r2 (1

hence, is

we have

Art. 21, Fig. 13,

-

= VI

+ tan.

2

=r=

+s =b 2

2

)

.

.

.

(1)

the required condition. COR. 1. If we substitute the value of b

the equation y

-

-==

sec y

= sx + y = sx

b,

AVC

_j_

r

drawn from

(1) in

have

Vl +s

2

...

(2)

for the equation of the tangent in terms of its slope.

48. Two tangents are drawn from a point without the circle ; the points oftangency. required the equation of the chord joining

FIG.

26.

y ) be the given point, and tangents through it to the circle. Let

P

r

f

(x

,

let

PT",

P P,

be the

THE CIRCLE.

75

deduce the equation of The equation of a tangent through It is required to

PP

/X .

is

P"

y")

(#",

\yy^_

xx"

O

P

Since

on this

is (a/, ?/)

the equation

i

line, its co-ordinates

must

satisfy

hence

;

x

The point

_

rt

I

(x",

1/1/ = 1 + ^~ a

x"

*

.

2 aT

2

?/"),

therefore, satisfies the equation

A

it is a point on the locus represented by similar (1). source of reasoning will show that P is also a point of this locus. But (1) is the equation of a straight line hence, since .-.

;

it

is satisfied

for the co-ordinates of both

P"

equation of the straight line joining them. the required equation. 49.

A

chord of a given

and P, It

circle is revolved

is,

it is

the

therefore,

about one of

its

points ; required the equation of the locus generated by the point of intersection of a pair of tangents drawn to the circle at the points in which the chord cuts the circle.

Let

P

f

f

Fig. 27, be the point about which the chord It is required to find the equation of the locus generated by Pj (x l5 yj, the intersection of the tangents revolves about P 1? 1? as the line P

P AB

(x

,

y

),

revolves.

AP BP From

AB

.

the preceding article the equation of the chord

X^X

,

?/!?/ =

-,

^"""^

Since

P

(a/,

y)

on this

is

a2

xx hence Cb

^

we have

line,

a"

.

-\

y y == l 1 % Cf

.

.

/1

,

(1J

AB

is

PLANE ANALYTIC GEOMETRY.

76 is satisfied

P

for the co-ordinates of

the locus represented by

(1).

any pair of tangents drawn

x

But

(x lt

P

x

y-^)

is

;

hence

P

x

lies

on

the intersection of

to the circle at the points in

hence (1) position, cuts the circle will be satisfied for the co-ordinates of the points of intersec

which the chord, in any

;

tion of every pair of tangents so drawn.

the equation of the required (1) is, therefore, observe that equation (1) is identical with (1) of the preceding article hence the chord PP" is the locus whose

Equation

locus.

We

;

equation

we

sought.

The point P

(**

+& =

THE POLAR 1-1-21 (i

=

1

f

(x

Y

,

y)

is

and the

of the point

called line

P

THE POLE

PP"

(x>,

y>)

of the line

(^ + ^f

=

1\

is

PP"

called

with regard to the circle

THE CIRCLE.

77

the principles here developed are perfectly general, the circle. within or be without^ on, pole may Let the student prove that the line joining the pole and the centre is perpendicular to the polar.

As the

The terms pole and polar used no connection with the same terms used NOTE.

I.

co-ordinates, Chapter

If the polar of the point P (x y ), .Fig. 27, passes through XD yi), then the polar of P (x 1? yi) will pass through P 1

50.

PI

(

The equation

of the polar to

xx ~a*

In order that

P

x

+

yu

But f

(x

,

(x

,

y

) is

=

y^ may be on

(aj 1?

this line,

we must

have,

a2

this is also the equation of condition that the point f y ) may lie on the line whose equation is

^ _i_M

1

"

a2

a2

But

P

^~

a2

P

in this article have in treating of polar

this is the equation of the polar of P! (x^ y-^

;

hence

the proposition.

To ascertain the relationship between the conjugate diam

51. eters

of the

A pair

circle.

to be conjugate when they are the curve is referred to them as axes its equation will contain only the second powers of the variables.

of diameters are said

so related that

when x2

Let

+

2 ?/

ascertain

OY OX ,

,

a2

.

.

.

(1)

its centre and axes. becomes when referred to axes making any angle with each other, we must

be the equation of the

To

=

what

circle,

referred to

this equation

substitute in the rectangular equation the values of the old

PLANE ANALYTIC GEOMETRY.

78

co-ordinates in terms of the new.

have

=x =x

x y

+y +y

cos 9 r

sin

From

the

1

y

*

+ 2 x y cos

-

((f

FIG.

1,

we

cos 9 sin cp

equations of transformation. values in (1) and reducing, we have,

for

Art. 33, Cor.

Substituting these

+x =a 2

0)

2 .

.

.

(2)

28.

diameters Now, in order that OY OX may be conjugate the term containing x y in (2) that related so be must they must disappear; hence the equation of condition, ,

- 6) = = 90, 0*9-e 9

cos ...

(cp

;

The conjugate diameters of the As there dicular to each other.

circle are therefore

are an infinite

perpen

number

of

the circle which satisfy the condition of being pairs of lines in the circle there at right angles to each other, it follows that in diameters. are an infinite number of conjugate

THE CIRCLE.

79

EXAMPLES. 1.

-|-

Prove that the line y = V3 x -+- 10 touches the circle 2 = 25, and find the co-ordinates of the point of tangency. ?/

_ y f5 __

2.

y

What must be the value of b in 2 may touch the circle x +

=2x +b

order that the line

=

2

?/

16

?

^7i. 3.

y

What must be the value of 4 may touch the circle x

= sx

s

=

6

4. _|-

The y

2

=

V80.

-L,

in order that the line

=2?

2

-f if

Ans.

#2

5 3,

s

=

-L_

V7.

slope of a pair of parallel tangents to the circle 16 is 2 required their equations. ;

Ans.

^j

= 2x -

V80.

Two tangents are drawn from a point to a circle required the equation of the chord joining the points of tangency in each of the following cases ;

:

5.

From

6.

From

7.

From

8.

From

(4, 2)

(3,

to x

4) to or

(1, 5) to

x

+ if = 9.

2

2

2

+ if = +

4x

+2y =

^4ws.

3x

+ 4 y = 8.

8.

2

=

2

=c

?/

Ans.

16.

Ans. x (a, 6) to

2

cc

+

7/

9.

10.

Of

= 16.

2 .

v4?is.

What

5 y

-\-

9.

ax

by

-{-

=c

2

are the equations of the polars of the following points (2, 5)

Of

(3,

with regard to the

4)

circle

with regard to the

x

circle

2

.

:

+ y- = 16 ?

^

2

cc

.

5 y

-,

16+16 x + if = 9 ? ^715. 3 x + 4 y = 9. 2

PLANE ANALYTIC GEOMETRY.

80 11.

Of

(a, b)

with regard to the

circle

x2

-\-

What 12.

are the poles of the following lines

Of 2 x

-f 3 y

= m?

y-

Ans.

ax

:

= 5 with regard to the circle x

2

-f

y

Ans. 13.

Of ~

+ y = 4 with regard to the Jg

14.

Of y

2 -

= 25 ?

(10, 15).

circle

^

+ fg = 1 ?

= sx + 6

= m.

by

-\-

5

-

(

2 4 )>

with regard to the circle

Ans

-

(-^r>i

Find the equation of a straight line passing through x 2 -\- if 3 x -\- 4 y = 0. 0) and touching the circle Ans. y = - x.

15. (0,

GENERAL EXAMPLES. 1.

Find the equation of that diameter of a drawn parallel to y = sx + b.

circle

which

bisects all chords

Ans. 2.

line

sy

+#=

0.

Required the co-ordinates of the points in which the x-|-l = intersects the circle

2y

3. Find the co-ordinates of the points drawn through (3, 4) touch the circle

in

which two

lines

and the [The points are common to the chord of contact circle.]

THE CIRCLE. 4. (4,

The centre

0)

;

required

which touches the Y-axis

of a circle its

81

equation. 2

Ans.

(x

4)

is

at

+ y = 16. 2

Find the equation of the circle whose centre is at the = x -|- 3 is tangent. origin and to which the line y Ans. 2 x2 + 2 y* = 9. 5.

+y =

=

+

2

6.

Given x 2

2 16 and (x 4 required the if 5) equation of the circle which has their common chord for a diameter.

7.

Required the equation of the circle which has the dis (3, 4) from the origin as its diameter. Ans. x* -j- ?/ 2 4 y = 0. 3x

5

tance of the point

8. Find the equation of the circle which touches the lines x. 0, and y 3, y represented by x

=

9.

=

=

Find the equation of the

points

(-

(1, 2),

(-

2, 3),

circle

- 1).

1,

which passes through the

10. Required the equation of the circle which circumscribes the triangle whose sides are represented by y 4 x, 0, 3 y 4x and 3 y 6. Ans. x 2 -f ?/ 2 0. | x 1| y

=-

+

=

Required the equation of the circle whose intercepts are a and b, and which passes through the origin. 11.

12. is

The

points

in the line

13.

circle 14.

y

The point x2

ax

4

;

(4,

on a

whose centre

its

required Ans. 2 x 2

2) is the

(3.

6) lie

circle

by

equation.

+2

2 ?/

17 x

y

= 30.

middie point of a chord of the

= 16 required the equation of the chord. = 16 and the chord y 4 x = 8. x +

-\- if-

Given

and

(1, 5)

=x

= 0.

x 2 -f y z

Ans.

;

2

2

?/

Show

that a perpendicular from the centre of the circle bisects the chord. 15.

Find the locus of the centres of

pass through

(2, 4), (3,

-

2).

all

the circles which

PLANE ANALYTIC GEOMETRY,

82 16.

Show

that

if

the polars of two points meet in a third is the pole of the line joining the first

point, then that point

two points. 17.

= 8,

Required the equation of the and whose sub- normal = 2.

circle

whose sub-tangent

= 20.

x 1 -f y 2

Ans.

Required the equation of ihe circle whose sub-normal = 2, the distance of the point in which the tangent intersects the X-axis from the origin beinr = 8. 18.

x2

Ans. 19.

ax 2

Required the conditions

in

order

that

2

-f- ?/

the

= 16.

circles

+ aif + cx + dy + e = Q and ax + af + kx + ly + m =

may

z

be concentric. Ans.

20.

c

d

k,

I.

Required the polar co-ordinates of the centre and the

radius of the circle r2

2 r (cos

+

^/3 sin

0)

= 5.

Ans. 21.

A

line of fixed length so

moves that

60)

(2,

its

;

r

= 3.

extremities

remain in the co-ordinate axes required the equation of the circle generated by its middle point. ;

22.

Find the locus of the vertex of a triangle having given = 2 a and the sum of the squares of its sides = 2 b 2

the base

.

Ans.

x2

-j-

y

2

=

&

2

a2

.

Find the locus of the vertex of a triangle having given the base = 2 a and the ratio of its sides 23.

=

.

Ans.

A

circle.

n 24. Find the locus of the middle points of chords drawn from the extremity of any diameter of the circle

THE PARABOLA.

CHAPTER

83

VI.

THE PARABOLA. THE

parabola is the locus generated by a point moving same plane so as to remain always equidistant from a fixed point and a fixed line. The fixed point is called the Focus the fixed line is called the DIRECTRIX the line drawn through the focus perpendic ular to the directrix is called the Axis the point on the axis midway between the focus and directrix is called the VERTEX 52.

in the

;

;

;

of the parabola. 53.

To find the equation of the parabola, given the focus and

directrix.

Let

EC

be the directrix and let

F

the axis of the curve, and the tangent

be the focus.

OY

drawn

Let OX,

at the vertex

PLANE ANALYTIC GEOMETRY.

84

0, be the co-ordinate axes. to OY, PB

and draw

PA

||

(OA, AP)

From

Take any point P on the curve OX and join P and F. Then

to

||

=

y) are the co-ordinates of P.

(x,

FAP, we have 2 ... (1) generating the curve, we have

the right angled triangle

= AP = FP - FA 2

if

But from the mode of

2

FP = BP = (AO 2

and from the

+ OD) =

2

FA = (AO - OF) = 2

2

Substituting these values in

=

or, after

(

X

2

y

reduction,

As equation

true for every point

;

then

2

= 2 px

hence

(x

-

(x

-

OF)

OD = OF =

true for

is

(3)

(x

2 ,

.

OF)

2 .

we have

(1),

+ OD) -

DF =p,

Let

+ OD)

2

we have

figure,

if

;

.

.

2 .

.

.

(2)

hence

(3)

of the parabola it the equation of the curve.

any point

it is

is

COB. 1. If (x y ) and (x are the co-ordinates of any two points on the parabola, we have, 1

,

hence

,

2 ?/

2 :

y"

\:x

\

y")

x"

;

i.e., the squares of the ordinates of any two points on the bola are to each other as their abscissas.

para

SCHOL. By interchanging x and y, or changing the sign of the second member, or both in (3), we have 2 y 2px for the equation of a parabola symmetrical

=~

with respect to X and extending to the left of Y; x 2 = 2 py for the equation of a parabola symmetrical with respect to Y and extending above X. x2 = 2 py for the equation of a parabola symmetrical with respect to Y and extending below X. Let the student discuss each of those equations. See Art. 13.

THE PARABOLA. 54.

85

To construct the parabola, given the focus and

FIG.

First Method.

Let

DR

directrix.

30.

be the directrix and

let

F

be the

focus.

From F

FD on the directrix it Take a triangular ruler ADC and make its base and altitude coincide with the axis and Attach one end of a string, whose directrix, respectively. the perpendicular

let fall

;

will be the axis of the curve.

A

is AD, to the other end to a pin fixed at F. Place the point of a pencil in the loop formed by the string and stretch it, keeping the point of the pencil pressed against the

length

;

base of the triangle. Now, sliding the triangle up a straight edge placed along the directrix, the point of the pencil will describe the arc OP of the parabola for in every position of the pencil point the condition of its being equally distant from the focus and directrix is satisfied. It is easily seen, for ;

instance, that

FP = PD

when

the triangle

is

in the position

ADC

that

.

Second Method.

Take any point C on the

axis

and erect

PLANE ANALYTIC GEOMETRY.

86

the perpendicular P CP. as a centre and DC (=

With F

Measure the distance DC.

as a radius describe the arc of a

FP)

P CP in P and P P and F will be points of the parabola. By taking other points along the axis we may, by this method, locate as many points of the curve as may be cutting

circle,

.

desired.

55. To find the Lotus-rectum, or parameter of the parabola. The L AT US-RECTUM, or PARAMETER of the parabola, is the

double ordinate passing through the focus. The abscissa of the points in which the latus-rectum pierces

the parabola

is

=

x

.

2i

Making

this substitution in the equation

= 2px * = 2^y* = 2 p 2 y = 2 p. 2

?/

we have Hence COR.

1.

Forming a proportion from the equation if

i.e.,

any

= 2px, (

we have

x:y:\y. 2p;

the latus-rectum of the parabola is a third proportional to abscissa and its corresponding ordinate.

EXAMPLES. Find the latus-rectum and write the equation of the parab ola which contains the point 1.

5.

8,

?/-

=

8

Ans. 4.

What is it

is

?/

=

8

x.

(_,

,

y

2

= b- x. a

2).

-,?/=a parabola x* =

Ans.

the latus-rectum of the

defined in this case ?

2

b*

a

2

8,

(a, I).

x.

(-2,4). Ans.

How

3.

(2,4).

Ans. S.

:

-x. a

THE PARABOLA.

87

6. What is the equation of the line which passes through the vertex and the positive extremity of the latus-rectum of any parabola whose equation is of the form if 2px ?

=

Ans. y 7.

The focus

of a parabola

vertex of the curve

;

what

is

is its

when symmetrical with

(a)

"

"

at 2 units

= 2 x.

distance from the

equation respect to the X-axis ?

"

"

"

(b)

Ans.

y

(a)

2

"

Y-axis?

= 8x,

(b)

x*

=

8

y.

Construct each of the following parabolas by three differ ent methods. 8. 9.

12. 2 ?/

10.

x2

11.

x2

= 6 y. = - 10 y.

What

are the co-ordinates of the points on the parabola where the ordinate and abscissa are equal ? Ans. (0, 0), and (6, 6).

=6x 13.

x2

= Sx. - 4 x. if = if

Required the co-ordinates of the point on the parabola whose ordinate and abscissa bear to each other the

=4y

ration 3

:

Ans.

2.

What

(6, 9).

the equation of the parabola when referred to 2 the directrix and X-axis as axes ? Ans. y 2 2px p 14.

is

.

Find the points of intersection of the following 15.

if

16.

x2

= 4 x and 2 y

x

Ans.

17.

18.

:

= 0. (0, 0), (16, 8).

= 6 y and y x 1 = 0. 8 x and y* = + 3 = 0. = = 8. 2x and x + if a:

2

2

?/

Ans. 19.

x*

20.

x2

(2, 2), (2,

= - 4 y and 3 x + 2 if = 6. = 4 y and if = 4 x. 2

- 2).

PLANE ANALYTIC GEOMETRY.

88

To deduce the polar equation of

56.

the parabola, the focus

being taken as the pole.

The equation To

.

.

.

we have

FX

= + r cos = r sin

0.

0.

Substituting these values in

(1),

we have

= p~ + 2pr cos sin = 1 cos 9 = p -\-2pr cos 9 + r cos 9 = (p + r cos 0) r cos r =p 2

2

9.

?-

But

sin

2

r2

.-.

2

;

2

2

.-.

2

2 ,

9,

-\-

solving, 1

cos B

the required equation.

We Let

might have deduced this value directly as follows P (r, 9) Fig. 29 be any point on the curve then ;

= D A = DF -f FA = p + r cos 9 r=p + rvosO. r = cos 9 1 9 = 0, r = oo e = 90, r = p. 9 = 180, r = | 9 = 270, v = p. 9 = 360, r =

FP i.e.,

Hence COR.

*-

.

If If If

If If

An

;

.

.

-,

1.

is

and the pole F

1,

y

is

Fig. 29,

for the equation of transformation, Art. 34,

]

x

or,

OY, OX,

(1)

refer the curve to the initial line

,

Cor.

of the parabola referred to

f = 2px

.

.

oo.

these results. inspection of the figure will verify

:

THE PARABOLA.

89

To deduce the equation of the tangent to the parabola. be the points in which a secant line cuts (V, y ),

57. If

y")

(x",

the parabola, then

y-y =

y

,~

y

will be its equation.

parabola,

(-**)

",

(!)

Ou

OC-

Since (V,

?/),

(#",

y")

are points of the

we have y

2 /2

7/

= 2px = 2X

...

(2)

/

(

3)

These three equations must subsist at the same hence, subtracting (3) from (2) and factoring, we have y x

i.e.,

y"

time

2p

== f

-x"

y

+y"

Substituting this value in (1), the equation of the secant

becomes

When

the secant, revolved about y ) coincides with

(x",

to the parabola (x

=

y

or,

y".

Making

,

y"},

this substitution in (4),

simplifying, recollecting that yy"

=p(x+x")

2 y"

=2

...

becomes tangent hence x r

(x",

;

y")

we

px".

x",

have,

we have

(6)

for the equation of the tangent to the parabola.

To deduce the value of the sub-tangent. Making y = in (6), Art. 57, we have = OT, (Fig. 31) x = -

58.

x"

which the tangent intersects But the sub-tangent CT is the distance of this

for the abscissa of the point in

the X-axis.

point from the foot of the ordinate of the point of tangency; i.e., twice the distance just found; hence

Sub-tangent

=2

x" ;

PLANE ANALYTIC GEOMETRY.

90 i.e.,

the sub-tangent

is

equal

to

double the abscissa of the point

of tangency. 59.

The preceding

principle affords us a simple method of constructing a tangent to a parabola at a given point. Let be any point of the curve. Draw the ordinate P"C, and measure 00. Lay off OT = 00. P"

(a;",

y")

FIG.

A at

line joining

T and

P"

31.

will be tangent to the parabola

P".

60.

To deduce the equation of the normal

The equation

of any line through

= s(x-

y-y"

We have found Art. 57, s

r

-

p

to the

P"

(#",

.

.

x")

.

y"}

Fig. 31,

(1)

-

hence, for the slope of the normal

,P

is

(5) for the slope of the tangent

~7"

.__*:.

parabola.

P"N,

we have

P"T

THE PARABOLA. Substituting this value of

y

-y"

for the equation of the

=-

we have

s in (1),

-

^ (*

normal

91

(2)

*")

to the

parabola.

To deduce the value of the sub-normal.

61.

Making y

=

in (2) Art. 60,

we

have, after reduction,

x=*p+af = ON; Fig. 31, Sub-normal = NC = p .-.

Hence to the

-\-

the sub-normal in the parabola

is

x"

x"

constant

= p.

and equal

semi-parameter FB.

62.

To

shoiv that the tangents

drawn

at the extremities of

the latus rectum are perpendicular to each other. The co-ordinates of the extremities of the latus-recturn are ? ,

p

j

for the upper point,

and

I

,

p

for the lower point. j

Substituting these values successively in the general equa tion of the tangent line, Art. 57 (6), we have

yp

= plx

-yp=plx + or, cancelling,

y

=

x+|

.

.

.

(1)

y=-*-f

(2)

for the equations of the tangents. As the coefficient of x in (2) is minus the reciprocal of the coefficient of x in (1), the lines are perpendicular to each other.

COR. that x

1.

=

Making y = ?.

2

;

in (1)

and

(2),

we

find in each case

hence, the tangents at the extremities of the

PLANE ANALYTIC GEOMETRY.

92

and the same point.

latus-rectum in the

The values

directrix meet the axis of the parabola

of the coefficients of

these tangent lines

make

x in

angles of 45

(1) and (2) show that with the X-axis.

63. To deduce the equation of the parabola when referred to the tangents at the extremities of the latus-rectum as axes.

FIG.

The equation

of the parabola if = 2px

We

.

.

wish to ascertain what

curve

is

Let

P

DY DX

referred to (x

,

,

.

when

referred to

the figure,

OC

CP )

we

OY, OX,

is

(1).

this equation as axes.

becomes when the

,

be any point of the curve

i/)

(OC,

From

32.

= (x,

y),

;

and (DC

then, Fig. 32 ,

CP

)

have,

= DC - DO = DK + C M - DO;

=

(a*

,

tf).

THE PARABOLA.

93

but

DK = x

= r^C, C M = /cos 45 =

cos 45

V2

,

=

x

hence

_

;

*

V2

V2

Substituting the values of x and (2/

^

- *T -

V2

?/

DPF, we

we

in (1),

(C

In order to simplify this expression the triangle

;

?

= MP - C K

..,

| *

DO =|

-\

CP

have, also,

V2

p --^=-- 5r 2 V2

JK

V2

We

-X-,

+

let

-^

)

2/

have, 2

DP = a

.

-

-

;

then from

(2)

have,

DF =p =

=

a cos 45

-^=-

.

V2<

Substituting this value of

by

2,

we

have,

x )2

(/

+

2

?/

or,

Adding 4

a?

2

ic

^

= 2a 2

a:

/

.-.

transposing, x ...

or,

is

a2

4- ?/)

2

^

2

= 4;ry,

2

<X+/-a)

x

(V

and multiplying through ,

/+

a2

= 0.

both members, the equation takes the form

?/ to

or

in (2)

+y -[-

^x

f

a

=

2x *y *

yx

=

-J-

-+- ?/

a

2

a;

?/

=a ...

symmetrically, dropping accents,

the required equation.

^

^ ;

;

(3)

PLANE ANALYTIC GEOMETRY.

94

/EXAMPLES.

What is the polar equation of the parabola, the pole 1. being taken at the vertex of the curve ? Ans. r 2p cot cosec 0.

=

Find the equation of the tangent to each of the following value of the subtangent in each case parabolas, and give the 2 Ans. y = x + 1 2. = 4 x at (1, 2). 2. :

y

3.

;

x2 2

4.

v/

5.

x2

= 4 y at (- 2, 1). = - 6 x at 6, ord = 8 y at (abs +, = 4 ax at (a, 2 a). = at (m, m). = ^y at (abs +, (

2

6.

?/

7.

y

2

2

8.

a;

9.

x*

y + 1 = =3 Ans. 2 y + ^rcs. x + y = 2

Ans.

a;

+

oj

+). 2).

2.

;

12.

;

;

4.

??ia;

= 2py

at

(abs-, \

p).

p8

Write the equation of the normal to each of the following parabolas 10. 11.

12.

13.

:

= 16 x at (1, 4). To x = - 10 y at (abs +, - 2). mx at To if = m, m). To x = 2 m?/ at ofo To

if 2

(

2

,

-

o

\ \

=

K

* of a parabola is x* _L_ y ? curve the of are the co-ordinates of the vertex 14.

The equation

Ans.

5

-a, -- a 4 V4 I

= 4x

and the line y which is, the of the tangent equation required 15.

Given the parabola

if

what

x

=

;

to the line, (a) parallel (b)

perpendicular to the line. x Ans. (a) y 1, (b) y

= +

+ + 1 = 0. a;

THE PARABOLA.

95

16. The point (1, 2) lies outside the parabola if = 6x; what are the equations of the tangents through the point to

the parabola ?

The point (2, 45) is on a parabola which is symmetri with respect to the X-axis required the equation of the parabolk, the pole being at the focus. - 2 V2) x. Ans. if (4 17.

cal

;

=

=

18. The subtangent of a parabola 10 for the point (5, 4) required the equation of the curve and the value of the sub normal.

Ans.

64.

The tangent

focal line

drawn

to the

= ^x;*. 5

>

parabola makes equal angles with the

the point of tangency

to

y*

and

the axis of the

curve.

From

We

Fig. 31

we

have,

FT

= FO + OT =

4-

x".

have, also,

= DC = DO + OC = f + FT =

FP"

.-.

The

triangle

x".

FP".

FP"T is FP"T

=

therefore isosceles and FTP".

/

65.

To find

the condition that the line

in order to touch the parabola y 2

= 2 px.

y

= sx + c must fulfil

Eliminating y from the two equations, and solving the resulting equation with respect to x, we have,

p

;

- SC

V

(C8

- PY ~ C S

2 2

m

for the abscissae of the points oflntersection of the parabola and line, considered as a secant. When the secant becomes

PLANE ANALYTIC GEOMETRY.

96

a tangent, these abscissas become equal but the condition for in the numerator of equality of abscissas is that the radical ;

(1) shall

be zero

hence

;

c s

2

= 0,

= --

c

or, solving,

2

2^

(cs

S

the condition that the line must

is

fulfil in

order to touch the

parabola.

COR.

we

1.

Substituting the value of sx c, y

= + V = sx +

have,

c in

the equation

(2)

^-S

for the equation of the tangent in terms of

its slope.

To find the locus generated by the intersection of a tan to it from the focus as the point of gent, and a perpendicular the curve. around moves tangency 66.

The equation

of a straight line through the focus

I?

is )

In order that this line shall be perpendicular to the tangent

y

we must hence

r

have,

s

= sx + L =

.

.

.

(2)

;

S

y

=

-

+ ^-

-

;

-

(3)

the equation of a line through the focus perpendicular to the tangent. Subtracting (3) from (2), we have is

or,

x

= 0, =

is the But x equation of the required locus. the perpendiculars from the Y-axis the of hence, equation

for the

;

THE PARABOLA. focus

to the

97

on the tangents of a parabola intersect the tangents

Y-axis.

67.

To find

the locus generated by the intersection of two tan to each other as the point of tan-

gents which are perpendicular curve. gencij moves around the

The equation

of a tangent to the parabola

y

The equation

= 8X + f-g

-

-

-

=

-l*-fA

...*

=-

we

(1),

-

-

|.

is

...(2)

S

Subtracting (2) from

Art. 65 (2),

(1)

of a perpendicular tangent

y

is,

have,

(3)

the equation of the required locus. But (3) is the equa hence, the intersection of all perpendicu lar tangents drawn to the parabola are points of the directrix.

is

tion of the directrix

;

68. Two tangents are drawn to the parabola from a point without ; required the equation of the line joining the points of

tangency.

be the given point without the parabola, and let f Since (x y is X be the points of tangency. ( 2, 2/2) (X on both tangents, its co-ordinates must satisfy their equations

Let (V, >

?/)

,

2/")?

~)

;

hence, the equations of condition,

y

y"

2/2/2

The two points

yy r

yy

+X"\

+x

(x

of tangency

satisfy

or

=p(x

=P

=p =p

(* (x

+ +x

z ).

(x",

y"),

(x 2

,

2/ 2 )

must therefore

),

)

.

.

.

(1)

Since (1) is the equation of a straight line, and is satisfied for the co-ordinates of both points of tangency, it is the equation of the line joining those points.

PLANE ANALYTIC GEOMETRY.

98 69.

To find the equation of the polar of the pole

(x,

y

)

with

=

2 px. parabola if The polar of a pole with regard to a given curve is the line generated by the point of intersection of a pair of tangents drawn to the curve at the points in which a secant line through

regard

to the

the pole intersects the curve as the secant line revolves about the pole.

a course of reasoning similar to that of Art. 49, prove the required equation to be

By

As the reasoning by means fectly general, the pole

may

we may

which (1) is deduced is per be without, on, or within the

of

parabola.

COR.

1.

If

we make,

hence, the directrix

is

in (1), (x

r ,

y

)

= | 0\ I \z (

,

we have

the polar of the focus.

70. To ascertain the position and direction of the axes, other than the axis of the parabola and the tangent at the be referred its equation will vertex, to which if the parabola

remain unchanged in form.

FIG.

33.

THE PARABOLA. Since the equation

to retain the

is

99

form

= 2px ... (1) y* y = 2p x ... (2) f

*

let

when referred to the we are now seeking.

be the equation of the parabola

axes,

It is whose position and direction be the whatever that obvious at the outset position of may must be -axis Y new the each to the axes relatively other, on the must be new the and the to origin curve, tangent 0, a curve for, if in (2) we make x = 0, we have y = ;

which we can only account for by assuming the Y -axis and the new origin in the positions indicated. This conclu verified by the analysis which sion, we shall see, is fully result

follows.

Let us refer the curve to a pair of oblique axes, making in the any angle with each other, the origin being anywhere

The equations

plane of the curve. Art. 33

(1),

x

y

= a -j- x cos 6 + y cos = b x sin + y sin r

-f-

q>

cos <p)

y

Now,

+

+2

(1),

(b

sin

-f-

we have, x

2

sin 2

(9

+2

=

2

(a)

sin

(b)

sin 2

(d)

= 0, then sin

and

p

(b sin

sin g>

b

2

= 0. 2 pa = 0.

-

b sin

p

g>

sin g>

.

p

q>

.

satisfied

:

= 0. cos

<p

= 0.

=

and sin 2 = 0; i.e., conditions assumed value of 6. But is -axis makes with the old X-axis

(b) are satisfied for this

X

the angle which the new hence, these axes are parallel. If (a, b) be a point of the parabola ?/ 2 pa is an analytical expression of the fact

;

= 2px,

that the

.

we must have the following conditions

(d)

(a)

(1), q>

cos 0) x -f b 2 pa (3) in order that this equation shall reduce to the same

form as

If

q>

g>.

Substituting these values in 2 2 x y sin sin sin 2

?/

transformation are,

of

new

orisrin lies

on the curve.

;

then

hence

(c)

b

2

=2

shows

PLANE ANALYTIC GEOMETRY.

100 S

1!L^

If

cos

= tan

= P, then

<f

b

gp

p

But

(d) is satisfied.

-

the

is

o

whose ordinate is b, Art. slope of the tangent at the point is the slope of the new Y -axis and tan. hence, the new Y -axis is a tangent to the parabola at the point whose ordinate is b .-. at (a, b) .% at the new origin.

57, (5),

;

<p

;

;

COR.

1.

Substituting

= cos =

that cos

and (d) in (3), recollecting after dropping accents,

(a), (&), (c),

we have,

1,

2

-^S-*sin* y % =p

y

f

letting

or,

,

sin

we have

y

"

cp

= 2p x

2

.

.

.

(4)

for the equation of the parabola when referred to O The form of (4) shows that for every value Fig. 33.

y has two

for x,

OX

Y OX

values, equal but of opposite sign

bisects all chords,

drawn

parallel to

OY

and

line

which

is

,

,

assumed ;

hence,

therefore a

diameter of the parabola.

A DIAMETER of a curve NOTE. tem of parallel chords.

is

a

bisects

a sys

\l

To show that the parameter of any diameter

71.

to

the focus four times the distance from that diameter cuts the curve. Draw the focal line FO and the normal

to

Since the triangle 2

O FN

=

Since 61.

O FT

N

Hence

is

a

normal

at

in the triangle

AO = FO

AO N =

,

<y>

and

sin 2

g>

= FO

NO A, AO = AN cot v = FO

2 sin

cp

AN = p,

FO A 2 sin

In the triangle

hence

cos

equal

O N, Fig. 33. Art. 64, the angle isosceles;

is

<r.

O

is

the point in which

<JP

=p

sin

.

cp

-

cos

cp

;

(jp

cos

<JP.

Art.

THE PARABOLA. .-.

P

FO =

2 sin 2

q>

i* -4 Sin* 2p = 4 FO

But

101

go

f

.-.

72.

.

To find the equation of any diameter in terms of the

and

slope of the tangent

The equation

of

= AO =

y But from the triangle b

the semi-parameter.

any diameter as

AO N,

= AX cot =

*L_ tan

is

y

=

,

Fig. 33,

is

we have,

<p

hence

OX

b.

CP

=

<:

s

-

.

.

.

(1)

the required equation.

To show that the tangents drawn at the extremities of chord meet in the diameter which bisects that chord. any 73.

R

FIG.

Let

Y

f

(x

,

/),

P"

(x",

y")

FP";

then

y

-

y

=

y

34.

be the extremities of the chord

y"

n (_;*>... (1)

PLANE ANALYTIC GEOMETRY.

102 its

is

The equation

equation. are

P"

of the tangents at

P

(V,

y")

0",

^-p(* + y/ -*(* Eliminating x from

(2)

-

+

and

(

2) (

*")

3 >

(3) by subtraction,

we

have,

intersection of the tangents. for the ordinate of the point of ~~

x

But

X

(see (1)

"

is

n

f

the reciprocal of the slope of chord

Hence, since the chord

).

are parallel,

we

P

P"

P

and the tangent

P",

YT

have,

y

-y"

Substituting in (4)

it

becomes

-*.value of y with (1) of the preceding is on the diameter. the see that point of intersection

Comparing article,

we

this

EXAMPLES. 1.

y J

= 2.

y

What must be the value of c 4 x _L_ c may touch the parabola

?/

What

=3x + 3.

in order that the 2

Ans.

is

.

the line the parameter of the parabola which 2 touches ? is

to the parabola y slope of a tangent ? the of tangent the equation

The

What

line

=8x?

2

=

6x

is

=

= 3 ^ _L

3.

i

on a tangent to a parabola required the equation of the parabc the equation of the tangent and ,= the slope of the tangent - 16 *. l; 4a 4

The point

(1,

3) lies

;

4.^

_

^=

THE PARABOLA.

=

8 x

is

y

In the parabola y z diameter whose equation 5.

what

the parameter of the

is

W=

103

?

Ans.

136.

6. Show that if two tangents are drawn to the parabola from any point of the directrix they will meet at right angles.

From

=

2 the point 8 x 5) tangents are drawn to ?/ required the equation of the chord joining the points of Ans. 5 ?/ 4^ 8 0. tangency.

7.

8.

What

(2,

the

are

equations of the tangents to

which pass through the point

2,

(

;

+ = = 6x 2

?/

4) ?

Find the equation of the polar of the pole in each of the following cases 9.

Of (-

:

3)

1,

with regard to y 2

= 4*. Ans.

10.

Of

(2, 2)

with regard to if

=

4

11.

Of

12.

Given the parabola

(ay b)

with regard to y

?/

=x

Q.

2y

+ 2a; + 4 = 0.

= 4 x. Ans.

2

+2=

x.

Ans. 2

2x

3y

2x

by

and the point

(

2 a 4,

= 0.

10)

;

to

find the intercepts of the polar of the point.

a

Ans. 13.

The latus-rectum of a parabola 8x

of the line y

-

4,

=4

= 0.

;

= 4:,b=-.

5 required the pole

Ans.

Given

=

=

(i,i).

10 x and the tangent 2 y x 10 required the equation of the diameter passing through the point of tangency. Ans. y 10. 14.

if-

;

=

GENERAL EXAMPLES. 1.

Assuming the equation

point on the curve directrix.

is

of the parabola, prove that every equally distant from the focus and

PLANE ANALYTIC GEOMETRY.

104

Find the equation of the parabola which contains the

2.

points

(0, 0), (2, 3),

2, 3).

Ans.

What

3.

(-

are the parameters of the parabolas

through the point

3 x2

which pass

(3, 4) ?

Ans.

Find the equation of that tangent 4 = 0. 2 x parallel to the line y 4.

Ans.

The parameter

5.

y

=

of a parabola

is

4

;

?/

8y

Y, and

f.

= 9x

which

is

16 a

9

=

0.

required the equation

perpendicular to the line tangent 2 x -\- 2. Give also the equation of the normal which is

which

line

the

of

to

2

= 4 y.

is

line. parallel to the given

=

A

2 4 x makes an angle of 45 with the tangent to ?/ X-axis required the point of tangency. Ans. (1,2).

6.

;

Show

that tangents

drawn

at the extremities of a focal

chord 7.

Intersect on the directrix.

8.

Meet

9.

That a

focus 10. 11.

is

at right angles.

line joining their point of intersection with the perpendicular to the focal chord.

Find the equation of the normal

in terms of its slope.

Show that from any point within may be drawn to the curve.

the parabola three

normals 12

Given the parabola

r

=

to construct the tan-

+ cos 6 vectorial angle = 1

60, and to find the gent at the point whose line. initial angle which the tangent makes with the 60. Ans. 61

=

13.

Find the co-ordinates of the

extremity of the latus-rectum being

pole, the its polar.

normal

at one

THE PARABOLA.

=

In the parabola y 2 4 x what chord which the point (2, 1) bisects ? 14.

is

105 the equation of the

Ans. 15.

The polar

of

any

point in a diameter

is

=2x

y

3.

parallel to the

ordinates of that diameter. 16.

The equation

of a chord of y*

=

10 x

is

y

=2x

1

;

required the equation of the corresponding diameter. 17.

Show

that a circle described on a focal chord of the

parabola touches the directrix. 18.

The base

of a triangle

gents of the base angles vertex is a parabola.

=

b.

=2a

and the sum of the tan

Show

that the locus of the

Required the equation of the chord of the parabola =2px whose middle point is (m, n}. n = x m A

19.

y-

Ans.

.

p 20.

A

angle

=

n

=

chord of the parabola if 2 px makes an with the X-axis required its length.

focal cp

y

;

Ans.

-?-.

Show

that the focal distance of the point of intersec to a parabola is a mean proportional to the focal radii of the points of tangency. 21.

tion of

two tangents

Show that the angle between two tangents to a parab one-half the angle between the focal radii of the points of tangency. 22.

ola

is

23.

=

The equation

is y = a

;

of a diameter of the parabola y 2 2 px required the equation of the focal chord which this

diameter bisects. 24. The polars of all points on the latus-rectum meet the axis of the parabola ?/ 2 2 px in the same point required the co-ordinates of the point.

=

;

Ans.

I

PLANE ANALYTIC GEOMETRY.

106

CHAPTER

VII.

THE ELLIPSE. 74.

THE ellipse

that the

sum

is

the locus of a point so moving in a plane

of its distances from

constant and equal to a given called the Foci of the ellipse.

two fixed points

line.

If

is

always

The the

fixed points are points are on the

the given given line and equidistant from its extremities, then line is called the TRANSVERSE or MAJOR Axis of the ellipse. 75.

To deduce

the equation of the ellipse, given

the,

foci

and

the transverse axis. Y

FIG.

Let F, F! be the

OY

J_ to

AA

foci

at its

co-ordinate axes.

and

35.

AA

the transverse axis.

Draw

OX

as the

middle point, and take OY,

THE ELLIPSE. Let also

P

PD

=

Let

FF

X

AA =

V?/

From

the

+ (x

2

X

2

c)

mode

y) are the co-ordinates of P.

=

=

20F

201\

hence

and

FPD

Vy +

=2

=2a Vy + (x - c) + V y + /

and FjPD, we

2

have,,

2

(#-{- c)

.

we

.

.

(a)

have,

2

+ c) = 2 a 2

(x

;

.

.

.

(1)

and reducing,

clearing of radicals,

+ x -c 2

(if

and

r

;

2

a2

FP =

c,

2

r

of generation of the curve,

r -f

or,

(x,

the right angled triangles

=

r

2a,

= /.

From

draw

;

OY.

to

Then (OD, DP)

FP

Draw PF, PFi

be ary point of the curve. ||

107

)

2

=

x2

a 2 (a 2

-

2

c )

...

(2)

As

this equation (2) expresses the relationship between the co-ordinates of any point on the curve, it must express the

relationship between the co-ordinates of every point the required equation.

;

hence

it is

Equation

(2)

elegant form.

may be made, however, to assume Make x = in (2), we have,

=

2 ?/

a2

-

c

2

for the square of the ordinate of the point in

curve cuts the Y-axis this distance

by

we

b, 2

.-.

2

2

C

(= OB

which the

/2

).

Eepresenting

2 ?

-b* ...

Substituting this value of

or,

2

have,

=a _ c = a 2

OB

i.e.,

;

a more

c

2

in (2)

(3)

and reducing, we have,

symmetrically,

for the equation axes.

of ellipse

when

Let the student discuss equation

referred to

(4).

its

centre and

See Art. 12.

PLANE ANALYTIC GEOMETRY.

108 COR.

we make

If

1.

2

x -f y

which

is

COR.

=a =a 2

the equation of a

circle.

we interchange a and

If

2.

we have,

in (4),

b

2

for the equation of an ellipse lies along the Y-axis.

COR. 3. If (z y we have from (4) ,

y

f

)

and

=

*L

(z",

(^

-z

we

b in (5),

have,

whose transverse axis

y")

- a/

are

2

+

)

(=2 a)

two points on the curve,

and

= -^ (a -

2

2

y"*

z"

^

-

+

)

;

x ) (a (a ) (a ordinates the of any two points on the of i.e., the squares other as the rectangles of the segments in ellipse are to each

hence, y

2

*

::

:

(a

y"

:

which they divide the transverse COR.

4.

= x a and y and dropping accents, ^x 2

tf ,f

_|_

2 atfx

for the equation of the ellipse, of co-ordinates.

The

line

BB

,

Fig. 35,

x")

;

.

axis.

By making x

after reduction

76.

x"}

=

=

in

y

.

.

.

(4),

we have

(7)

A

being taken as the origin

is

called

the

CONJUGATE

or

and A from the figure that bisects all lines drawn through it and terminating the point the CENTRE of is called For this reason in the curve.

MINOR

axis of the ellipse

VERTICES of the

ellipse.

the points

;

A

are called the

It is evident

the ellipse.

The is

~ ** - -?

ratio

called the

this ratio is

a

ECCENTRICITY always

<

See

.

a

1.

(3)

of the ellipse. of c

The value

ures the distances of the foci F,

F from x

Art. 75

...

(1)

It is evident that

= i Va - b 2

the centre.

2

meas

THE ELLIPSE. If a

=b

then

in (1),

If b

=

=

e

=1

in (1), then e

when

the ellipse becomes a

zero. i.e.,

;

when

the ellipse becomes a

becomes unity.

straight line the eccentricity

77.

i.e.,

;

becomes

circle its eccentricity

109

To find the values of the focal

radii,

/, of a point on

r,

the ellipse in terms of the abscissa of the point.

The FOCAL RADIUS of a point on the ellipse of the point from either focus.

From equations

(a),

Art. 75,

= Vy +

we

2

r

from the equation of the

;

ellipse, Art.

^_ ( a 2 a2

2

have,

cf

(x

the distance

is

x 2\

^2

we

75

(4),

_

&_ X 2

have,

.

a2

hence, substituting *

\

o 2 ex

/ t/V -

+

2 ex

T

= r

Similarly

we

c

a

78.

c-

,.-

a

x

;

See

ex.

(1) Art.

76 ... (1)

find

r

%

=

,

<Z-

a hence

2

-(X

+

= a + ex

.

.

.

(2)

Having given the transverse

axis

and the

foci of

any

ellipse, the principles of Art. 75 enables us to construct the ellipse by three different methods.

First Method.

verse axis

AA

.

Take a cord equal Attach one end of

in length to the trans at F, the other at F

it

.

Place the point of a pencil in the loop formed by the cord and stretch it upward until taut. Wheeling the pencil around, while keeping, the point on the paper and tightly pressed

PLANE ANALYTIC GEOMETRY.

110

against the cord, the path described will be an arc of the After describing the upper half of the ellipse, re ellipse. move the pencil and form the loop below the transverse axis.

By

a similar process the lower half

FIG

may

be described.

It is

36.

evident during the operation that the sum of the distances of the point of the pencil from the foci is constant and equal to the length of the cord i.e., to the transverse axis. Take any point C on the transverse axis Second Method. ;

and measure the distances

CA

A C,

AC.

With F

as a radius describe the arc of a circle

;

as a centre

also with

F

and as a

CA as a radius describe another arc. The points these arcs intersect are points of the ellipse. which in B/ R, the radii two other points P, P may be interchanging By A smooth curve traced through a number of determined. centre and

points thus located will be the required ellipse.

Let the axes AA = 2 on any straight edge = OA = a and DL = OB

Third Method.

Lay given. will do)

off

KD

MX

a,

BB =

2 b be

(a piece of paper == b. Place the

the position indicated in the straight edge on the axes in and L slide along the axes, the point Then as figure.

K

D

THE ELLIPSE. describe the

will

DKE

For from the

ellipse.

are similar triangles

DK = --

Ill

:

DL lji

-a

_-

;

i.e.,

=

x

_ b

-//

DLH

figure

2

(x

and

and y being

the co-ordinates of D).

Hence, squaring, clearing of fractions, and transposing, we have 2 z a?tf _j_ &2X = a b ^

That is the locus described by D is an ellipse. ment based upon this principle is commonly used

An for

instru

drawing

the ellipse. 79.

To find the

The

latus rectum or

parameter of an ellipse. parameter of an ellipse is the double or dinate passing through the focus. The abscissas of the points in which the latus rectum 2 b -JSubstituting either pierces the ellipse are x of these values on the equation of the ellipse latus rectum, or

=

we have Hence

y

2

=

2

a2

(a

vV

-

Latus rectum

Forming a proportion from

2

(a

.

- b )) = 2

=2y=

.

a2

9 A2

-

-

a

.

y

...

=

.

a (1)

this equation there results,

2y:2b::b:a ; hence i.e.,

2y

:

2 b

::

2

b

:

2 a

;

the latus rectum is a third proportional to the two axes.

EXAMPLES. Find the semi-axes, the eccentricity, of each of the following ellipses

and the latus rectum

:

1.

2.

3 x 2 -f 2 y 2

= 6.

^,^ = L

3.

4>

x2 4

+ 3 if = 2. o __ 6 = 2 2

,

3.2

PLANE ANALYTIC GEOMETRY.

112

60y c

i

-\-

9

x-

=

O

7

a.

o.

Write the equation of the 9.

The transverse

=

V -^-

l

-f-

m

?i.

having given the distance between the foci

ellipse

= 10

axis

9

x-

;

:

or

Ans.

25 10.

Sum

= 18

of the axes

;

difference of axes

36

=

Transverse axis

10

-4^

the conjugate axis

;

=

l.

9

= 6.

Ans. 11.

$-

+

l-

= 1.

9

= y2

the

transverse axis.

x*

A 12.

tween

Transverse axis

= 20

conjugate axis

;

=

distance be

foci.

Ans.

13.

=1

4y2 25

25

Conjugate axis

= 10

;

-

distance between foci

Ans.

-j-

y

= 50.

2

= 10.

^+

y*

= 25.

Given 3y 2 + 4 a? 2 = 12 required the co-ordinates of the point whose ordinate is double its abscissa. 14.

15.

;

Given the

ellipse 3

2 ?/

+ 2 x = 12, and the line y 2

x

1

;

to find the co-ordinates of their points of intersection.

X1 (--=3-1, and the abscissa of a 64 15 .-2

16.

Given the

f

ellipse

point on the curve

=

;

required the focal radii of the point. 8T Ans. r 7 T%, /

=

= V

THE ELLIPSE. 80.

113

To deduce the polar equation of the

ellipse, either focus

being taken as the pole.

FIG.

Let us take F as the

=

From

the figure,

x

i.e.,

and

pole,

the co-ordinates of any point r a ex (1) we have, .

37.

P .

.

-\-

or,

reducing,

P FA)

=

From

(r, (9)

be

Art. 77

;

e

(ae

-f-

(1),

we have

r cos 0),

we have ,

*L

_ 1

+ e cos

for the polar equation of being taken as the pole. From Art. 77 (2),

FP

We

,

r cos 0.

Substituting this value of x in

=a

(FP

(1)

OD = OF + FD

= ae r

let

of the ellipse.

...

(2)

the ellipse, the right-hand focus

= / = a + ez

.

readily determine from this value

1

-

e

cos

for the polar equation of the ellipse, the left-hand focus being taken as tlje pole.

PLANE ANALYTIC GEOMETRY.

114 COR. If

= a (1 = FA, = / a (1 + = F A. =a-a r = a (1 - e r

0,

0)

e)

= 90,

If

2

= (1 + = FA = a (1 - = F A FM r = a (1 - e

r

ft

/ If

= 270,

If

= 360,

To deduce

e)

,

e)

.

2

)

r

/ 81.

,

= a (1 - e) = FA, = a (1 + = F A. e)

the equation of condition for the supplemental

chords of an ellipse. Y

FIG. 38.

Let AP,

AP

The equation

be a pair of supplemental chords. of a line through A (a. o) is y

2

= a(l-e )=a-a- -=-^- = - = FN. = 180,

If

)

= FM.

= r

2

=

s (a;

r/).

THE ELLIPSE. The equation

V =

Where

A

of a line through s

(x

?/

= ss

a, 6) is

(

+ a). we must have

these lines intersect 2

115

f

2

a 2)

(x

.

.

.

(1)

In order that the lines shall intersect on the ellipse their equations must subsist at the same time with the equation of the ellipse

Dividing

*- 5 (a ^^ (1)

or

(2),

ss

=-

.

.

for the required condition. COR. If a the ellipse b,

=

ss

f

2)

we have

by

comes

(

= -

.

(3)

becomes

-a circle

and

(3)

be

1,

a relationship heretofore deduced. Art. 40 (1). SCHOL. The preceding discussions have developed a remark able analogy between the ellipse and circle. As we proceed we shall find that the circle is only a particular form of the ellipse and that all of the equations pertaining to it may be deduced directly from the corresponding equations deduced for the ellipse

82.

Let P"S

by simply making a

To deduce the equation of the tangent P"

(x",

y"),

V

cuts the ellipse.

As

= b in those equations.

2

= i!_(V2

2 2/"

the ellipse.

,

we must have

the points are on the ellipse, ,

to

(x ?/ ) be the points in which a secant Its equation is, therefore,

= 4 (a- a-

AA

3/2}

2 *"

)

...

(3)

PLANE ANALYTIC GEOMETRY.

116

FIG.

39.

These three equations must subsist at the same time subtracting (3) from (2) and factoring, we have

-

2/0

(/ r

hence

y x

r

+ 2/0 =

P_

T

^

becomes

v^alue in (1) it /

/,2

Eevolving the secant

_|_

line

"

becomes a tangent and x we have

f

(

"-

=

,

x",

y

=

y"

;

for the equation of the tangent

(x",

.

.

(5)

P"

?/")

and

secant

hence, substituting,

,*;... 1 2

P"

x

r

xx b

-

P (x y ) will approach When this occurs the

i.e.,

a2

X)

upward about the point

the other point of intersection will finally coincide with it.

or

x

-^-7T7

>-

hence

-

- i/\ -x

Substituting this

;

(4)

THE ELLIPSE. COR.

If b

= a,

117

we have _i_

yy n

I

a* for the equation of the tangent to the circle. SCHOL. If we make x and y successively tion of the tangent (5),

we have y

=

-

f

See Art. 41

=

and x

values of the variable intercepts

=

,,

hence

y"

IP

,

OT d2 x

=

..

and x

y

These values

,

(6).

equa

= ^ for x

the

"

y ,

in the

OT, Fig. 39

;

.

in the equation 2

2 *" .

^~"

?/

_

J

give, after reduction,

for the equation of the ellipse, the intercepts of its tangents

on the axes being the variables. 83.

To deduce the value of the sub-tangent.

Making y

=

.-.sub-tangent

in (5), Art. 82,

= DT = ^1 -

we have

x"

COR. If .

.

a*

~x x

x"

b = a,

=

"*

ff

then from Art. 41, Schol. a 2

sub-tangent in the circle

=

2 x"

rr<2

2

.

SCHOL. The value of the sub-tangent being independent of the value of the minor axis (2 b) it follows that this value is the same for every ellipse which is concentric with the given

ellipse,

and whose common transverse axis

is

2

a.

PLANE ANALYTIC GEOMETRY.

118

of condition that a line shall pass through the centre of the ellipse and the point of tangency is, Fig. 39,

84.

The equation

=

tx",

y"

.-.

the slope of this line

t==

The

is

^7

slope of the tangent at

a2

(x",

y")

is,

Art. 82,

y"

Multiplying,

member by member, we have

But

(3)

Art. 81

...

i.e.,

ss

=

the tangent to the ellipse

and

the line joining the centre

and

the point of tangency enjoy the property of being supplemental I)

chords of an ellipse whose semi-axes bear

is

COR. If

s

parallel

to

then s = t a diameter of the

=

f

t,

;

parallel to the tangent diameter.

chord

85.

is

to

each other the ratio

-.

chord if one supplementary ellipse, the other supplementary i.e.,

drawn

at the extremity of that

84 afford us two different principles of Arts. 83, a given of constructing a tangent to the ellipse at

The

methods point.

First Method.

Art. 83, Schol. Let draw the ordinate

P",

Through point. until it meets the circle described P"

of the ellipse Join at F.

(AA )

P"

and

in

T

;

F; draw P"T

Fig. 40, be the given

P"D

and produce

it

upon the transverse axis tangent to the circle

FT

will be the required tangent.

THE ELLIPSE.

FIG.

P"

||

86.

from

RA

40.

Draw

Art. 84 and Cor.

Second Method. centre, and to

A

draw

AR

119

||

to

P"R

;

will be tangent to the ellipse at

To deduce the equation of the normal

The equation

of

any

-

line

through

=

P"R

P"T

P".

to the ellipse.

P"

-

(x",

through the

drawn through

y"),

Fig. 39, is

S (X y (1). In order that this line and the tangent at shall be perpendicular their slopes must satisfy the condition ?,"

.

.

.

X")

P"

1

We

+ S8 = o

.

.

.

(x" ,

y")

(2).

have found Art. 82 for the slope of the tangent

/==

;

"5-^ hence, the slope of the normal

Substituting this value of

for the equation of the

is

s in (1),

normal

to

we have

the ellipse.

PLANE ANALYTIC GEOMETRY.

120 COR.

1.

.

=

a

If

then

b,

-

"

yx

which

xy"

becomes, after reduction, 0,

normal

line to the circle.

To deduce the value of the sub-normal

=

Making y

...

-

is the equation of the

87.

we

(3)

in the equation of the normal, (3), Art. 86,

ON = x

have, Fig. 39,

*x",

Sub-normal COR.

If

1.

=

a

b,

then

Sub-normal for the

circle

=

x".

EXAMPLES. Deduce the polar equation of the ellipse, the pole being with the X-axis. the centre and the initial line coincident

1.

at

A Ans.

r

=

-. 2

sin

2

+b

2

cos 2

the

of following Write the equation of the tangent to each of the sub-tangent in each case. value the and give ellipses,

2z

2.

El

3.

2

+ 4y = 38at 2

+ 2.

(1, 3).

Ans.

!.

+

+ 6 y = 19

;

18.

at (1, ordinate positive).

1,

An,.

4.

x

._l,

a

at (2,0). 0.

5

6

.

.

2

^ + 3^- 11

J/l

a

+

_ o

=

1,

at

(2,-

at (0,

1)

V)

4as

.

_ 3y . 11;i

.

THE ELLIPSE.

8.

9.

+ te - 2, 2

*

y

m +

=

2 ?/

- V2 -

at (1,

at (afo

1,

+,

121

6).

.5).

Write the equation of the normal to each of the following the sub-normal. ellipses, and give the value of 7/

+ 4 x = 39,

at

if

+2

= 44,

at

2

2

10.

3

11.

4

12.

^!_

13.

-+{ o o

14.

^1

_|_

4-

ic

2

=

j

7/2

=

m

2

2 ?/

-f- ?

2

1,

at (1,2).

at

^ ord

=mn 2

1,

(

2 ,

ord negative).

ord

at

1?

2

ic

2,

(

1.

a,

15.

(3, 1).

).

_|_^

at (m, o).

=

+

2x 6 of a chord of an ellipse is y the equation of the supplementary chord, the axes of the ellipse being 6 and 4 ? Ans. y f f x 16.

what

The equation

=

17.

;

is

Given the equation

j-

y

MID

= 1,

and

2

?/

+

=

;

.

re-

quired the equation of the tangents to the ellipse at the points in which the line cuts tne curve. 18.

2

Given the ellipse

= 0; (a)

%

= 1,

and the

line

x

y

required

The equation "

(b)

(-

"

of a tangent to the ellipse

||

"1

to the line. "

"

"

-{-

PLANE ANALYTIC GEOMETRY.

122 19.

The point

(4, 3) is

1G

+ 1P

outside the ellipse

1;

required the equations of the tangents to the ellipse which pass through the point. 88. The angle fanned by the focal lines drawn to any point of an ellipse is bisected by the normal at that point.

FIG.

41.

Let P"N be a normal at any point

We

have found, Art.

From

Art. 76

P" (a;",

?/ ).

= OF = ae; NF = OF OX = ae NF = OF + OX = ae + NF NF r

FP"

:

FP"

.-.

::

:

NF NF

::

P"F

hence

e?x"

But

P"F,

87, that

we have OF

.-.

Draw

(a

:

ex")

:

::

e*x"

(a

:

(

ex")

(a

+

FP"

:

ex")

F

= e (a - ex = e (a + +

)

ex")

ex")

Art. 77, (1)

P".

ff

and

(2)

3

THE ELLIPSE. The normal,

F

P"F

into

cent sides.

therefore,

divides the base of the

triangle

two segments which are proportional to the adja

Hence ==

FP"jST

SCHOL.

123

1.

If

P"T

F

F

P"N.

be a tangent drawn at

=

P"C

FP"T

P",

we must have

;

between a FP"N and the the angle Hence, right angle (= FP"N). tangent to the ellipse makes equal angles with the focal radii for each of these angles is equal to the difference

drawn

to

SCHOL.

method

the point of tangency. 2.

The

principles of this article afford us another

drawing a tangent to the ellipse at a given point. be a point at which we wish to draw a tangent. Let Pro to R, making P"R = FP" join F and R. duce F A line J_ to FR will be tangent to the ellipse P"T, drawn through of

P"

P"

;

P"

at

P".

89. To find the condition that the straight line y must fulfil in order that it may touch the ellipse

4 + y. = a

If

b

= SX O

a2

we obtain the

I

7

-f-

c

i.

2

we consider the line y

= sx

as a secant

and combine the equations

-\- C,

O

?

lr

co-ordinates of the points of intersection.

Eliminating y from these equations, we have

-

sa 2 c

ab

for the abscissas of the points of intersection. Now, when the secant line becomes a tangent, these abscissas become equal.

Looking

at (1)

we

see that the condition for equality of ab-

PLANE ANALYTIC GEOMETRY.

124

numerator shall disappear

scissas is that the radical in the

hence

sV

2

or is

s*a

-f-

+

2

^

2

b

2

-

c

=c

2

=

;

0,

2 .

.

.

(2)

the required condition.

COR. If we substitute the value of equation of the line, we have

y

= SX -.

V*

2

+&

a2

for the equation of the tangent

drawn from

c

...

(2) in the

(3)

the ellipse in terms of

to

its

slope.

To find the

90.

gent

to

a tan a from focus as

locus generated by the intersection of

and a perpendicular

the ellipse

to it

the point of tangency moves around the curve. The equation of a straight line through the focus (ae, o)

is

=

s (x ae). y In order that this line shall be perpendicular to the tangent

=

its

SX y be must equation

y

J-

V* 2

= - i (x -

a

+b

ae)

.

2

(1),

.

.

(2)

S

we now combine (1) and (2) so as to eliminate the slope the resulting equation will express the relationship be (s), tween the co-ordinates of the point of intersection of these hence it will be the lines in every position they may assume If

;

equation of the required locus. Transposing sx to the first member in (1), of fractions

and clearing

(2)

and transposing, we have y sy

= Vs a + b x = ae.

SX -}-

2

-t-

2

2 .

Squaring these equations and adding, remembering that 2 a z e 2 Art. 76, we have, ab

=

or

,

THE ELLIPSE.

125

hence, the circle constructed on the transverse axis of the ellipse

and

the locus of the intersection of the tangents let fall from the focus on them.

is

the perpen

diculars

This circle

known

is

(See Fig. 45.

ellipse.

as the Major-Director circle of the

)

To find the locus generated by the intersection of two tangents which are perpendicular to each other as the points of tangency move around the curve. 91.

The equation

of a tangent to the ellipse

= SX +

y

The equation

s

a2

+b

is

2 .

.

.

(1)

of a tangent perpendicular to (1)

is

hence, by a course of reasoning analogous to that of the pre

ceding

article,

we have xz

+ ?/ = a

2

2 -f b

.

.

.

(3)

The required locus is, therefore, a circle 2 ellipse and having its radius equal to Va 92. out

;

concentric with the -f-

b

2 .

Two tangents are drawn to the ellipse from a point with required the equation of the line joining the points of

tangency.

Let

P

7

?/), Fig. 42, be the given point, and let be the points of tangency. Since 2/2) (x y ) is a point common to both tangents, its co-ordinates must satisfy

?2

(a:

P"

,

V

(*2>

their equations

;

x

(x" ,

f

,

hence, x"

a2

x xz a2

Hence

(x",

y")

and

~W_ y yz

(a; 2

.

1

y z ) will satisfy the equation

y"),

PLANE ANALYTIC GEOMETRY.

126

As (1) is the equation of a straight line, and is satisfied for the co-ordinates of both points of tangency, it must be the equation of the straight line which joins them. 93.

regard

To find the equation of the polar of the pole (V, /), with to

the ellipse 2 ~

_

1--1

FIG.

42.

the aid of Fig. 42, and a course of reasoning similar to PiP", the polar to P may be shown to be

By

that of Art. 49, the equation of

xx a2 COR. PI

P

(^i>

,y_y_

,

1.

b*

If the polar of the point P (x f ?/) passes yi)j then the polar of P l (x^ T/J) will pass

(V, /).

,

through through

(See Art. 50.)

94. To deduce the equation of the ellipse when referred to a pair of conjugate diameters as axes. A pair of conjugate diameters of the ellipse are those diam-

THE ELLIPSE.

127

which if the ellipse be referred its equation will contain the second powers of the variables. only eters to

The equation axes

of the ellipse

when

referred to its centre and

is

we

refer the ellipse to a pair of oblique axes at the centre, we have, Art. 33, Cor. 1, origin If

x y

=x =x

having the

+ i/ cos + y sin y

cos

(p

sin

for the equations of transformation.

Substituting in

(1),

we

have

+

2 2 (a sin 2 2 (a sin

+ W cos 0) x + (a sin + tf cos ij sin + 6 cos cos a y = aW ... (2) 2

2

2

2

2

2

q>

q>)

2

(?)

q>

for the equation of the ellipse referred to oblique axes. But, by definition, the equation of the ellipse when referred to a

pair of conjugate diameters contains only the second powers of the variables hence ;

a- sin

sin

2

cp

-f b cos

=

cos y

the condition that a pair of axes must conjugate diameters of the ellipse. is

Making the

co-efficient of

.

.

.

(3)

fulfil in

x y equal to zero in

order to be

(2),

we have

after dropping accents 2 2 (a sin

+6

2

cos 2 0) x 2 -f (a 2 sin 2

<p

+6

2

cos 2 <p)

y

2

=a6 2

2

... (4)

for the equation of the ellipse when referred to a pair of con jugate diameters. This equation, however, takes a simpler form when we introduce the semi-conjugate diameters. Mak

ing y

=

and x

= 0,

successively, in (4),

a2

sin 2

(p

-M

2

cos 2

+6

2

cos 2

<p

we have

J

PLANE ANALYTIC GEOMETRY.

128 r

which a and have (5). we

in

ft

From

represent the semi-conjugate axes.

a? sin

2

+

|72

2

2

2 ft

cos 2

S

2

= ^L _

^2

;

2 ft

in Substituting these values of the co-efficients

(4),

we

have,

after reduction,

for the required equation.

COR. As equation (6) contains only the second powers of the variables, it follows that each of the two diameters to which the curve is referred will bisect all chords drawn parallel to the other.

SCHOL. The equation of condition for conjugate diameters be put under the forms (3) may

= --7 2

tan 6 tan

.

.

(7)

.

<p

we

see that the

81, Comparing this expression with (3) same result was obtained for the supplementary chords of an

Art.

if ellipse; hence, Fig. 40,

ary chords, then

RP",

A R R A be

PR",

,

a pair of supplement

drawn through the centre

parallel

Again to these chords, will be a pair of conjugate diameters. we see that the same relation Art. with 84, (1) comparing (7) for a diameter and the tangent drawn at ship was obtained diameter and P"T its hence, Fig. 40, if P"R be a :

extremity be a tangent drawn at its extremity, then PR", drawn through to RP". the centre parallel to P"T, is the conjugate diameter a The equation of condition (7) being single equation con ;

unknown quantities (tan. 0, tan. assume any value we please for one of them, and will make known the value of the other hence,

taining two

;

there are

an

infinite

<r),

we may

the equation in the ellipse

number of pairs of conjugate diameters.

THE ELLIPSE.

129

To find the equation of a conjugate diameter.

95.

Let

P"R,

RP

be a pair of conjugate diameters.

to find the equation of

R/P

We

wish

.

Y

FIG.

The equation P" y"}

(*",

of the

wr"

By

tangent line

i

in/ yy

drawn through

#

f

_i

2

P R/

Art. 94, Schol., the diameter

equation must be the same

its

P"T,

is

xx a2

hence

43.

is parallel to P"T as that of the tangent,

;

the constant term being zero.

xx .

=

.

.

.

(1)

or

...

is

the equation of a diameter expressed in terms of the co

(2)

ordinates of the extremity of its conjugate diameter. COR. Let s represent the slope of the diameter P"R

from

(2) _

bV 2

.

.

?/

2

jr

a

2

1 s

;

then,

PLANE ANALYTIC GEOMETRY.

130

^=^=

since

i ;

s

DP" y"

hence we have

-~x a

y=

2

!

.

.

s

(3)

for the equation of a diameter in terms of the slope of its

conjugate diameter.

To find the co-ordinates of either extremity of a diam

96. eter,

the co-ordinates of one extremity of

its

conjugate diameter

being given.

Let Let

P"R

(x",

RT

and

y")

Fig. 43, be a pair of conjugate diameters. be the co-ordinates of We wish to find the ,

P".

f

P

terms of the co-ordinates of The equation of condition that P (a/, y ) shall be on the diameter P R is, Art. 95, (1) co-ordinates (x

,

y

X

Since

P

f

(x

,

)

of

P."

X"

y ) is on the V2 o/2

Eliminating y and

we

in

a;

,

ellipse,

we have

also

successively, from these equations,

find

These expressions, taken with the upper signs, are the co ordinates of P taken with the lower signs, they are the ;

co-ordinates of 97.

R

.

To show that the sum of the squares on any pair of

semi-conjugate diameters

on the semi-axes. Let P"

(x",

y"}

and

is

P

equivalent f

(x

,

y

).

to the

sum of the squares

Fig. 43, be the extremities

THE ELLIPSE.

131

two semi-conjugate diameters. Let OP" = a the triangles ODP", OD P we have, from then, of any

,

OP =

b

r ;

,

...

"

and

V

But, Art. 96,

cc

=x +y

2

(1)

2

2

2

.

.

.

(2)

=~

and

hence

=

2

b

Adding

(1)

and

+

2

b

2

i/

2 x"

_

98.

2

a

hence,

_|_

P"R

The area

(

= 2a

),

is

+ -TT

v

//

i

\

.

;

2

==

-

2

-f

ft

.

.

(4)

.

PR

(

=

2ft

of the parallelogram

From

the figure sin (180

),

to

the rectangle constructed

Fig. 44, be

OP"TR

is

OP"TR

=

0) )

= afb

OP"

=a

sin

-

sin (<p

P"OR

sin (>

(9)

.

<9)

.

.

any two conju

= area BB H H.

P"P.

P"P

(<p

area of

equivalent

To prove that area CTC T

OR X

.-.

2

/

To show that the parallelogram constructed on any two

gate diameters.

= a!

2_ = l

-f 6

conjugate diameters on the axes.

Let

(3}

.

we have

(3),

70X

but

.

a2

;

(1)

PLANE ANALYTIC GEOMETRY.

132

B

c Fm.

From

the triangles

OD P

sn

Hence

sin

,

44.

0DP",

we have .y"

OF

(qp

Mb ab

a fb

0)

=

V

sin

go

cos 6

cos

g>

sin

THE ELLIPSE.

133

Substituting this value in (1) and multiplying through by 4,

we have

area

OP"TB

area

CTC

X 4 = 4 ab = area BB H H. ;

i.e.,

T"

To show that the ordinate of any point on

99.

the ellipse is

the ordinate of the corresponding point on the circumscribing circle as the semi-conjugate axis of the ellipse is to the semitransverse axis. to

Let

P

(*

,

DP

,

DP"

/) and

Since

P

Since

P"

be the ordinates of the corresponding points

P"

(x",

r

(x

,

(x" ,

y

) is

y"}

y"}.

on the

is

ellipse,

on the

circle

we have

whose radius

is a,

we have

Dividing these equations, member by member, we have /

=

7

2 ,

(since

oj

=

a;"

)

;

PLANE ANALYTIC GEOMETRY.

134 Similarly

we may prove x1

x.2

:

::

a

:

that b,

any point on the ellipse, and x 2 the corresponding abscissa of a point on the inscribed circle.

where

Xi is the abscissa of

is

The

100.

method

principles of the preceding article give us a of describing the ellipse by points when the axes are

given.

From

0, Fig. 45, as a centre with radii equal to the semiDraw any OB describe the circles RA, BCB

A

axes OA, radius

draw

N

OB

MN

||

OA

R

,

Since MJST a point of the ellipse. have we the of O, triangle is

BD D N D R OM OR ::

:

i.

y

e.,

.

of the larger circle, cutting the smaller circle in in to cutting the or din ate let fall from

b y"

^

:

is

M

;

N

;

parallel to the base

;

;

hence, the construction.

101.

To show that the area of the

the circumscribing circle as the to its

semi-major

ellipse is to the

semi-minor axis of the

axis.

FIG.

46.

area of

ellipse is

THE ELLIPSE. Inscribe in the ellipse any polygon its vertices draw the ordinates

from

135

AEE E E E A 2

1

ED^D^

3

4

/ ,

and

producing etc. Joining

them upward to meet the circle in P, P 1? P 2; these points we form the inscribed polygon

etc.,

APP^P^A

in

the circle.

Let (x, y ), (x yj, y 2 ) etc., be the co-ordinates of r P, P 15 P 2 etc., and let (a;, y), (x , /), be the co-ordi y"), etc., nates of the corresponding points E, 1} 2 etc., of the ellipse. ,

(x",

,

(aj",

R R

,

Then Area Area RPD.R, Area

hence

But, Art. 99,

a

Hence

AreaRDD^ Area

We

may prove in like manner that every corresponding pair of trapezoids bear to each other this constant ratio; hence, by the Theory of Proportion, the sum of all the trapezoids in the ellipse will bear to the sum of all the trapezoids in the circle the same ratio. these sums 2,t and Representing 2T, respectively, we have 2*

2T As

this

=

by

b

a

relationship holds true for any

zoids, it holds true for the limits to

number

of trape

which the sum of the

trapezoids of the ellipse and the sum of the trapezoids of the circle approach as the number of trapezoids increase.

PLANE ANALYTIC GEOMETRY.

136

But these the circle

limits are the area of the ellipse

area of ellipse area of circle

b

a

Since the area of the circle

COR.

and the area of

hence

;

area of ellipse TT

Since

a2

a2

:

TT

a 2 we have ,

b

a

area of ellipse TT ab TT ab

.. TT

is

:

:

= :

TT

ab.

TT b*,

we

see that the area of the ellipse tween the areas of the circumscribed

a mean proportional be

is

and

inscribed circles.

EXAMPLES. 1.

What must

be the value of

c

in order that the line

y = 2 x + c may touch the ellipse ! + JL == i ? 9 4 2

Ans.

c

= 5.

of an ellipse is 10 what must be the value of the semi-conjugate axis in order that the ellipse 2.

may

The semi-transverse

touch the line 2 y

-\-

;

x

14

= 0?

Ans. 3.

What

~,2 x

_i_

whose inclination

drawn

,,,2 y

i

to X-axis

locus of

-4 a2

ellipse

= 45

?

the intersection of the tangents to the

+

=l b

2

at the extremities of conjugate diameters

required

its

V24.

6

5

The

=

are the equations of the tangents to the ellipse

"*"

4.

b

is

an ellipse

;

equation.

Ans

,

+

=2.

THE ELLIPSE. 5.

Tangents are drawn from the point a

+

frequired the

2 2/

137 (0,

8) to the ellipse

= i; the

equation of

joining the points Ans. 8 y 1

line

of

=

tangency.

Eequired the polar of the point following ellipses 6.

*2

9.

What

with respect to the

:

= 9.

3/ 2

(5, 6)

0.

=1.

7.

are the polars of the foci ?

x

Ans.

=

-e

10.

What

is

the pole of y

= 3x

-f-

1 with respect to

(-

.

11.

The

line 3

y T2

_^L

=

5x 2

_i_

4

7/ iL

is

_i

.

9

required the equation of the conjugate diameter. Ans. 20 y 12.

A

12, 9).

a diameter of

+ 27 # = 0.

pair of conjugate diameters in the ellipse ?/ 2

T 2

_-f JL.^1

16

make

angles whose tangents are

with the X-axis 13.

9

What

is

;

o

o

^ and

^,

respectively,

required their lengths.

the area of the ellipse

?4

+

-

= 1?

10 .

2

TT

VlO.

PLANE ANALYTIC GEOMETRY.

138

The minor

14.

axis of an ellipse

to the area of a circle

and its area is equal what is the length

is 10,

whose diameter

16

is

;

25.

Ans.

of the major axis ? 15. The minor axis of an ellipse is 6, focal radii to a point on the curve is 16

axis, the distance between the

foci,

and the sum of the required the major

;

and the

area.

GENERAL EXAMPLES.

What

1.

(2, 4) 2.

(6.

(

The major

4) is

3.

The

lines

y

;

1

=

=

18, and the point the equation of the ellipse. required

axis of

on the curve

tal chords

x

an

ellipse is

13

+ 6 and y = ~ x -f ^ are supplemen

drawn from the extremities of the transverse

of an ellipse 4.

the equation of the ellipse which passes through the centre being at the origin ?

is

2, 4),

;

The minor

axis of an ellipse

=

is

12,

and the

centre divide the major axis into four equal parts the equation of the ellipse. 5.

sum is

in

Assuming the equation of the distances of

constant and 6.

=

of the ellipse

any point on the for a point

an ellipse whose eccentricity

7.

What

;

foci

and

required

show that the

ellipse

from the

foci

to the transverse axis.

The sub-tangent

of the ellipse.

axis

required the equation of the ellipse.

is

whose abscissa ;

is

2

is

=6

required the equation AnSf

^L

+ ]_ =

l.

are the equations of the tangents to

which form with the X-axis an equilateral triangle

?

that the tangents drawn at the extremities of any chord intersect on the diameter which bisects that chord. 8.

Show

THE ELLIPSE.

139

9. What are the equations of the tangents extremities of the latus-rectum ?

drawn

at the

10. Show that the pair of diameters drawn parallel to the chords joining the extremities of the axes are equal and conjugate.

A chord

11.

of the ellipse H

16

:

"~9~

passes through the point (2, 3) and is bisected by the line x required the equation of the chord.

=

y

;

What

12.

are the equations of the pair of conjugate 2 9x 2 144 which are equal ? ?/

eters of the ellipse 16

+

=

diam

Show

that either focus of an ellipse divides the major two segments whose rectangle is equal (a) to the rectangle of the semi-major axis and semi-parameter to 13.

^

axis in

;

the square of the semi-minor axis. 14.

Show

(&)

that the rectangle of the perpendiculars let fall is constant and equal to the square

from the foci on a tangent of the semi-minor axis. 15.

A system

of parallel chords which make an angle whose 2 with the X-axis are bisected tangent by the diameter of an ellipse whose semi-axes are 4 and 3; required the equation of the diameter.

=

16. Show that the polar of a point parallel to the conjugate diameter. 17.

on any diameter

is

Find the locus of the vertex of a triangle having given = 2 a, and the product of the tangent of the angles

at the base

=

.

^2

Ans.

PLANE ANALYTIC GEOMETRY.

140 18.

Find the locus of the vertex of a triangle having given = 2 a, and the sum of the sides = 2 b.

the base

Ans 19.

-

Find the locus of the intersection of the ordinate of

the ellipse produced with the perpendicular let fall from the centre on the tangent drawn at the point in which the ordi

nate cuts the ellipse. 20. Find the locus generated by the intersection of two tangents drawn at the extremities of two radii vectores (drawn from centre) which are perpendicular to each other. a2 Ans. a^

bW =

21.

A

line of fixed length so

remain in the co-ordinate axes

by any point of the

;

moves that

its

extremities

required the locus generated

line.

=

The angle AOP" y (Fig. 45) is called the eccentric Show that (a; ?/) P the of (# tf) on the ellipse. angle point and from these values of the co-ordinates (a cos cp, b sin 22.

,

,

=

<p)

deduce the equation of the 23.

ellipse.

Express the equation of the tangent at

(a;",

y")

in terms

of the eccentric angle of the point.

Ans.

- cos

a 24.

If

(a/, ?/),

(x",

?/ )

4-

y~

sin

<p

=

1.

b

are the ends of a pair of conjugate and qo show that cp

diameters whose eccentric angles are

90.

q>

,

THE HYPERBOLA.

CHAPTER

141

VIII.

THE HYPERBOLA. THE hyperbola is the locus of a point so moving in a the difference of its distance from two fixed points that plane The fixed is always constant and equal to a given line. 102.

points are called the Foci of the hyperbola. If the points are on the given line produced and equidistant from its extremities, then the given line is called the TRANSVERSE Axis of the hyperbola.

103.

and

To deduce the equation of the hyperbola, given the foci

the transverse axis.

FIG.

Let F,

F

be the

OY 1 to AA

at its

foci,

and

47.

AA

the transverse axis.

Draw

OX

as the

middle point, and take OY,

PLANE ANALYTIC GEOMETRY.

142

PF

;

draw

P

Let

co-ordinate axes.

PF.

also

PD

||

OY.

= (x, y) are = AA 2 a, FF = 2 OF

Then (OD, DP)

the co-ordinates of P.

Let

== 2

OF =

the right angled triangles

FPD

=r

Draw

be any point of the curve. to

2

c,

FP =

r

and F P

r .

From r

= Vy + 2

From

the

and /

2

(x

mode

~c)

= Vy +

and

2

(x

of generating the curve,

/

F PD, we

+ c)

have

2 .

.

.

(a)

we have

= 2 a.

?

Hence, substituting,

Vy +

+ c) 2

2

or,

(x

V?/

2

+

(x

-

clearing of radicals and reducing,

_a (c*

2

)

- a if = a

x2

2

2

2

(c

2

c)

= 2a;

.

.

.

(1)

we have

-

a 2 ) ...

(2)

for the required equation. This equation, like that of the ellipse (see Art. 75), may be put in a simpler form.

Let

c

This value in

or,

2

=b

a2

2 .

.

(3)

.

(2) gives, after

changing signs,

symmetrically,

4a~

b~

= l...(5)

for the equation of the hyperbola and axes.

Let Cor.

1.

the

student

If b

discuss

a in

x2

(5),

if

=

this

when

referred to

its

centre

(See Art. 14)

equation.

we have a2

.

.

.

(6)

The curve represented by this equation is called the ^Equi lateral Hyperbola. Comparing equation (6) with the equation of the circle

we the

see that the equilateral hyperbola bears the

common hyperbola

same

that the circle bears to the

relation to

ellipse.

THE HYPERBOLA. COR.

If (V,

2.

143

are the co-ordinates of two y ) and we have from (4) (x" ,

y")

points on the curve,

y

hence

= -*! (x -

2

2

a

/

2

f

2 : ?/"

a 2) and if 2

::

a) (x

(x

=

+ a)

_

-*i

a2

(35"*

a)

:

(x"

^)

(x"

;

-f a)

;

the squares of the ordinates of any two points on the hyperbola are to each other as the rectangles of the segments in which they divide the transverse axis. i.e.,

COR.

= x a and y and dropping accents,

By making x

3.

after reducing

V-

2

b

x2

+ 2 ab x = 2

for the equation of the hyperbola,

104.

From equation b =

4-

...

A

(3) Art. 103,

=y

f

in

(4)

we have

(7)

being taken as origin.

we have

vr~

this distance off above

and below the origin on the the points B, B Fig. 47, Art. 103. The line BB i called the CONJUGATE Axis of the hyperbola. The points A and A are called the VERTICES of the curve. The Laying

Y-axis,

we have

7

,

bisects all lines drawn through it and point terminating in the curve for this reason it is called the CENTRE of the ;

hyperbola.

The

ratio

Vet 2

-I-

62 -

=

a

1 a

>

=

e.

See

(3) Art.

103 ... (r

called the ECCENTRICITY of the This ratio is hyperbola. 2 1. The value of c b 2 measures the evidently -t Va distance of the foci F, from the centre. is

=

>

F

If b

=a

in (1),

we have

e

= V2

+

for the eccentricity of the

equilateral hyperbola.

105. To find the values of the focal radii, r, r of a point on the hyperbola in terms of the abscissa the of point. From equations (a) Art. 103, we have

PLANE ANALYTIC GEOMETRY.

144

From

the equation of the hyperbola, (4) Art. 103,

we have

Hence, substituting r

=

x2

\/

V a2

b

2

+x

2

a2

= hence

r

Similarly,

we

106.

Tio

and

= ex

a

2 ,

Art. 104 (1),

:

a ...

= ex + a

.

.

(1)

.

(2)

construct the hyperbola having given the transverse

the foci of the curve.

FIG.

First Method. foci.

+c

find

/

axis

x

,

2 ex

48.

Let A A be the transverse axis and F, F the ruler whose length is L and attach

Take a straight-edge

,

THE HYPERBOLA.

145

its ends at F so that the ruler can freely revolve about Cut a piece of cord so that its length shall be that point. L 2 a, and attach one end to the free end of the ruler,

one of

=

and the other end to the focus F.

Place the ruler in the

position indicated by the full lines, Fig. 48, and place the Stretch point of a pencil in the loop formed by the cord. the cord, keeping the point of the pencil against the edge of

the ruler.

If

we now

revolve the ruler upward about

F

,

the

the pencil, kept firmly pressed against the ruler, point will describe the arc AP of the hyperbola. By fixing the end of the ruler at F, we may describe an arc of the other of

It is evident in this process that the difference of the distances of the point of the pencil from the foci F ,F, is always equal to 2 a.

branch.

Take any point D on the transverse axis. Second Method. Measure the distances A D, AD. With F as a centre and A D with F as a centre and as a radius describe the arc of a circle ;

AD

as a radius describe another

arc.

The

intersection

of

these arcs will determine two points, P 1? P 2 of the curve. By interchanging centres and radii we may locate the points E, 1? ,

R

2

In this manner we

n the other branch. >

many

may determine as may require.

points as the accuracy of the construction

107. To find the lotus-rectum or parameter of the hyperbola. The LATUS-RECTUM, or PARAMETER of the hyperbola, is the double ordinate passing through either focus.

Making x

we have

=

-j-

y

V&

2

+

=

.-.

a

b

2

in the

2y

=

Forming a proportion from

2y:2b i.e,

equation of the hyperbola

.

a this equation,

we have

::b:a-,

the latus-rectum of the hyperbola is a third proportional to

the axes.

PLANE ANALYTIC GEOMETRY.

146 108.

The equation of the

and axes

is

a 2y 2

The equation and axes

-+-

b

2

x2

ellipse

=ab 2

of the hyperbola

in the sign of b 2 2

its

centre

its

centre

.

when

referred to

is

expressions

-

referred to

2

Comparing these equations, we is

when

.

If,

see that the only difference

therefore, in the various analytical for the ellipse, we substitute

we have deduced

+ V

for & 2 or,

what is the same thing, I 1 for b, we will obtain the corresponding analytical expressions for the b

,

hyperbola.

To deduce the equation of the conjugate hyperbola. hyperbolas are CONJUGATE when the transverse and con jugate axes of one are respectively the conjugate and trans verse axes of the other. 109.

Two

FIG.

Thus

49.

AA

be the transverse axis of the hyper in Fig. 49, if for its conjugate axis, then the hyperbola

bola which has

which has

BB

BB

for its transverse axis

and

AA

for its conjugate

THE HYPERBOLA. axis

its

is

147

conjugate; and, conversely, the hyperbola whose is BB and conjugate axis is AA has for its

transverse axis

conjugate the hyperbola whose transverse axis

whose conjugate axis is BB We have deduced, Art. 103,

is

AA

and

.

*a1

= b

(5),

1

.

.

2

(1)

for the equation of the hyperbola whose transverse axis lies wish to find the equation of its conju along the X-axis.

We

has

from the figure that the hyperbola which for its conjugate axis for its transverse axis and

It is obvious

gate.

BB

AA

bears the same relation to the Y-axis as the hyperbola whose and conjugate axis is BB bears to the transverse axis is

AA

X-axis

;

hence, changing a to I and b to

a,

x to y and y to x

we have

in (1),

-

=

-...

for the equation of the conjugate hyperbola to the hyperbola

whose equation is (1). Comparing (1) and (2) we see that the equation of any hyperbola and that of its conjugate differ only in the sign of the constant term.

COR.

Since

V^

2

+ u2 = V

any hyperbola and those of

The

2

its

+b

2 ,

the

focal

distances

eccentricities of conjugate

hyperbolas, however, are

For the hyperbola whose semi-transverse axis a and semi-conjugate axis is b, we have

not equal.

Art. 104, (1) e

For

its

=

Vj

of

conjugate are equal.

.

conjugate hyperbola, we have

is

PLANE ANALYTIC GEOMETRY.

148

EXAMPLES. and

Find the semi-axes, the eccentricity of each of the following hyperbolas

the- latus-rectum

:

1.

9

2.

^1

2,2

_

4 X2

-

-^!

4 2

3.

?/

=

36

5.

3 y2

6.

a?/

1.

-

2

2

cc

bx*

2

=

12.

=-

aft.

9

-

16 x 2

= - 16.

7.

4

4 z2

4.

=_

-

16 y z

= - 64.

8.

y

2

- if = m. -

=

ma?

TO.

Write the equation of the hyperbola having given

foci

axis

= 12

axis

= 10

The transverse

9.

:

the distance between the

;

= 16.

10.

The transverse

=

parameter

;

8.

Ans.

25 11.

Semi-conjugate axis

=6

;

The equation

-L 36 64 X2

13. The conjugate axis double the conjugate.

is

10,

x2

The transverse

axis

is

8,

.<

1.

-

3 if

3 yz

=

3? -

Io

and the conjugate axis

=

distance between foci.

16

6.

+ 6 = 0.

and the transverse axis A Ans

14.

2

?/

j

2 of the conjugate hyperbola, x

Ans.

l.

= 10.

the focal distance

Ans.

12.

-= 20

-fID

is

THE HYPERBOLA. 15.

149

Given the hyperbola 2

.2

10

,,2

jr

_i

.

4

required the co-ordinates of the point whose abscissa its

is

double

ordinate.

2

l 16.

Vf Vj

Write the equation of the conjugate hyperbola

to each

of the hyperbolas given in the first eight examples above. 2 4 x* = Given the hyperbola 9 y focal radii of the point whose ordinate

36

17.

is

;

=1

required the

and abscissa

positive. 18.

Determine the points of intersection of

49 = ^~

= + $16

and

1.

16

1.

110. To deduce the polar equation of the hyperbola, either focus being taken as the pole.

F as the pole, Fig. 47. PFD) = (r, 0) be the co-ordinates

Let us take Let (FP,

on the curve.

From

From

r = ex - a = OD OF + FD r cos x = ae

Art. 105, (1),

FP =

i.e.,

Fig. 47,

.

of

any point

P

we have .

.

(1)

;

0.

-f-

and reducing, we have

Substituting this value in (1)

a

(I

1

-^ e

...

cos

(2)

for the polar equation of the hyperbola, the right being taken as the pole.

Similarly from Art. 105,

(2),

a-* / = *1 e cos

hand focus

we have

2

)

for the polar equation, the left

.

(3 )

hand focus being the

pole.

PLANE ANALYTIC GEOMETRY.

150

=

If

COR.

r

0,

/ 90,

r

= =

r

=

a

If

If

a

ae2

-f-

=

= a ae = FA = a + ae = F A.

=

-

=

,

semi-latus rectum.

= semi-latus rectum. = = a a r = - a + ae = FA, T = a -ae= - F A b = semi-latus rectum. a 4- ae r =

ae 2

= ISO

,

.

_

If $

2

270

2

a

=

r

a

ae

2

= semi-latus rectum.

= a

111. To deduce the equation of condition for the supple mentary chords of the hyperbola. By a method similar to that of Art. 81, or by placing b~ 2 for b in (3) of that article, we have =

aa hence, the product of the slopes of any pair of supplementary chords of an hyperbola is the same for every pair. If

COR.

=

a

we have

b,

1, or, s

ss

=

1 ,

S .

.

tan

sum of

= cot

;

the two acute angles which

any pair of sup make ivith the plementary chords of an equilateral hyperbola hence, the

X-axis

is

equal

to

90.

To deduce the equation of the tangent

112.

to

the hyperbola.

adopted in the entirely analogous or substituting 57 Arts. 41, 82, circle, or ellipse, or parabola, 2 2 find we Art. of b for b in (5) 82,

By

a

to that

method

;

-

xx"

~a7"

yy"_ "

b

2

1

a)

to be the equation of the tangent to the hyperbola.

THE HYPERBOLA.

151

To deduce the value of the sub-tangent. operating on (1) of the preceding article (see Art. 83),

113.

By we

find

Sub-tangent

114.

The

hyperbola

(0,

=

x"

-

x

=

*-

"*

~

a* .

slope of a line passing through the centre of an is 0) and the point of tangency (x",

y")

x

The

slope of the tangent t

=

Art. 112, (1)

is,

^L

.*1

a2

y"

member by member, we have

Multiplying these equations,

= Comparing

a)

$...

(1) of this article ss

=

tt

.

.

.

with

(1) of Art. Ill,

we

find

(2)

Hence, the line from the centre of the hyperbola to the point of tangency and the tangent enjoy the property of being the supplemental chords of an hyperbola whose semi-axes bear to each other the ratio a

=

.

COR. If s = t, then s t if one i.e., supplementary chord of an hyperbola is parallel to a line drawn through the centre, then the other supplementary chord is to the ;

parallel

tangent drawn to the curve at the point in which the line through the centre cuts the curve. 115. The preceding principle affords us a simple method of drawing a tangent to the hyperbola at any given point of the curve.

PLANE ANALYTIC GEOMETRY.

152

FIG.

50.

Let P be any point at which we wish to draw a tangent. 7 to P O join C and A. Join P and 0, and from A draw A C will be the required from P to CA The line P T, drawn 7

||

;

||

tangent.

To deduce the equation of the normal to the hyperbola. can do this by operating on the equation of the tangent, 2 in the equa as in previous cases, or by changing b into tion of the normal to the ellipse, Art. 86, (3). By either 116.

We

b"

method, we obtain

for the required equation. 117. To deduce the value of the sub-normal. By a course of reasoning similar to that of Art. 87,

sub-normal

COR. If

b

d

we have

*/ %

$-

= a, sub-normal

i.e.,

=

7/2

in the equilateral

=

x"

;

hyperbola the sub-normal

the abscissa of the point of tangency.

is

equal to

THE HYPERBOLA.

153

EXAMPLES. Deduce the polar equation of the hyperbola, the pole

1.

being at the centre. 7.2

=

^!__ a 2 sin 2

b

-\-

2

cos

2

Write the equation of the tangents to each of the follow ing hyperbolas, and give the value of the sub-tangent in each case.

-

3.

4

-

9 y2

2.

-

"I

._

= - 36, 1,

at

=

at

(

4 ^2

J

= _ 36j = abj

5.

y

6.

a?/

7.

^1-J^ = 1,

2

m

at

~=T7T

-

i

-

4 x2

to 2

(4,

(5,

ord.

ord.

4 Ord >

+).

+).

+)

at

at

at

(aft, ord. +).

(Vm,

0).

7^

Write the equation of the normal to each of the above hyperbolas, and give the value of the sub-normal in each 8.

case. 9.

=

;

x 6 The equation of a chord of an hyperbola is y what is the equation of the supplemental chord, the

axes of the hyperbola being 12 and 8 ? A Ans.

10.

y

= 4- x

-

8

_

.

Given the equations -

Y-

=~

-

-f-

1,

and y

-x=

;

required the equations of the tangents to the hyperbola at the points in which the line pierces the curve.

PLANE ANALYTIC GEOMETRY.

154 11.

9

One

of z

?/2

the supplementary chords of the hyperbola 144 i s parallel to the line y -=x what are

_ if} x = _

;

the equations of the chords ?

16

Ans.

aj

y

Given the

12.

2x

hyperbola

What

is

if

T is

118.

=

6

3 the

required

;

at the positive

end of

the equation of a tangent to

x2

which

3 if

normal

equations of the tangent and the right hand focal ordinate. 13.

2

_16.

9

(

_ :

_,

"T

x

parallel to the line 2?/

+ l = 0?

lines drawn to any point by the tangent at that point. Art. equation of the tangent line,

The angle formed by the focal

of the hyperbola

is bisected

in the

o Making y we have

112, (1),

x

= OT.

=

Fig. 50.

x

From

OF = OF = ae OF - OT = FT = ae - -^ =

Art. 104, (1)

hence

;

9

x

OF

\ x

(ex"

+ OT = F T =ae + -^ = ~-a FT F T + a. JC

.-.

::

:

+ a);

**.

ex"

:

ex"

(ex"

a).

But from Art. 105 we have a FF = = FT +a -a FF FT FT F T FF FT ex"

ex"

.-.

Hence

:

PT

;

::

:

::

ex"

:

:

ex"

+ a.

;

FFF

divides the base of the triangle i.e., the tangent are proportional to the adjacent which into two segments, the angle at the vertex. bisect therefore must it sides ;

THE HYPERBOLA.

155

P N, Fig. 50, is perpendicular to bisects the external angle formed by the focal

Since the normal

COR.

the tangent,

it

radii.

The

SCHOL.

method

of

Let

point.

FP FT ,

.

P

this

article

us

gives

Draw

be the point, Pig. 50.

The

P T drawn

line

between the focal 119.

of

principle

another

drawing a tangent to the hyperbola at a given so as

the focal radii

to bisect

the angle

radii will be tangent to the curve at

To find the condition that the

fulfil in order that

it

line

y

P

= sx + c

.

must

touch the hyperbola

may

^ K=l a2

By

a

b

2

method similar s

2

a2

to that b

2

=c

employed in Art.

89,

we

find

2 .

.

.

(1)

for the required condition.

COR. 1. Substituting the value of equation of the line, we have y

= sx

sV -

4-

b

c

2

drawn from

...

(1) in

the

(2)

for the equation of the tangent to the hyperbola in terms of its slope.

120.

To find the to the

hyperbola and a perpendicular

to it

tangent as the point of tangency moves around the curve.

x 2 -f if is

of a a from focus

locus generated by the intersection

=

a2

.

.

.

(1)

the equation of the required locus.

(See Art. 90.)

121. To find the locus generated by the intersection of tivo tangents which are perpendicular to each other as the points of

tangency move around the curve. x* is

+if

=a

2

b*

.

.

the equation of the required locus.

.

(1)

(See Art. 91.)

PLANE ANALYTIC GEOMETRY.

156

Two

122.

without

tangents are draivn

;

tangency.

^_ a2

is

yy_ b

=i

.

.

.

2

(i)

To find the equation of the polar of

with regard

to the

the pole (x

1

,

/

),

hyperbola

_-!... is

hyperbola from a point

(See Art. 92.)

the required equation.

123.

to the

required the equation of the line joining the points of

the required equation.

(1,

(See Arts. 49 and 93.)

To deduce the equation of the hyperbola when referred a pair of conjugate diameters.

124. to

A pair

of diameters are said

to be

conjugate

when they

are so

related that the equation of the hyperbola, when the curve is referred to them as axes, contains only the second powers of the variables. ~2

a is

2

,,,2

%b

=1

.

:

.

2

(1)

the required equation, and

a 2 sin

sin

tan 6 tan

or

2 b cos 9 cos

<p

q>

=~

.

.

.

a2 -

is

y

= 0,

(2)

the condition for conjugate diameters. (See Art. 94.) From the form of (1) we see that all chords

COR.

parallel to

SCHOL.

drawn

one of two conjugate diameters are bisected by the other.

From

hence

Art. Ill, (1)

ss

= tan = tan

we have

tan

<p.

we have

=

tan cp i.e., if one of s s 0, a to is diameters tivo conjugate chord, the other conju parallel to the is diameter supplement of that chord. parallel gate If,

therefore,

;

THE HYPERBOLA. From

157

we have

Art. 114

= il; a2

hence If,

tt

therefore,

t

= tan = tan

we have

0,

t

= tan

<p

;

i.e.,

if one of

a tangent of the hyper the other conjugate diameter coincides with the line joining

two conjugate diameters bola,

tan

the point of tangency

125.

From

is

and

to

parallel

the centre.

the condition for conjugate diameters, tan

tan

9?

=

b

2 .

a*

we

see that the products of the slopes of any pair of conju is positive; hence, the slopes are both positive

gate diameters

or both negative.

gate diameters must

126.

It appears, therefore, that lie in the same quadrant.

any two conju

To find the equation of a conjugate diameter.

FIG.

51.

PLANE ANALYTIC GEOMETRY.

158 Let

P"R"

be any diameter

P

then

;

B/,

drawn through the /A

parallel to the tangent at Art. 124, Schol. jugate diameter. The equation of the tangent at

centre

(P

P"

N)

P"

a2

xx"

or

y

_

=

y")

is

2

b

hence, the equation of

(#",

will be its con

PB

is

yy"

_

*

"

7)2

^.^x a

.

.

(2)

2

y"

^V

But

is

= cot

=1

P"OX

;

s

the equation of a diameter in terms of the slope of

its

conju

gate diameter.

127.

To find the

diameter, the

of either extremity of a

co-ordinates

of one

co-ordinates

extremity of

its

conjugate

diameter being given.

Let the co-ordinates of

By

P"

y"),

(x",

Fig. 51, be given.

a course of reasoning similar to that of Art. 96,

The upper

the point

signs correspond to

lower signs to the point B/

(

x

y

,

P

we

f

(x

,

y)

;

find

the

}.

128. To show that the difference of the squares of any pair the of semi-conjugate diameters is equal to the difference of squares of the semi-axes. By a course of reasoning similar to that of Art. 97, or, by 2 2 2 for b in (4) of that article, we b for b\ - b substituting

-

find

a

*

-

b

2

=

a2

-

b

2 .

.

.

(1)

THE HYPERBOLA.

=

=

b COR. If a b, then a has equal conjugate diameters.

129.

;

i.e.,

the equilateral hyperbola

To show that the parallelogram constructed on any two

conjugate diameters on the axes.

By

f

159

is

to the

equivalent

rectangle constructed

a method similar to that of Art. 98,

4 a

Area

i.e.,

-

r

b sin (g>

0)

= 4 ab

we can show

that

;

MNM N = Area CDC D

Fig. 51.

.

EXAMPLES.

The

1.

line

y

= 2 x + c touches _ _ ""

-i

=

the hyperbola

.

4

9

what

,y

is

the value of

A

tangent to the hyperbola

c ?

Ans. 2.

10 its

Y-intercept

=

j i 12 2 required ;

its

V32.

-L,

slope and equation.

VI76

Ans.

A

2

tangent to the hyperbola 4?/ angle of 45 with the X-axis required 3.

=

=

_*!

has

c

;

y

2x

2

= =

VlTO x 6

+ 2.

makes an

its equation. 2 are drawn to the 9 x2 tangents hyperbola 4 y - 36 from the point (1, 2) required the of the chord equation of contact. ;

4.

=

Two

;

Ans. 9 x 5.

What

focus ? 6.

the equation of the polar of the Of the left-hand focus ?

What

is

is

the polar of 4 if x2

(1,

=

7.

)

=

8 y

36.

right-hand

with regard to the hyperbola

^5.

4?

Find the diameter conjugate

to

y

=x

2

cc

?/

= 4.

in the hyperbola

.2

J/_

16

Ans.

y

=

PLANE ANALYTIC GEOMETRY.

160 8.

Given the chord y

^ _ t.

= 2x =1

6 of the hyperbola

;

4

9

-f-

required the equations of the supplementary chord. Ans. y

=

x

.

9. In the last example find the equation of the pair" of conjugate diameters which are parallel to the chords. 2 x. Ans. y 2 x, 9 y

=

10.

144

The point

(5,

^)

lies

on the hyperbola 9

=

if

16 x 2

=

required the equation of the diameter passing through also the co-ordinates of the extremities of its conjugate it diameter. ;

;

130.

To deduce the equations of the

rectilinear asymptotes

of the hyperbola.

An ASYMPTOTE

of a curve

is

a

line

passing within a finite

distance of the origin which the curve continually approaches, and to which it becomes tangent at an infinite distance.

FIG.

52.

THE HYPERBOLA.

161

The equation of the hyperbola whose transverse axis along the X-axis may be put under the form y*

=

~ (^ -

2

)

...

lies

(i)

(ju

DD CO

The equations of the diagonals, constructed on the axes BB are ,

,

AA

=

y or,

squaring,

,

of the rectangle

-x, a

2y"

=

7,2

x2

.

.

.

(2)

a/ r

where y represents the ordinates of points on the diagonals. Let P (x, y] be any point on the X-hyperbola and let Sub (x, y } be the corresponding point on the diagonal DD tracting (1) from (2) and factoring, we have

D"

;

.

<y-z/)0/+2/)

y =

r

hence

y

D"P

=

=

2 ;

-

+

.

.

(3)

y y As the points P recede from the centre, 0, their ordi nates D"]ST, P N increase and become infinite in value when and P are at an infinite distance. But as the ordinates D",

D"

increase the value of the fraction

(3), which represents their becomes zero when y and y are and P are continually approach ing each other as they recede from the centre until at infinity they coincide. But the locus of during this motion is the

difference, decreases and infinite hence, the points ;

D"

D"

DD

infinite

diagonal hence, the diagonals of the rectangle constructed on the axes of the hyperbola are the asymptotes of ;

the curve.

Therefore

y

= + * x and y = - - x a a

are the required equations. COB. 1. If a b, then

= y =

i.e.,

45

-4-

x and

?/

=

x

;

the asymptotes of the equilateral hyperbola with the X-axis.

make

angles of

PLANE ANALYTIC GEOMETRY.

162

COR. 2. The equation of the hyperbola COD jugate to may be put under the form

Subtracting (1) from

_

y

y

hence, aw hyperbola and totes of each other.

COR.

3.

ti"

if/ie

p

i

its

p/ == _

conjugate are curvilinear

Subtracting (2) from

y hence,

we have

(4),

=

i,

y

-=

(1)

"P

asymp

we have

(4),

Y\"

"\v

y

+y

-jr~>

rectilinear asymptotes of

/

>

an hyperbola and of

its

con

jugate are the same. 131.

To deduce the equation of the hyperbola when referred

to its rectilinear

asymptotes as axes.

FIG.

The equation is

53.

of the hyperbola

when

referred to

OY, OX,

THE HYPERBOLA.

163

We wish to ascertain what this equation becomes when OY OX the rectilinear asymptotes are taken as axes. Let P be any point of the curve let Y OX = XOX = 0. Then (OC, CF) = (x, y) (OD, DP = (x y From the figure, OC = OK + DR CP = RP - DK ,

;

)

;

}.

,

;

;

y = But from the triangle OAB, we have x

i.e.,

=

hence, x

=

+y

f

(x

r

)

cos

;

(ij

-x

)

sin

B.

r

(x

-+-

+

a

Va^+ft

Substituting these values in f

(x or,

(9

+ 7/) 2

(y

we have

(1),

-

a

reducing and dropping accents, 2 a2 b Xy ...

= -+

J__

34

2

)

= a* -f ^

2 ,

(2)

for the equation of the hyperbola referred to its asymptotes. In a similar manner we may show that

is

the equation of the hyperbola conjugate to

ferred to

COR.

asymptotes as axes. Multiplying (2) by sin 2 6

(1),

when

re

its

we may

place the result in

the form

yx sin 2 that

is

therefore

=

^1A

+*

2 "

.

DP P H = ON. AH area ODP P = area OMAN

8jn

2

;

.

;

;

hence, the area of the parallelogram constructed upon the co ordinates of any point of the hyperbola, the asymptotes being axes, is constant and equal to the area of the structed upon the co-ordinates of the vertex.

rhombus con

PLANE ANALYTIC GEOMETRY.

164 132.

when

To deduce

the equation of the tangent to the hyperbola

the curve is referred to its rectilinear asymptotes as axes. Y,

FIG.

54.

a course of reasoning similar to that employed in Arts, 41, 57, 82, we find the required equation to be

By

y-Y = -^(*-*") X or,

...

(i)

symmetrically,

1^=2... COR. If we make y

=2 OM = x

But

=

x"

in (2),

=

x",

OT.

.-.

-

(2)

we have Fig. 54.

OM = MT

.-.

TD

= TD

;

hence, the point of tangency in the hyperbola bisects that por tion of the tangent included between the asymptotes.

THE HYPERBOLA.

D

Since

133.

=

+ b\ = a +b OT OT = a + b

4 2

or

x"y"

x"

.

a2

2

2 if

2

2 ;

2

.

i.e.,

we have

a point of the hyperbola,

is ?/")

(x",

(see Fig. 54)

165

.

.

.

(1)

hence, the rectangle of the intercepts of a tangent on the totes is constant

and equal

to the

sum of

asymp

the squares on the

semi-axes.

From

134.

(1) of the last article

sm

^

ing through by

we

have, after multiply

,

OT-^LfYl v sin 20 =

n- J_ A 2 "f-^-

sin 2

=

2

(a

+b

2

)

sin

cos

0.

But, Art. 131,

=

sin

-J 2

;

2

sin 2

=

arm OTT

i.e.,

(t

Va + b

2

OT or

hence

.-.

6

=

cos

,

Va +

= a&

is

equivalent

to

;

arm OAD B.

Ae triangle formed by a tangent

asymptotes

2

to

the hyperbola

Draw the chord RB/, Fig. 54, parallel Draw also the diameter OL through D. Since TD = T D, we have B/L = BL.

135.

T

and

its

the rectangle on the semi-axes. to the tangent

T.

Since

i.e.,

OL

is

LK = LH LK = BL LH

a diameter,

we have

BL B K = BH

;

hence

;

;

hence, the intercepts of a chord between the hyperbola asymptotes are equal.

and

its

PLANE ANALYTIC GEOMETRY.

166

EXAMPLES.

What

1.

are the equations of the asymptotes of the hyper~2

bola

,.2

f-gAns.

What

y

=

| x.

-J-

are the equations of the asymptotes of the follow

ing hyperbolas

:

_. 3.

3 ^2

_

2 ^2

Ans.

=_ y = -^ Vf x.

mx

5

2

_

Uy 2

=

c

6. What do the equations given in the four preceding ex amples become when the hyperbolas which they represent are referred to their asymptotes as axes ?

=

The semi-conjugate axis of the hyperbola xy what is the value of the semi-transverse axis ?

7.

6

;

25

Ans.

What

is

8.

are the equations of the tangents to the following

hyperbolas

:

8.

To xy

=

9.

To xy

=+

10. at (1, 10).

Ans.

Ans. 10.

To xy

11.

To xy

m, at

= - p,

1,

(

at

(

\

+ 10 x = 20. = 3 x 12.

y

12, at (2, 6).

y

-\-

m). 2, /

Required the point of the hyperbola xy 4. the sub-tangent 12.

=

12 for which

=

Ans.

(4, 3).

an hyperbola

13. The equations of the asymptotes of whose transverse axis = 16 are 3 y = 2 x and 3 y

+

2x

=

;

required the equation of the hyperbola. *

"

64

256

THE HYPERBOLA.

167

14. Prove that the product of the perpendiculars from any point of the hyperbola on the asymptotes stant and

let fall is

con

GENERAL EXAMPLES. 1.

10

;

The point

(6, 4) is on the hyperbola whose transverse required the equation of the hyperbola.

is

-

Am. 25

400

Assume

the equation of the hyperbola, and show that the difference of the distances of any point on it from the foci is constant and 2 a. 2.

=

Required the equation of the hyperbola, transverse axis 2 x and 3 y 6, which has 5 y 13 x for the equa tions of a pair of conjugate diameters. 3.

=

=

=

2

^_^/_ = 1

An*.

9 78 that the ratio of the sum of the focal radii of any point on the hyperbola to the abscissa of the point is con stant and 2 e. 4.

Show

=

5.

fulfil

What

are the conditions that the line

y

= sx

-f c

must

a

= 0.

in order to touch

-^ a 2

-

-2 b

=

1 at infinity J ?

Ans.

s

=

_L_

,

c

6. Show that the conjugate diameters of an hyperbola are also the conjugate diameters of the conjugate hyperbola. 7. Show that the portions of the chord of an hyperbola

included between the hyperbola and its conjugate are equal. 8. What is the equation of the line which passes through the focus of an hyperbola and the focus of its conjugate hyperbola ?

Ans.

x

PLANE ANALYTIC GEOMETRY.

168

Show

9.

that

^

=

e

when

e

and

e

b

_ a

are the eccentricities of

two conjugate hyper

bolas.

Find the angle between any pair of conjugate diame

10.

ters of the hyperbola.

Show that in the hyperbola the curve can be cut by of two conjugate diameters. one only 11.

Find whether the line y = f x intersects the hyperbola 9 x2 = 144, or its conjugate.

12.

16 y~

Show that the conjugate diameters of the hyperbola make equal angles with the asymptotes. 13.

Show

14.

eral

drawn from any point

that lines

equilateral

of the equilat

hyperbola to the extremities of a diameter

make equal

angles with the asymptotes.

In the equilateral hyperbola focal chords drawn parallel to conjugate diameters are equal. 15.

A

from the focus of an hyper perpendicular is drawn show the bola to asymptote that the foot of the perpendicular is at the distance a 16.

:

(a)

from the centre, and (b)

that the foot of the perpendicular

from the 17.

is

at the distance b

focus.

For what point of an hyperbola

is

the sub-tangent

=

the sub-normal ? that in the equilateral hyperbola the length of the normal is equal to the distance of the point of contact 18.

Show

from the 19.

centre.

Show

chord of

that the tangents drawn at the extremities of any diameter which hyperbola intersect on the

the

bisects the chord.

THE HYPERBOLA. 20.

Find the equation of the chord of the hyperbola J?L -

J^!

is

=1

12

9

which

169

bisected at the point

(4, 2).

Required the equations of the tangents to

21.

J*L.

I*-

16

10

=

which make angles of 60

1

with the X-axis.

that the rectangle of the distances intercepted on drawn at the vertices of an hyperbola by a drawn at any point is constant and equal to the

Show

22.

the tangents

tangent

square of the semi-conjugate axis.

Given the base of a triangle and the difference of the

23.

tangents of the base angles

Show

24.

;

required the locus of the vertex.

that the polars of

(m, n) with respect to

the

hyperbolas

^1 2

_ J/l =

a

If

25.

1,

b*

ll

- ^1 =

V2

a2

1 are parallel.

from the foot of the ordinate of a point

hyperbola a tangent be drawn to the

circle

(x,

y) of the

constructed on

the transverse axis, and from the point of tangency a line be drawn to the centre, the angle which this line forms with the Show transverse axis is called the eccentric angle of (x, y). values deduce from these and b tan sec that go, go), y) (a

=

(x,

the equation of the hyperbola. 26.

If

r

(x

,

y

),

(x",

y")

are the extremities of a pair of

conjugate diameters whose eccentric angles are that

<p

+

<p

=

90.

go

and

cp,

show

PLANE ANALYTIC GEOMETRY.

170

CHAPTER

IX.

THE GENERAL EQUATION OF THE SECOND DEGREE. 136. The most general equation of the second degree be tween two variables is 2 2 (1) ay + bxij + ex + dy + ex

+f=

.

.

.

whatever. d, e, /are any constant quantities this which loci the of the To investigate equation properties of the constants as to represents under all possible values

in

which

a, b,

c,

sign and magnitude

is

the object of this chapter.

The equations of the lines in have had to do in preceding chapters, 137.

By + C = 0. 2 = (Ax + B# + C) _ _ Two o. yS. 2/2

Ax

-f

_j_

x2

7/2 _|_

xt

7/2

2 7/

=a _ o.

2

= 2px.

.

0.

a plane, with which are

we

Straight line. Two coincident straight lines.

straight lines.

Circle.

Two

imaginary straight

lines.

Parabola.

= a 2^ Ellipse. _ = crb tfxay Hyperbola. = a 2j^ Hyperbola, a 2^2 _

0,2^2 _|_

2^,2

2

.

#83.2

we Comparing these equations with the general equation, them may be deduced from it by making the constants fulfil certain conditions as to sign and magnitude. the lines which We therefore, prepared to expect that see that all of

are,

these equations represent will appear among the loci repre sented by the general equation of the second degree between

two

variables.

find that these

equation.

In the discussion which lines

is

to ensue

are the only loci represented

we

by

shall

this

EQUATION OF THE SECOND DEGREE.

171

DISCUSSION. 138. To show that the

locus represented by

a complete equa

tion of the second degree between two variables is also represented by an equation of the second degree between two variables, in which the term containing xy is wanting.

Let us assume the equation ex* +dy ex aif bxy (1) and refer the locus it represents to rectangular axes, making the angle with the old axes, the origin remaining the same. Prom Art. 33, Cor. 2, we have

+

+

+f=

+

x

=x

cos 6

y

x

sin 6

in

+bx

cos

Substituting these values

+ V + d y + e x +/= 2

C

y>

.

y sin

+y

for the equations of transformation. in (1), we have,

a y*

.

.

.

.

.

(2)

which

= a cos = 2 (a c = a sin d! = d cos e = d sin a!

V

+ c sin

2

2

6 sin B cos 2 -\- c cos e sin -J- e cos

c)

2

cos

b sin

}

+ b (cos 6 + 6 sin cos 2

sin 2 0) I

.

.

.

(3)

0, the angle through which the axes have been turned, entirely arbitrary, we are at liberty to give it such a value as will render the value of b equal to zero. Supposing it to

Since

is

have that value, we have or

2 (a

c)

(a

sin 2

c)

or

cos 6

sin

tan

20

=

2

-

2

(cos

=

.

^ .

c

Since any real

+b

-f b cos

a

number between

-f-

=

sin 2 0)

.

.

0,

(4)

(5) GO

and

o>

is

the tan

gent of some angle, equation (5) will always give real value for 20; hence the above transformation is always possible.

Making

b

=

in (2),

a lf for the

we

have, dropping accents,

+ eV + d y + e x +f =

equation of the locus represented by equation, then, we shall confine our attention.

.

.

(1).

.

(6)

To

this

PLANE ANALYTIC GEOMETRY.

172

To find the value of a and c in terms of a, b, Adding and then subtracting the first and third of

COR.

and

1.

c.

the equations in

we have

(3),

+a =c+a (7) = (c - a) cos 2 + b sin

c

-

c

Squaring

a

(4)

and

...

c

.

...

2

-

a

=

V(c

-

(7)

+ b*

8

a)

and

.

.

.

(8)

we have

adding to the square of (8),

Subtracting and then adding

a

.

.

(9)

we have

(9),

= i\ G + a- V(c - ay +Tj = $ \c + a + V(c-a) + ^

.

.

(10)

.

2

c

COB.

and

To

2.

find the signs of a!

and

c

y= ...

Hence, the 2

a

c

i |( C

+

=-

i

of a

si#?is

(V

and

-(( -- 4 ac)

a

2

= 0,

.-.

2

c

.

) .

-

(12)

depend upon the sign of the

ac.

<

b

+ * )h

2 <0

4 quantity b The following cases present themselves 2 4 ac. The sign of the second 6 1.

2.

Multiplying (10)

we have

(11),

positive,

(11)

.

a and

c

= 4 ac. The or c = 0.

:

member of (12) are both positive, or both negative. second

member

of (12)

becomes

zero,

is

.-.

cannot be equal to zero would reduce (6) to at the same time, for such a supposition an equation of the first degree.] 2 4 ac. The sign of the second member of (12) is 3. b and c negative, or a must be a must be be observed that [It will

a and

c

>

negative,

positive

.-.

negative and

c positive. 7

To transform the equation a if + c x -f a ?/ + e a -f the vari f=Qinto an equation in which the first powers of 2

139.

ables are missing.

EQUATION OF THE SECOND DEGREE.

173

Let us refer the locus to a parallel system of rectangular From Art. 32, we axes, the origin being at the point (m, n). have a y = n x = m y Substituting these values in the given equation, we have 2 * = c x + +f" (2)

+

y +

in

+

f

+

,

.

e"x

.

.

.

d"ij

which d"

c"

/"

Since

m

=2an +d = 2 c w -f e = a + cm + ?i

2

+

rf w-

e

??t

+/

n

j

make

as to

+d =

and 2

c

w

-f e

=

m=

and

= .

2 a see

3)

;

we may make

in general,

We

(

f

and w are entirely arbitrary, we may, in general, 2 a n

i.e.,

*

2

them such values

give

]

2

.

f

(4)

c

from these values that when a! and c are not zero, is possible and equation (2) becomes,

this transformation also

;

after dropping accents,

a if +cV+/"

Equation

;

...

(5)

we

observe, contains only the second power of hence it is satisfied for the points (x, y) and

(5),

the variables

=

But only the equation of curves with centres can satisfy this condition hence, equation (5) is the equa tion of central loci. When either a f or is zero, then n or in (

y).

a?,

;

</

and the transformation becomes impossible. two cases which require special consideration.

is infinite

arise

CASE 1. a! == o. Under this supposition equation

Hence

140.

(6),

Art. 138,

cV-MV + ^+/=0

...

becomes

(1)

Referring the locus of this equation to parallel axes, the origin being changed, we have for the equations of trans formation x m x y n -f- y

=

+

,

=

.

PLANE ANALYTIC GEOMETRY.

174

Substituting in C

V + dy + (2 2

Now, make i.e.,

we may

in general,

2

c

m+

we have

(1),

m + e ) x + c m* + d n + e m

c

= 0,

e

c

+f=Q

.

.

.

(2)

and w such values as to

m + d w + e m +/= 2

;

we may make

m= m If d mation

is is

-|- e^/^i, -f-

/_

e

x

7

2

4/c 4

V

rZ

c

=

not zero (since a! 0, c is not zero), this transfor possible and (2) becomes, after dropping accents,

x2

c

Or 2

COR.

and

f ,

g

^

If

d

=

-\-

= 0,

dy

jr a

_

,

-^^...

0, (1)

c x* or,

and

m

give

/Q\ (3)

becomes

+ e x+f=0

solving with respect to

...

(4)

x,

CASE 2. c = o. Under this supposition equation

141.

(6),

Art. 138, becomes

Transforming this equation so as to eliminate ?/ and the constant term, by a method exactly similar to that of the preceding

article,

we

find r

n =

d

J* d*

nyi

- 4 a!f J_

A

i

r

4fcV and,

if e

is

not zero,

we have

f=--,*

(of is

not zero since

(2)

c

= 0)

EQUATION OF THE SECOND DEGREE. COB. If

142.

e

= 0,

equation

Summarizing the

175

becomes

(1)

results of the preceding articles,

we

find that the discussion of the general equation

aif

+

bxy

+ ex + dy + ex +/ = 2

has been reduced to the discussion of the three simple forms 1.

a! if

+ cV +

x2

/"

= -

.

c

\

=

--

y

.

x.

f

= 0.

:

Art. 139, (5)

Art. 140, (3)

Art. 141, (2)

^-.

Art. 140, (5)

3.

^-.

Art. 141, (3)

The discussion now involves merely a consideration sign and magnitude of the constants

of the

which enter into these

equations.

143.

tf

<

4

ac.

Under this supposition, since of and d are both positive or both negative, Art. 138, Cor. 2, neither a! nor c can be zero hence, forms 2 and 3 of the preceding article are excluded ;

from consideration. The first form becomes either

ay

or

which

c a

f

of and c may have any real value and and any sign any value. Hence arise four cases

in

f"

:

may have

PLANE ANALYTIC GEOMETRY.

176

CASE

1.

If

has a sign different from that of a and c whose semi-axes are ,

f"

equations (1) are equations of ellipses

GASP:

2.

If

f"

has the same sign as that of a and

c

,

equa

curves.

tions (1) represent imaginary has a different sign from that of c and CASE 3. If a

=

f"

has If circles. equations (1) are equations of the same sign as of and c then the equations represent imagi

a and

e

/"

,

,

nary

curves.

CASE nary

4.

If/"

= 0, equations (1)

are equations of taw imagi

the origin. straight lines passing through

Hence, when

b

z <

4

ac, every

equation of the second degree

an imaginary curve, between two variables represents an ellipse, at the a circle, or two imaginary straight lines intersecting origin. b

Under

=

;

2

= 4 ac.

either a this supposition, Art. 138, Cor. 2, form (1) of Art. 142 is excluded.

- 0,

or

hence,

Resuming the forms

(3)

y

we have

=

~~

(>

Z

a

four cases depending upon the sign and magnitude

of the constants. are not zero, CASE 1. If d and c in the first form of (2) not are of zero, then form second the in (2) and if e and a of parabolas. equations (2) are equations

EQUATION OF THE SECOND DEGREE.

177

Since the first form of (3) is independent of y, it lines parallel to each other and to the Y-axis. two represents The second form of (3) represents, similarly, two lines which

CASE

2.

are parallel to the X-axis.

CASE

3.

If

2 e" <

4/b the

form of

first

(3)

represents two

lines.

imaginary two imagi If d 2 4/o/, the second form of (3) represents lines. nary CASE 4. If e 2 = 4/c the first form of (3) represents one <

,

straight line parallel to the Y-axis. 4 fa the second form of (3) represents one straight If d 2 line parallel to the X-axis.

=

,

=

2 4 ac, every equation of the second degree Hence, when b between two variables represents a parabola, two parallel straight lines, two imaginary lines, or one straight line.

2

145.

l>

>

4

ac.

Under this supposition, Art. 138, Cor. 2, since a! and f must have opposite signs, neither a nor c can be zero hence forms (2) and (3) of Art. 142 are excluded from con The first form becomes either sideration under this head. c

;

ay

-<fc

-y +

or

We

+/"

cV+/"

1.

If

=OJ

f has a different sign from that of a

(1) are equations of hyperbolas

/r

0=v If

1

have here three cases.

CASE

still

-

,

equations

whose semi-axes are

andJ=v/r,

f has a different sign from that of

c

,

equations (1) are

equations of hyperbolas.

CASE

2.

If a

eral hyperbolas. CASE 3. If f"

=

c

,

= 0,

secting straight lines.

equations (1) are equations of equilat

equations (1) are equations of two inter

PLANE ANALYTIC GEOMETRY.

178

2

4 ac, every equation of the second degree between two variables represents an hyperbola, an equilateral hyperbola, or two intersecting straight lines.

Hence, ivhen

b

>

The preceding discussion has

SUMMARY.

146.

elicited the

following facts 1. That the general equation of the second degree between two variables represents, under every conceivable value of the :

constants which enter into bola, or one 2. 3.

it,

of their limiting

When When

b

2

b

2

When

b

2

4&c

<

=

it

4ac

an

ellipse,

an

represents it

a parabola, an hyper

cases. ellipse, or

a limiting

case.

represents a parabola, or a limiting

case. 4.

4&c

>

if.

represents

an hyperbola,

or

a limiting

case.

EXAMPLES. 1.

Given the equation 3y 2

-\-

2 xy

+3x

2

Sx

Sy

=

;

to classify the locus, transform and construct the equation. Write the general equation and just below (a) To classify. it the given equation, thus :

+ bxy + ex + dy -f ex +/ = 8x = 8y 3 if + 2 xy + 3 x ay

2

2

2

Substituting #>

_ 4 aCj

the

co-efficients

we have

4 ac

tf

the

in

=4-

2 ac. b hence and the locus belongs to the ellipse

36

class

= -

.

.

.

(1)

characteristic

32

;

<4

(b)

To

class, Art. 146.

refer the locus to axes such that the term containing

xy shall disappear.

From

Art. 138, (5),

we have

=

tan 2

hence

tan

20

=

-

o

o .-.

2(9

= 90

.-.

6>

=+ = 45

oc,

.

.

.

(2)

EQUATION^OF THE SECOND DEGREE. new X-axis makes an angle of X-axis. Taking now (10), (11), (3), Art. ing values, we have i.e.,

the

of

179

45 with the old

-j-

138,

and substitut

= i \c + a - V(c - a)- + = 2. = i \c + a + V(c- a) + J=4. = dcosO e sin = V2 (d e) = = d sin + e cos = V^ (^ + e = l>*\

f

c

d e

2

these

being zero), 2 y2

8 V2.

)

in

values

0.

J-

(9

Substituting

2

Art.

(6),

+ 4 x - 8 ViT x =0 2

138,

...

we have (/

(3)

To re/er ^^^ ^oci^s to its centre and axes. (c) Substituting the values found above in (4), Art. 139,

n

have

d

=

=

U.

2c

= an + cm 2

Hence/"

(3).

2

8

-f

cf

found above in 2 if

f"

8,

Art. 139,

+4x

2

for the reduced equation.

= V2 (d)

Draw X-axis.

curve

and

To

b

=

together with the values of a

we have

(5), Art. 139,

8

+ %- = 1

or

a

+ em +/=

dn

.

Substituting this value of

and

we

=

...

0,

(4)

The semi-axes

of the ellipse are

2.

construct.

the axis

OX

See

Draw OY

when

(b).

,

making an angle of 45 with the old The equation of the -Lto OX

referred to these axes

.

is

given in

(3).

Constructing

PLANE ANALYTIC GEOMETRY.

180 the point (c).

(V2,

Draw O

when

Y"

referred to

i-

See 0) we have the centre of the ellipse. The equation of the curve to OX" at .

O

Y",

X

O

as axes is given in (4).

FIG.

55.

V2 and 2, we can construct the by either of the methods given in Art. 78.

Having the semi-axes, ellipse

DISCUSSION. If

in (1),

y

x If

x

If

x

to the If

y

= 0,

x

for the X-intercepts 0,

=-

=

in (1),

we have

=

in (3),

we have y

OD,

.

for the Y-intercepts 0,

=

_|_0;

i.e.,

OC,

the ellipse

is

tangent

Y -axis.

=

in (3),

x If

we have

x = o in

(4),

we have

=

0,

x

we have

y=

=1=2.

for the

X -intercepts

0, OB,

= 2 V2. for the

Y"-intercepts

O

A,

OA

.

EQUATION OF THE SECOND DEGREE. If

y

=

in (4),

we have

=

x

X -intercepts O B, O O,

for the

V2.

-j-

Given the equation ?/ 2 2 xy -f # 2 2y 1 and construct the equation. To classify.

2.

181

=

0, class

ify the locus, transform (a)

+

aif

y hence

b

2

2

2

bxy

+ ex

ajy 4-

4 ac

a?

2

dy

-f

2

2y

=4

+/ =

-f ex 1

=

.

.

.

= 0,

4

(1)

hence the locus belongs to the parabola class, Art. 146. To refer the locus to axes such that the term (b) containing

xy shall disappear.

From

Art. 138, (5),

we have b

20=

tan

a

c

;

hence, substituting

tan 2

=

?

-1

1

= -45

...

...

5

(2)

Substituting the values of the coefficients in (10) of Art. 138,

(11)

(3)

we have

= \ c + a + V(e a) = 2. b*\ = d cos = 2 ( V2) = V2. e sin e = d sin + e cos = - 2 (- 1 V2) = + V2. Substituting these values in (1), Art. 140 (since a = 0), we have 2 x - V2 y + V2^ - 1 = (3) 2

c

-f-

\

d!

2

.

(c)

To

refer the

parabola

to

.

.

a tangent at the vertex and the

axis.

Substituting the values of the constants in (a), Art. 140, r

we have

m=-

n

=

2

e

^-7

~

= - V2 = - -35

e 4fc = ~~T^~~ 4 d c

5

T~7^ 4 V2

=-

-

nearly.

90 nearly. J

PLANE ANALYTIC GEOMETRY.

182

is

Substituting the values of d and not zero), we have

= iV2.y

*

.

.

.

c>

in (3), Art. 140 (since

d

1

(4)

for the reduced equation. (d) To construct.

V

Draw

OX

making an angle

OY 1 to OX

of -- 45

with the X-axis

;

See (). The equation of the parabola when referred to these axes is given in (3). vertex .35, Constructing the point ( .90), we have the

draw

.

and O of the parabola See (c). Draw O parallel to the axes OY respectively. The equation of the parab The curve can now ola referred to these axes is given in (4). be constructed by either of the methods given in Art. 54. .

OX

,

X"

Y"

EQUATION OF THE SECOND DEGREE.

183

DISCUSSION. If

If

If

x

=

y

=

x

=

y

we have

in (3),

we have

_

x

==

OD

,

Y -intercept OK,

.707.

we have

for the

- V2 + yio

Jj

.

X -intercepts OL, OL - V2 - Vio Z=

4 If

OD,

,

V2

in (3),

*/

for the

~=-

=

00

.4.

for the Y-intercept

-l.

=

=-

y

in (1),

00,

for the Y-intercepts

y = 2.4

y If

we have

in (1),

in (4),

4

=

?/

if

;

=

y

x

in (4),

=

_j_

2

-\-

bxy

+

ex 2

-f-

dy

+

2y + 6x

ex

3

-\-

=

f= .

.

0.

= 0, classify

Given the equation ?/ -^ 2 cc 2 2 ?/ + 6 x 3 the locus, transform and construct the equation. (a) To classify. 3.

ay

,

0. .

(1)

hence, the locus belongs to the hyperbola class, Art. 146. (b) To ascertain the direction of the rectangular axes

(xy

being wanting). tan 6

.-.

i.e.,

the

new X-axis

26

=

= 0; is

c-

= -5_ = Q -3

:

parallel to the old X-axis.

To refer the hyperbola

(c)

a

to

its

centre

and

axes,

we

have,

Art. 139, (4),

hence

n

=

1,

m=

-. t

Substituting in the value of

/ = hence

a n2

+cm + 2

f"

dn

=1

+em

/",

Art. 139, (3),

+/ = 1

2

we have

+9

3

;

PLANE ANALYTIC GEOMETRY.

184

This value, together with the values of a? and 2 if 4 x* = - 1 139, gives (3) for the required equation.

-

To

(d)

.

in (5), Art.

c

.

construct.

FIG.

Construct the point

OX, and O

Y" ||

(%, 1),

is

and through

The equation

OY.

to

referred to these axes

57.

given in

We

(3).

tion that the semi-transverse axis

is

-.

u

it

draw O

of

the

X" ||

hyperbola

see from this equa

Laying

tance to the right and then to the left of vertices of the curve A, A

,

off this dis-

we

locate the

.

DISCUSSION. If

x

=

y If

y

we have

in (1),

= o in

(1),

x

= 3, y = we have

=

3

for the Y-intercepts

-

OC,

00

OD,

OD

,

1.

for the X-intercepts

+ V3

to

3- V3

,

EQUATION OF THE SECOND DEGREE.

=

x

If

in (3),

=

y

we have

= iVH^

2/

If

185

in (3),

we have

for the X-intercepts

O

A,

OA

,

.-4 From

this data

the student

readily determine the focal distances of the

may

and the

eccentricity, the parameter,

hyperbola.

= 0,

+

Given the equation y 2 -\- x z 4 ?/ 4# 1 the ify locus, transform and construct the equation. 4.

b2

(a)

<4

=

(b)

ac .-.

the locus belongs to the ellipse class.

.-.

new X-axis

(m,w)=(-2,

(c)

x2

hence

class

-f-

is

to old X-axis.

||

2)and/"

= -9

=9 y-

the transformed equation of the locus, which from the form of the equation is evidently a circle. is

Locate the point

(d)

and with 3 as a

(2,

With

2).

this point as a centre, it will be the re ;

radius, describe a circle

quired locus. 5.

y

2

2 xy

+x

b = 4 ac = 45 2

(a) (b)

.-.

2

new X-axis

We

to the old X-axis.

...

= 0.

parabola class.

.-.

a!

2

= 0, 2x

i.e.,

2

x

c = 2, d = 0, - 2^0; = 1 and x =

are the equations of the locus (c)

inclined at an angle of

45

have also

when

The construction gives the

e

=

1

...

(1)

new

referred to the

lines

OX OY ,

axes.

as the

new

axes of reference.

Equations

drawn

||

(1) are

to the

the equations of the two lines and at a unit s distance from

Y -axis

CM, C it.

M

PLANE ANALYTIC GEOMETRY.

186

FIG.

58.

We may

construct the locus of the given equation without going through the various steps required by the general method. Factoring the given equation, we have (y

hence

-

x

+ V2)

=x

y

V2

(y

-x- V2) =

and y

=x

-f-

;

V2

the equations of the locus. Constructing these lines (OY, being the axes of reference), we get the two

are

OX

CM, C

parallel lines

M

Classify, transform,

equations 6.

2

?/

.

and construct each of the following

:

2 xy

+x +2y

2

+

2

2 x -f 1

= 0. y

7.

8.

if

+

ajy

2

^

x2

- 1 = 0.

4. 5

a-s

_

12 x

- 12 y = 0.

=x

1.

EQUATION OF THE SECOND DEGREE. 9.

10.

2?/

2

4x

+ 2x -4?/2

+1

=0. x2

7/2 _j_ jpa

_ 2 + 1 = 0.

+

x2

+ 2*+2 = 0.

-

2

187

+

2 7/

=

f

.

aj

(1, 0).

11.

2

2/

Imaginary 12.

y

2

ellipse.

+ x - 8 x + 16 = 0. 2

XT/

Parabola. 13.

2 xy -f x

if

2

y

+2x

1

= 0. Parabola.

14.

4 ay

15.

?/

-

2x

+

2

=

0.

Hyperbola.

16.

2

2 or

+ 2 + 1 = 0. ?/

Two

intersecting lines.

- x + 2 y + 2 x - 4 = 0. 2

if

Equilateral hyperbola. 17. 18. 19.

20. 21.

-

+ x + 2 y + 1 = 0. / + 4 xy + 4 - 4 = 0. 2 XT/ + 2 x 2 y + 2 x = 0. 4 + 4 x = 0. if x + 2 = 0. 2 jcy 2

?/

2

2

jcy

2

ic

2

2

2/

2

a;//

2

?/

2

PLANE ANALYTIC GEOMETRY.

188

CHAPTER

X.

HIGHER PLANE CURVES. 147. Loci lying in a single plain and represented by equa tions other than those of the first and second degrees are

We

HIGHER PLANE CURVES. shall confine our atten tion in this chapter to the consideration of a few of those curves which have become celebrated by reason of the labor called

expended upon them by the ancient mathematicians, or which have become important by reason of their practical value in the arts and sciences.

EQUATIONS OF THE THIRD DEGREE. 148.

THE SEMI-CUBIC PARABOLA.

This curve ordinate

OP

TT

let fall

is

the locus generated by the intersection of the common parabola with the perpendicular

of the

from

its

vertex upon the tangent drawn at

T

as

the point of tangency moves around the curve. 1.

To deduce the rectangular equation.

T be the point of tangency, and let P (x y ) be a point of the curve. Let y 2 == px be the equation of the common parabola. to the parabola Since the equation of the tangent line T Let

(x" ,

,

y")

M

is

Art. 57,

(6),

the equation of the perpendicular vertex is

{r

(OM)

let

fall.

from the

HIGHER PLANE CURVES.

189

FIG

Since

TT

is

*=*",

Combining

OY, we have

parallel to

(1)

and (2)>

.

.

for its equation

(2)

we have

But

_ V4j^_

hence

2p Squaring and dropping accents, we have

for the equation of the semi-cubic parabola. This curve is remarkable as being the first curve rectified,

that

is,

the length of a portion of

it

which was was shown to

PLANE ANALYTIC GEOMETRY.

190

be equal to a certain number of rectilinear units. It derives its name from the fact that its equation (3) may be written

xl

= p% y

,

To deduce the polar equation. r Making x = r cos 6 and y

2.

reduction, r

=p

tan

2

sec

sin 9 in (3),

.

.

.

we

have, after

(4)

for the polar equation of the curve.

SCHOL.

Solving

(3)

with respect to

y,

we have

An

inspection of this value shows (a) That the curve is symmetrical with respect to the

X-axis; (b)

That the curve extends

infinitely

from the Y-axis in

the direction of the positive abscissas.

To duplicate the cube by the aid of the parabola. Let a be the edge of the given cube. We wish to con struct the edge of a cube such that the cube constructed on it shall be double the volume of the given cube i.e., that the 3 8 = satisfied. shall be a 2 x condition 149.

;

FIG.

60.

HIGHER PLANE CURVES.

191

Construct the parabola whose equation is 2 2 ax i/ (1) be the curve. Construct also the parabola whose Let

=

.

.

.

MPO

equation

is

x*~y... NPO be this Then OA (= x) Let

}

is

the abscissa of their point of intersection

the required edge.

we have x8

(2)

curve.

For eliminating y between

(1)

and

(2),

= 2 a*.

This problem attained to great celebrity among the ancient We shall point out as we proceed one of the geometricians.

methods employed by them in solving 150.

The

THE

it.

CISSOID.

by the intersection (P) of (OMM T) with the ordinate DN

cissoid is the locus generated

the chord

(OM )

of the circle

s FIG.

61.

PLANE ANALYTIC GEOMETRY.

192

MN (equal to the M on the diameter

MN

ordinate

let fall

from the point

as the chord revolves about

through 0)

the origin 0. It may also be defined as the locus generated by the inter z 8 ax with the section of a tangent to the parabola y let fall on it from the origin as the point of perpendicular moves around the curve.

=

tangency

To deduce the rectangular equation. Let OT = 2 a, and let P (x, y) be any point From the method of generation in this case of the curve. 1.

First Method.

ON = N T. MN = M ON M we have lST

From

.-.

the similar triangles

ONP,

,

NP ON M N ON NP = y, ON = M N = VON N T = ON = 2 a - x :

:

But

.

:

:

.

oj,

V (2 -),

;

.

Hence is

.y x :

y.

=

::

V(2 a

^

x)

...

x 2 a :

x.

(1)

the required equation.

The equation

Second Method. parabola if

=

8 ax

y

=

is

of the tangent line to the

Art. 65, (2)

+ .

SX

2a s

of a line passing through the origin line is this to pendicular

The equation

Combining these equations

so as to eliminate

s.

and per

we have

for the equation of the locus.

This curve was invented by Diocles, a Greek mathematician of the second century, B.C.,

and called by him the cissoid from

HIGHER PLANE CURVES. word meaning

a Greek

It

"ivy."

193

was employed by him

in

solving the celebrated problem of inserting two mean propor tionals between given extremes, of which the duplication of the cube

is

PO^) = 0) = OP = M K = OK - OM/. OK = 2 a sec and OM = 2 a cos r = 2 a (sec cos 0), r = 2 a tan 6 sin

From

the figure (OP,

we have But

(r,

also r

hence

;

or is

a particular case.

To deduce the polar equation.

2.

the polar equation of the curve.

SCHOL. Solving

y

(1)

with respect to

y,

we have

= x

2 a

An

inspection of this value shows (a) That the cissoid is symmetrical with respect to the X-axis.

That x = and x = 2 a are the equations of its limits. That x = 2 a is the equation of a rectilinear asymp

(b) (c)

tote (SS

).

151. To duplicate the cube, by the aid of the cissoid. Let OL, Fig. 61, be the edge of the cube which we wish to Construct the arc BO of the cissoid, CO = a duplicate.

CD = 2 CA = B draw BO LR intersecting BO in R.

being the radius of the base circle. Lay off 2 a and draw DT intersecting the cissoid in

and

at

Then

L

LR

erect the perpendicular is the edge of the required cube

;

;

for the equation

of the cissoid gives

hence

HB = 2 a

The

HB = y, OH =

2

ig PL _L

(since

- x).

CDT and HBT CD CT HB HT.

similar triangles :

::

:

give

x,

and

HT =

PLANE ANALYTIC GEOMETRY.

194 But

CD =

2

CT by .-.

construction

;

hence

HT in the value of HB above gives HB = 2QH3 hence HB = 2 OH HB triangles OHB and OLE are similar hence HB OH LE OL HB OH ::LK ::OL HB = 2 OH hence LB, = 2 OL whence the 2

This value of

2

8

3

.

;

The

;

::

:

3

:

8

8

.-.

But

8

3

:

3

3

,

8

;

struction.

152.

HB = 2 HT

HT = HB

THE WITCH.

FIG.

62.

con

HIGHER PLANE CURVES. The witch nate

the locus of a point

is

P

011

DP

195

the produced ordi-

of a circle, so that the produced ordinate the diameter of the circle as the ordinate

DM

OA

DA

outer segment It

may

to

is

to the

is

of the diameter. of a point P its foot

also be defined as the locus

linear sine

DP

DM of an angle

from

at a distance

011

D

the

equal

to twice the linear tangent of one-half the angle.

To deduce the rectangular equation.

1.

From

First Method.

DP =

y,

we have

x

(2

a

- x),

;

y:2a:: V(2 a

x)

x 2a :

x.

the required equation.

is

Second Method.

Let

But a (1

(1

+ cos

hence

or,

MCO =

;

then by definition

= 2 a \ //a ^ ~ cos ^ V a (1 + cos 0) - cos 0) = a - a cos = OC DC = OD == x, and = a + a cos = OC + DC = OD = 2 a - x 0) u = 2a == 2

?/

a

of generation,

:

.

DA = 2 a hence

mode

::

:

But

the

DP OA DM DA OA = 2 a, DM = VOD DA = Vx

a tan 2

,

;

squaring

y

2

= ~2 a

.

x

This curve was invented by Donna Maria Agnesi, an Italian mathematician of the eighteenth century. SCHOL. Solving (1) with respect to y, we have

y

Hence

(a)

=

=t 2 a

the witch

is

x J-~ V 2a

.

x

symmetrical with respect to the

X-axis. (b) (6-)

x x

= and x = 2 a are the equations of its limits. = 2 a is the equation of the rectilinear asymptote

SS

.

PLANE ANALYTIC GEOMETRY.

196

EQUATIONS OF THE FOURTH DEGREE. THE CONCHOID.

153.

The conchoid is the locus generated by the intersection of a circle with a secant line passing through its centre and a fixed point A as the centre of the circle moves along a fixed

OX. As the

line

intersection of the circle and secant will give two points P, P, one above and the other below the fixed line, it is evident that during the motion of the circle these points will generate a curve with two branches. is called the SUPERIOR BRANCH

MBM

;

FERIOR BRANCH.

(= OB)

is

called the

the DIRECTRIX

1.

;

The upper branch the lower, the IN

The radius of the moving circle O P MODULUS. The fixed line OX is called

the point A, the POLE.

To deduce the rectangular equation.

Let

P

(x, y),

the intersection of the circle

PP P

and the

HIGHER PLANE CURVES. secant

and

let

AO P, be any OA = a.

point of the curve.

The equation

of the circle

The equation

of the line

y

Making y

=

= sx

a

.

.

.

OP

Let

whose centre

AO P

197 b,

f

at

is

= OB = (x

,

0) is

is

(1)

we have

in (1),

s

00

for the distance

But

00 =

x

;

.

hence a

-

^

= b-

.

.

.

(2)

the equation of the circle. If we now combine (1) and (2) the s, resulting equation will express the relationship between the co-ordinates of the locus generated by the intersection of the loci they represent. Substituting the value of s drawn from in we have is

so as to eliminate

(1)

(2),

*V-(P-tf) is

We

might have deduced

triangles

Draw AT ATP and O SP

||

PS:SO e

+

2

2/)

.

.

.

(3)

the required equation.

simple way:

-

<>

this equation in the following very to OY. OX, and Since the

PT

to

are similar,

::PT:

TA

||

we have

;

-

Hence _

This curve was invented by Nicomedes, a Greek mathema who flourished in the second century of our era.

tician

was employed by him in solving the problems of the duplication of a cube and the trisection of an angle. It

PLANE ANALYTIC GEOMETRY.

198 2.

To deduce

From

the polar equation.

the figure

we have (AY being the

the pole)

(AP,

PAB)

But

AP =

hence

r

is

ACT

=

= a sec

and

A

0)

(>-,

OP

-t

initial line,

;

b

the polar equation of the curve. SCHOL. Solving (3) with respect to

x,

we have

An

inspection of this value shows is symmetrical with respect to the (a) That the conchoid

Y-axis. (b) (c)

That y That y

= b and y = = gives x =

b are the -J-

tote.

If

(d)

comes a (e)

a

then

cc

=

-t

equations of its limits. the X-axis is an asymp

_

V&

.

2

a

2/

;

i.e.,

the conchoid be

circle.

If b

(/)

0,

oo,

.

>

If b

a,

=

a,

the inferior branch has a loop as in the figure. coincide and the loop and the points

A

A

disappears. (g)

If b

<

a,

the inferior branch

A

is

similar in form to the

a) is isolated] branch, and the point (o, its co-ordinates though entirely separated from the curve, satisfy the equation.

superior

i.e.,

still

To trisect an angle by the aid of the conchoid. Let PCX be the angle which we wish to trisect. From C with any radius as CD describe the semi-circle DAH. From the point A draw AB J_ to CX and make OB = CD. With A as a pole and OB as a modulus construct a conchoid on CX as a directrix. Join H, the intersection of the inferior bnuich and the circle, with A and produce it to meet the 154.

directrix in

K

;

then

CKA =

PCX.

HIGHER PLANE CURVES.

199

3

FIG.

64.

H and C then from the nature HK = HC = OB. From the figure PCX == CAR + CKA CAK = CH A = 2 CKA but PCX = 2 CKA + CKA. hence Therefore CKA = 1 PCX. For join

;

of the conchoid

;

;

We

might have used the superior branch for the same pur

pose.

155.

THE LIMACON.

FIG.

65.

PLANE ANALYTIC GEOMETRY.

200

The Iima9on is the locus generated by the intersection of lines OP, CP which are so related that during their revo lution about the points and C the angle PCX is always two

equal to | POX. 1. To deduce the polar equation. Let be the pole, and OX the initial point of the curve, and let

(OP,

From

the triangle

POX)

:

r

:

a

::

Let

line.

P

be any

then

;

(r, 0).

r

OCP

sin

sin f

::

a sin

TT Hence

=

a

POC, we have

OP OC i.e.,

OC

<9

sin

:

sin 1

:

OPC

;

0.

*

:

t

From Trigonometry

= 3 sin \ r =

sin

hence

= is

4 sin 3 \ = (3 a (3 4 sin 2 ^ 0),

+ 2 cos 0)

a (1

.

.

4 sin 2 \

.

0)

sin^ 0;

(1)

the polar equation of the limayon. 2.

To deduce

From

the rectangular equation.

we have

Art. 35, r

=

#

+

cos

/

for the equations of transformation from polar to rectangular co-ordinates. Substituting these values in (1), we have

ax

Vx x

or

- ^) =

2

4-

?/

2

2

2

y

2

(*

+y

for the required equation.

SCHOL.

1.

From

the triangle

ODA, we have

OD = OA cos = From hence

2 a cos

OP = a + 2 cos 6 OP - OD = DP = a .

(1)

;

;

0.

a

)

...

(2)

HIGHER PLANE CURVES.

201

ODA

and the limayon i.e., the intercept between the circle of the secant through is constant and equal to the radius of the circle. SCHOL.

If 6

2.

If B If 6 If

156.

= 0, r = 3 a = OB. = 90, r = a = OM. = 180, r = - a = OC = 270, r = a = OM

THE LEMXISCATA.

The lemniscata

is

the locus generated by the intersection of

a tangent line to the equilateral hyperbola with a perpen dicular let fall on it from the origin as the point of tangency

moves around the curve.

FIG. 1.

66.

To deduce the rectangular equation.

Since

T

(a",

?/ ) is a

have, Art. 103, Cor

x

The equation

-

point of the equilateral hyperbola,

1,

y

-

==

a2

.

.

.

of the tangent line xx" yy"

=

(1)

TP

is,

a 2 ... (2)

Art. 112,

we

PLANE ANALYTIC GEOMETRY.

202

rr

Since the slope of this line

OP

pendicular

is

the equation of the per

,

is

,*.

0)

and solving for Treating (2) and (3) as simultaneous y",

we

x"

and

find .r

Substituting these values in -

-+ ff

we have

(1),

2

(x 2

or

(x

+

2

T/

2

)

=a

2

2

(x

-

2 .

7/ )

.

.

(4)

for the required equation.

This curve was invented by James Bernouilli. It is quadto the square constructed on the rable, its area being equal semi-transverse axis

OA.

To deduce the polar equation. We have Art. 34, (3), for the equations of transformation x = r cos 0, y = r sin 9.

2.

These values in 2

2

(4) give

+ sin =a

6 \r (cos r4 therefore r2

or is

=

=^

2

2

z

0)J 2 r cos 2

a 2 cos 2

(9

-

2

\r* (cos

sin

2 <9)};

0,

...

(5)

the required equation. a. 1 .-. r cos SCHOL. If 0, cos 20 values with .-. r has two equal 90 2 cos B cos If 45, <

<

opposite signs. If 45, cos 2

=

If 6

45

and

If $

If

= cos 90 =

(9

<135

.-.

=

0.

.-.

r

=

.-.

r

r

r is imaginary.

= 135, cos 2 = cos 270 = - 1 e = 180, cos 2 6 = cos 360 >

=i

=

=

=

6

.

0.

a.

An examination of these values of r shows that the curve formed by the asymptotes of the occupies the opposite angles hyperbola. The curve

is

to both axes. symmetrical with respect

HIGHER PLANE CURVES.

203

TRANSCENDENTAL EQUATIONS. 157.

THE CURVE OF

This curve takes

y

= sin

and may be defined of the

SINES.

name from

its

its

equation

x,

as a curve

whose ordinates are the sines

corresponding abscissas, the latter being considered as

rectified arcs of a circle.

FIG.

To construct the

6?.

Give values to x which differ from each other by 30, and find from a "TABLE OF NATURAL SIXES the values of the corresponding ordinates. Tabulating the result, we have, curve.

-

Value of x

30

=5=

Corresponding .52

6

60

90

=

^= =^=

1.04

1.56

o

120

= i5 = 2.08 6

Value of y

.50

.87

1.00

.87

PLANE ANALYTIC GEOMETRY. Value of x 150 180

210

Corresponding

Value of y

=~- = 2.60 6 = = 3.14 = 3.66 =

.50

TT

.50

6

240

=

~=

4.18

6

270

-

= 4.70

=

1.00

6

300

= i^ =

-.87

5.22

6

330 360

-

= 5.75 = = 2 = 6.28

.50

TT

a smooth curve points and tracing As x may have locus. the have required through them, we of the the and GO equation to yet satisfy any value from

Constructing

these

the curve itself extends infinitely in curve, it follows that abscissas. the both of positive and negative the direction

158.

THE CURVE OF TANGENTS.

FIG.

68.

HIGHER PLANE CURVES. This curve also takes

y

= tan

its

name from

its

205

equation.

x.

To construct the curve. Give x values differing from each other by 30 and find from a Table of Natural Tangents the corresponding values of

Value of x

y.

Tabulating,

Corresponding

we

have,

Value of y

"

30

=

=

-

.57

"

.52

6

60

=

=

1.04

=

1.56

1.73

"

6

90

=

oo

6

120

= 4-^ = 2.08

"

= 5 - = 2.60

"

1-73

6

150

.57

6

180

210

= = 8.14 = 3.66 =

"

TT

.57

"

6

240

= 4.18

==

1.73

"

6

270

= 4.70

=

oo

"

6

300

=

330

=

360

=

= 5.22 ^= 2

TT

=

"

-

-

5.75

6.28

1.73

.57

"

Constructing these points and tracing a smooth curve through them, we have the locus of the equation. This curve, together with that of the preceding article, belong to the class of Repeating Curves, so called because

they repeat themselves infinitely along the X-axis.

PLANE ANALYTIC GEOMETRY.

206 159.

THE CYCLOID.

This curve is the locus generated by a point on the circum ference of a circle as the circle rolls along a straight line. The line is called the BASE of the cycloid the

OM

;

point P, the GENERATING POINT the circle BPL, the GEN ERATING CIRCLE the line HB perpendicular to OM at its middle point, the Axis. The points and M are the VERTICES ;

;

,

of the cycloid. 1. To deduce the rectangular equation, the origin being taken at the left-hand vertex of tJie curve. Let P be any point on the curve, and the angle

which the

circle has rolled,

of the circle,

=2

ButOA = ayOB

CK =

a cos 6

;

=

AB

and

through Let LB, the diameter

AP

= CB

CK.

=a AB = PK = a sin AP = y, CB = a, 0,

0,

hence, substituting,

=a9 y = a x

0.

a.

OA = OB

Then

PCB

a sin

a cos

we have ...

(1)

HIGHER PLANE CURVES. between these equations, we have

Eliminating

=a

x

207

-

1

V2 ay

-

cos"

- ^2 ay -if

.

y .

.

=

2

1

a

-

vers"

(2)

for the required equation. SCHOL. An inspection of (2) shows values of y render x imaginary. (a) that negative

When

(b)

or 4

TT

=

y

a, or 6

TT

a,

or etc.

When

y

;

= 0;

1

vers"

=2

but a vers" 1

TT

a,

hence there are an infinite number

and M.

of points such as (c)

=a

x

0,

=2

x

a,

=a

1

vers"

=

2

=

a

TT

=

OB

but

;

3 TT a, or 5 ?r a, or 7 TT a, or etc. hence, there are a vers" 2 an infinite number of points such as H. 2 a are equations of the limits. and y (d) y v and 2 a there of y between the limits (e) For e very value are an infinite number of values for x. 2. To deduce the rectangular equation, the origin being at the highest point H, 1

;

=

=

We

have for the equations of transformation x TT a x OA OB - PK 2 a AP B / y

= =

These values in x y

But H,

=

,

(1)

= HK =

+ +

above give

= a (0 = a

a sin

TT)

,\

1

a cos

the angle through which the circle has rolled from TT

Hence

= = H

;

hence x = a 6

-\-

a sin

y

= a (COS ff -

x

=

a

1

vers"

,.-.

]

1) }

-+V

2 ay

2 ?/

.

.

.

(5)

The usually attributed to Galileo. With the exception of the conic sections no known curve The fol possesses so many useful and beautiful properties. invention of this curve

is

lowing are some of the more important

:

PLANE ANALYTIC GEOMETRY.

208 1.

2.

3.

=

HDB = a = 3 HDB = 3 OPHM = 4 HB = 8 a.

Area OPHDB O area Area of cycloid OHMO Perimeter

TT

2

.

TT

a2

.

If two bodies start from any two points of the curve curve being inverted and friction neglected), they will (the reach the lowest point at the same time. 4.

H

5.

point

A

H

body rolling

down

this curve will reach the lowest

in a shorter time than if

it

were to pursue any other

path whatever.

SPIRALS. 160. The SPIRAL is a transcendental curve generated by a point revolving about some fixed point, and receding from it in obedience to some fixed law.

The portion of the locus generated during one revolution of the point is called a SPIRE. The circle whose radius is equal to the radius-vector of the generating point at the end of the first revolution the MEASURING CIRCLE of the spiral. 161.

THE SPIRAL OF ARCHIMEDES.

FIG.

70.

is

called

HIGHER PLANE CURVES. This spiral

209

the locus generated by a point so moving its radius-vector to its vectorial angle is

is

that the ratio of

always constant.

From

the definition,

we have

r

=c r = c

hence is

.

.

.

(1)

the equation of the spiral.

To construct the

spiral.

Assuming values for and finding from ing value for r, we have Values of

45

(1)

the correspond

Values of r

Corresponding

=^

^c

4

4

90=^ 4 135

= ^L

180

=<*

270

^ =^

360

=2

225

2

_^ c 4

3*

=

-

..

4

T"

Constructing these points and tracing a smooth curve through them, we have a portion of the spiral. Since Since

number

= =

gives r

=

0, the spiral passes through the pole. gives r makes an infinite oo, the spiral of revolutions about the pole. (9

=

oo

Since 2 TT gives r the measuring circle.

=

=2

IT c,

OA (=

2

IT

c)

is

the radius of

PLANE ANALYTIC GEOMETRY.

210 162.

THE HYPERBOLIC

This curve

SPIRAL.

the locus generated by a point so moving that the product of its radius-vector and vectorial angle is always constant.

From

is

the definition r

=

we have c,

or for the equation of the spiral.

FIG.

71.

To

construct the spiral. Giving values to 0, finding the corresponding values of

we have Values of

Corresponding

Values of r oc

45

=Z

"

r,

HIGHER PLANE CURVES. Values of

135= 180

211

Values of r

Corresponding

7T

O

4

=

7T

-

"

7T

7T

225

5

=

TT

4 270

OTT

=-TT

~

"

4

315

GTT

=-TT

"

4

360

=

2

TTT "

TT

oo

Constructing the points we readily find the locus to be a curve such as we have represented in the figure. oo there is no point of the spiral Since 6 gives r

=

=

corresponding to a zero-vectorial angle. Since oo gives r 0, the spiral makes

=

=

an

of revolutions about the pole before reaching Since 6 2-n- gives

infinite

number

it.

=

c is

the circumference of the measuring circle. Let P be any point on the spiral then

SCHOL.

;

(OP,

POA) =

(r, 0).

and OP as a radius describe the arc PA. circular r 6, and from (1) c r Arc PA measure, By hence Arc PA c

With

as a centre

=

=

i.e.,

=

;

;

the arc of any circle between the initial line and the

spiral is equal to the circumference of the

measuring

circle.

PLANE ANALYTIC GEOMETRY.

212

THE PARABOLIC SPIRAL. This spiral is the locus generated by a point so moving that the ratio of the square of its radius-vector to its vectorial angle is always constant. From the definition we have 163.

r*

or,

=

c

.

.

.

(1)

for the equation of the spiral.

FIG.

To construct the Values of

45

=

72.

spiral.

Corresponding

Values of r

!T

4 90

=

2* 4

135

=

^ 4

180

=

7T

((

V2 C7T V3c

HIGHER PLANE CURVES. Values of

Corresponding

^

225

=

270

= ^T

3i5

= IJL

360

=2

213

Values of r

-JV5C

4 7C7T

oo

00

Constructing these points and tracing a smooth curve through them we have the required locus. Since gives r 0, the spiral passes through the pole. Since oo gives r oo, the spiral has an infinite num ber of spires.

= =

= =

THE LITUUS

or

This curve has for

its

164.

r2

=

TRUMPET. equation

c,

FIG.

73.

PLANE ANALYTIC GEOMETRY.

214 If

=

0,

=

r

;

6

if

The Logarithmic

165.

This spiral the ratio of vector

oo

is

=

is

its

r

oo,

an asymptote to

initial line as

=

0.

its infinite

This curve has the branch.

/SJjiraZ.

the locus generated by a point so moving that vectorial angle to the logarithm of its radius

Hence

equal to unity.

or passing to equivalent r

a*

.

numbers (a being the .

.

(1)

for the equation of the spiral.

To construct

the spiral. r

is

Let a

=

2,

then

= 29

the particular spiral

we wish

to construct.

base),

we have

HIGHER PLANE CURVES. Values of 6

215 Values of r

Corresponding

1

= 57.3 2 = 114. 6 3 = 171.9 4 = 229. 2 1

2 4

"

8

16 "

CO

<

- 1

= - 57.3

.5

-2=-114.6 - 3 = - 171.9 _ 4 = - 229. 2

.25

.125 .062

oo

A

smooth curve traced through these points

will

be the

required locus.

=

Since a, it

=

1 whatever be the assumed value of gives r follows that all logarithmic spirals must intersect the

initial line at a unit s

Since

number

=

distance from the pole.

=

oo the spiral makes an infinite gives r of revolutions without the circle whose radius 1.

GO

,

OA =

Since 6

r =

=

GO gives 0, the spiral of revolutions within the circle

number

makes an

OA

infinite

before reaching

its pole.

EXAMPLES. 1.

Discuss and construct the cubical parabola B

y

X = ~rp*

2.

What

the polar equation of the Iima9on, Fig, 65, the

is

pole being at

C

?

Ans.

r

= 2 a cos -

0.

3 3.

cata

Let is

OF

= OF = a Vf

Fig. 66.

Show

that the lemnis-

the locus generated by a point so moving that the

PLANE ANALYTIC GEOMETRY.

216

product of its distances from the two fixed points F, constant and

v

F

2

Discuss and construct the loci of the following equations 4. 5.

6.

7.

x

= tan

y

= cos

2/

a;

8.

is

12.

y.

a3

=x

axy.

=

x.

13.

jcl -j-

= sec*.

14.

4 + |r =

y?

a2

= sin y.

15.

?

=

= cot x.

16.

r

=

:

s

1.

l.

&3

sin 2

0.

<?/

sin 2

9.

10.

y

= cosec

y

= 3x ~ l

20.

a?

=a

n

sin 3 -

.

o

Q 2

2

?/

,

-4- a-?/

18.

r 2 sin 2

20

=

1.

o 2

=

1 rv

-|

19.

1.

r

= -1 1

~T~ Sill

C .

sin

Discuss and construct the locus of the equation

y

_

96 a 2 ;/2

+ 100 ciV

2

J- V(ic 21.

r

x3

no .

17.

ic.

Show

asymptotes Ex. 20.

that y of the

6 a) (a

^ x are locus

x^

=

+ 6 a)

or (x

8 a) (x

+ 8 a).

the equations of the rectilinear represented by the equation of

SOLID ANALYTIC GEOMETRY,

PART

II.

CHAPTER CO-ORDINATES.

I.

THE TRI-PLANAR SYSTEM..

determined when we know its distance and direction from three planes which intersect each other, these distances being measured on 166. The position of a point in space

lines it

is

is

Although drawn from the point parallel to the planes. immaterial in principle what angle these planes make

with each other, yet, in practice, considerations and simplicity have made it usual to take They are so taken in what follows. angles. Let XOZ, ZOY, YOX be the CO-ORDINATE Let OX, secting each other at right angles.

CO-ORDINATE AXES and 0, their

intersection,

of convenience

them

at right

PLANES inter OY, OZ be the the ORIGIN of

CO-ORDINATES. - XYZ. Let P be any point in the right triedral angle the we know lengths Then P is completely determined when

and directions of the three lines PA, PB, PC let fall from this point on the planes. As the planes form with each other eight right triedral which satisfy angles, there are evidently seven other points the condition of being at these distances from the co-ordi nate planes.

The ambiguity

is

217

avoided here (as in the case

SOLID ANALYTIC GEOMETRY.

218

of the point in a plane) by considering the directions in which these lines are measured.

Assuming distances

to the right of

YOZ as positive,

distances

to the left will be negative.

4-Z

A/

-Y

-X

4-X

Assuming distances

XOY

6o*;e

as positive, distances

will be negative.

in front

Assuming distances

o/XOZ

as positive, distances ^o

^Ae rear will be negative. r Calling x 7/ z (= BP, ,

AP, CP, respectively) the co-orc?ithe FIRST ANGLE, we have the follow

,

wa^es of the point P in in the ing for the co-ordinates of the corresponding points

other seven

:

SECOND ANGLE, above

XY

plane, to left

of XZ plane, (- x y z ) P 2 THIRD ANGLE, above XY plane, to r of XZ plane, (- x - y z ) P 8

YZ

plane, in front

.

,

,

.

,

,

left

YZ

plane, in rear

CO-ORDINATES.

219

XY plane, to right YZ P y (x below XY plane, to right YZ

plane, in

FOURTH ANGLE, above

XZ

rear of

f

plane,

front of

XZ

,

,

FIFTH ANGLE,

f

plane, (x

y

,

SIXTH ANGLE, below

,

XY

.

)

4

s

PS-

)

plane, to left

YZ

plane,

in

plane, in front

P6 ) y plane, ( SEVENTH ANGLE, below plane, to left YZ plane, in f z ) P7 x rear of XZ plane, ( y EIGHTH ANGLE, below plane, to right YZ plane, in rear

of

XZ

x

.

,

,

XY

.

}

,

of

XZ

-

f

plane, (x

,

XY - P

i/,

)

8.

EXAMPLES. 1.

In what angles are the following points

(1, 2,

-

3),

(-

-

1, 3,

2),

(-

1,

-

2,

-

:

4), (3,

-

2, 1).

2. State the exact position with reference to the co-ordi nate axes (or planes) of the following points :

(0, 0, 2),

- 1, - 1),

0), (0, (5, 1,

3.

(0),

2, 1, 2), (3, 1, 0), (3, (3,

(1, 1,

0,

-

1),

(1,

-

2,

-

3),

1, 2), (2, 0, 3),

(0,

0,

-

2),

((4,

1,

1,

2,

2),

1).

In which of the angles are the X-co-ordinates positive ? In which of the angles are the Y-co?

In which negative

ordinates positive ?

In which are the Z-co-ordinates negative?

167. Projections. The projection of a point on a plane is the foot of the perpendicular let fall from the point on the Thus A, B, and C, Fig. 75, are the projections of the plane. point P on the planes XZ, YZ, XY, respectively.

The

projection of a line of definite length on a plane is the on that plane. OC, Fig. 75, is the projection of OP on the plane.

line joining the projections of its extremities

Thus The projection

XY

of a line of definite length on another line is that portion of the second line included between the feet of the perpendiculars drawn from the extremities of the line of definite length to that line.

SOLID ANALYTIC GEOMETRY.

220

Thus OM, Fig. 75, is the projection of OP on the X-axis. The projections of points and lines as above de ;NOTE. Unless otherwise stated, all projections fined are orthogonal. will be so understood in what is to follow. 168.

To find the length of a

line joining two points in space.

P"

FIG.

76.

be the given points. P (a? tf, ) and and the be L (=P required length. Draw N Join OX. to NB to OY and CD NA to OZ to NO. P M draw and and C We observe from the figure that L is the hypothenuse of a are P M and right angled triangle whose sides

Let Let

V"

(z",

,

y" ,

*"}

P"C

P")

||

;

||

;

||

||

P"M.

Hence

= NO =

but

= ...

L

=

(P"C

CO- OR DINA TE S.

=

= 0,

COR. If x 0, if with the origin and ...

L

=

= 0,

s

+

2 V*"

221

then the point

+

2 y"

2 s"

...

P

coincides

(3)

expresses the distance of a point from the origin.

Given the length and the directional angles of a line joining any point with the origin to find the co-ordinates of the 169.

point.

The Directional angles of a line are the angles which makes with the co-ordinate axes. Let its

P

(x, y, z),

Fig. 75, be

distance from the origin.

the line

OP = L will be POX, POY, POZ = a, fa y,

point, then

any Let

respectively.

OM, ON, OR

Since

on X, Y,

(== x, y, z) are the projections of

Z, respectively,

x

y z

OP

we have

= L cos u = L cos = L cos y

}

I

ft

...

(1)

for the required co-ordinates.

COR.

Squaring and adding equations

+y + =L x* + y + * = L

x* but

hence

That

2

2

2

2

2

2

2

cos u -f cos is,

the

2

/? -f-

(1),

we have

2

2 -f- cos (cos Art. 168 (3)

cos 2 /

sum of the squares of

=

;

1

.

.

ft

-f cos

2

y)

;

.

.

(2)

the directional cosines of a

space line

is equal to unity. SCHOL. The directional angles of any line, as P Fig. 76, are the same as those which the line makes with three lines drawn through 1J/ to X, Y, Z. The on projections of P - x f if tf, z three such lines are Art. 168; hence P",

||

x"

,

x"

y" "

P"

-

x = L COS a - y = L cos z = L cos

ft

"

_^

"1

[

...

(3)

SOLID ANALYTIC GEOMETRY.

222

EXAMPLES. Required the length of the points 1.

(-

(1, 2, 3),

(3,

- 2,

following

-

(0, 3, 0), (3,

(0, 0, 0), (2, 0, 1).

Ans. 5.

0), (2, 3, 1).

(0, 4, 1),

V27.

(-

2,

-

1,

^ws. 6.

1, 0).

Ans. 7.

joining the

V14.

^tws. 3.

4.

2, 1, 1),

Ans. 2.

lines

:

(1,

-

-/5.

-2). V38.

2, 3), (3, 4, 6).

Ans.

5.

Find the distance of the point

(2, 4,

3)

7.

from the origin

;

also the directional cosines of the line. 8.

A

line makes equal angles with the co-ordinate axes. are its directional cosines ?

What 9.

Two

What

is

of the directional cosines of a line are

If (x

10.

Vf

and

the value of the other ? f ,

y

,

z

)

and show that (a;",

y",

z")

are the co-ordinates of the

extremities of a line

x

+

x"

y

+ 2

y"

+

z "z"\ z"

2-J

are the co-ordinates of its middle point.

THE POLAR SYSTEM. 170. The position of a space point is completely determined when we know its distance and direction from some fixed point.

For a complete expression of the direction of the point it is The angles necessary that two angles should be given. usually taken are

The angle which the line joining the point and the makes with a plane passing through the fixed the line join and 2d, The angle which the projection of point line in the fixed a with makes that on the plane points ing 1st,

fixed point ;

plane.

CO-ORDINA TES.

FIG.

223

77.

Let be the fixed point and P the point whose position we wish to determine. Join O and P, and let XOY be any plane passing through 0. Let OX be a given line of the plane XOY. Draw PB 1 to XOY and pass the plane PBO through PB and OP. The intersection OB of this plane with XOY will be the projection of

BOX

and the distance

(go)

OP on XOY. The angles POB (0), OP (r), when given completely de

termine the position of P. For the angle cp determines the determines the line OP in that plane, plane POB, the angle and the distance r determines the point P on that line. This method of locating a point is called the POLAR SYS TEM. The angles and qo are called VECTORIAL and

ANGLES,

the distance r

The point terms of

called the

RADIUS VECTOR

when written (r, 9, POLAR CO-ORDINATES.

P,

its

is

<p),

is

of the point.

said to be expressed in

It is evident <P,

and

may

all

to 360 to and by giving all values from values from to oo to r that every point in space

be located.

171. Given the polar co-ordinates of a point to find its rec tangular co-ordinates. Draw OY 1 to OX and in the plane BOX draw OZ 1 to ;

SOLID ANALYTIC GEOMETRY.

224

OY

and OX, and

Draw

BA

||

to

(OA, AB, BP)

From

=

From

OB =

From

(x,

BOP, we have sin

6.

ABO, we have

= OB

r cos

cos

x

.:

cp.

= r cos

the same triangle

y y x

cos

cp.

we have

= OB sin = r cos 6 sin qo,

= r cos 6 cos = r cos 9 sin y z = r sin

Henee

co-ordinate axes.

then, Fig. 77, y, z) are the rectangular co-ordinates of P.

=r

the triangle

x

But

;

the triangle z

OX, OY, OZ be the

let

OY

(p.

cp

cp

1

V

...

(1)

express the required relationship. Con. If P (x, y, ) be the co-ordinates of any point on a locus whose rectangular equation is given then equations (1) are evidently the equations of transformation from a rectangu lar system to a polar system, the pole being coincident with the origin.

y,

Finding the values of we have

r,

6

and

cp

from

(1) in

terms of x and

r= Vx + if + = tan+ = tan2

1

(9

1

for the equations of transformation from a polar system rectangular system, the origin and 2)ole being coincident.

to

a

225

CO-ORDINA TES. EXAMPLES. Find the polar co-ordinates of the following points 1.

2.

3.

(2,1,1).

(V3,

1,

(10,2,8).

4.

2 V3).

(3,

-

1, 4).

Find the rectangular co-ordinates of the following 5.

(5,

:

:

7.

30, 60).

(6,=,

6

Find the polar equations of the following and origin being coincident

surfaces, the pole

:

9.

10.

+ ^2 + = a z + sx + ty - c = 0.

X2

Ans. r

z

,2

m

sin

+ 5 cos

Find the directional cosines of the ing pairs of points 11.

(1, 2,

12.

(4,

-

-

(-

1, 3, 2).

and 15. If (V, /, ) the point space points, show that (z",

/

\

mx"

g>

cos

-f

sin

a.

qo

lines joining the follow

:

1), (3, 2, 1).

1, 2),

cos

=

+ nx

r

divides the line joining n. each other the ratio :

14.

(0, 2, 0), (3, 0, 1).

1,

5), (4, 5, 6).

be the co-ordinates of two

")

+ ny m+w

them

-

(2,

my"

7wT+ %

m

y",

-

13.

into

+ nz m+n

f

mz"

two parts which bear to

SOLID ANALYTIC GEOMETRY.

226

CHAPTER

II.

THE PLANE. To deduce the equation of the plane. Let us assume as the basis of the operation the following 172.

property If on a perpendicular to a plane two points equidistant from the plane be taken, then every point in the plane is equidistant :

these two points, equally distant.

from

and any point not

in the plane

is

un

FIG.

Let it

ABC

in E.

Draw

be any plane.

Produce

OR

until

the plane is equally distant be any point of the plane let ;

OR 1

to

ABC, and meeting

RR = OR = p. and R from

Every point Let

.

ON, MN,

MR

,

P

in

(x, y,*,)

the co-ordinates

THE PLANE. of

R =

d,

e,

From

Then from

/, respectively.

PR

==

- xY + (e-

(d

the same article, equation

OF = 2

xs

227

(3),

+f+*

we have

Art. 168, (2),

yY + (/- zY we have

2 5

hence, by the assumed property, (d

- xY + (e-

yY

+ (/- *) = x + y + * 2

2

2

2 -

Simplifying this expression, we have 6 + f + ey +fz = d*+ *

dx

^

z

-

/1X (1)

...

for the required equation.

To find the equation of a plane in terms of

173.

pendicular

to it

from

the origin

and

the

per

the directional cosines of

the perpendicular.

be the directional angles of the perpendicu Since ON, MN, lar OR (=2p), Fig. 78. (= d, e, f) the projections of OR on the co-ordinate axes, we have (Art.

Let

/8,

,

and

7

169, (1)

=

MR

7

)

= 2p cos e = 2^ cos

d

f

1

(3

Substituting these values in cos2 ft H~ cos2 Y that cos 2

+

cc

cos

+

?/

cos

for the required equation. EQUATION of the plane.

Since

2

d

(1), 1?

(1)

Art. 172,

+ z cos 7 = p

^8

e

and remembering

we h ave

Equation

OR = 2p = V^ +

cos

...

[

cos 7

2p

2

+/

(2) is

.

.

.

called the

(2)

NORMAL

2 ,

equations (1) give e

ANALYTIC GEOMETRY.

.SOLID

228

Substituting these values in

we have

(2),

d

e ~

~

y

for the equation of the plane expressed in terms of the co-or dinates of a point on the perpendicular to it from the origin

and the perpendicular. COR.

1.

If

p

=

we have

in (2),

+ y cos + z cos y =

x cos

.

ft

.

.

(4)

for the equation of a plane through the origin.

COR.

2.

= 90,

If

y cos is

= 0,

cos

(3

+ z cos 7

hence

=p

.

(5)

.

.

the equation of a plane 1 to the YZ-plane. 90, we obtain similarly ft

If

=

x cos

+ z cos 7 = p

for the equation of a plane If Y 90, then

.

.

.

(6)

to the XZ-plane.

J_

=

x cos is

+

the equation of a plane

COR.

3.

= 90

If

cos

?/

and

1L .--cos

=p

/?

.

.

.

(7)

1

to the XY-plane.

(3

= 90,

...

then

(8)

7

is

1

the equation of a plane

to

YZ

and XZ, and hence

XY. Similarly,

we

find

y

= --* cos

(10)

COS for the equations of planes

to ||

XZ

and

YZ

respectively.

||

to

THE PLANE. COR.

If

4.

=

p

y

and

in (8), (9),

=

...

I

229

(10),

then

(11)

J

are the equations of

XY, XZ, and YZ,

respectively.

To find the equation of a plane in terms of

174.

in

its

tercepts.

OB =

OA

Let, Fig. 78, =a, perpendicular to the plane

=

OC

b,

OE (=p)

Since

c.

ABC, we have from

the right

is

tri

OEA, OEB, and OEC

angles

=

COS

...

0080=| COS 7

(a)

f

= J

Substituting these reducing,

we have * a

values

+ -|b + *c = 1

normal

the

in

.

.

.

and

(1)

for the required equation. Equation (1) METRICAL EQUATION of the plane.

175.

equation

is

called the

Every equation of the first degree between

SYM

three vari

ables represents a plane.

The most general equation of three variables is of the form

the

-f

By

-f

C*

=D

.

.

.

(1)

Dividing both members of this equation by we have

VA + 2

degree between

ABC Ax

first

B2

+C

-2 2

A2

x

VA ~+ B + ni C ^/A22

~"~

+B +C 2

i

T^22

i

2

* y*

i

VA

2

-|-

B2

+C

VA + B + C 2

2

2

2 ,

SOLID ANALYTIC GEOMETRY.

230

of Art. 173, we see that the co the directional cosines of some are efficients of the variables co-ordinates of one of its the of terms in line

Comparing

(2)

with

(3)

expressed

and that the second member measures the distance of the a plane from the origin hence (2) and therefore (1) is

points,

;

equation of a plane.

To find the equations of the traces and the values of the intercepts of a plane given by its equation. 176.

7

FIG.

ABC

Let

be the plane and

Ax

79.

let its

+ By + C2 = D.

To find the equations of the

1.

equation be

traces

AB, BC, AC.

the given plane with traces are the intersections of their equations, we the co-ordinate planes hence, combining

The

;

have

Ax

+ By + Cz = D | z

Ax

Ax

=

/<

Ax

+ B^ =

D<

Trace

011

XY (AB)

.

.

.(1)

)

+ By + C = D | Cz

=D

")

.

Ax + Cs = D I

. j>

r ,

Q,,

__

_

j)^

Trace on

X Z (AC)

Trace on YZ (BC)

.

.

,

(2)

... (3)

THE PLANE.

231

intercepts OA, OB, OC. The points A, B, C are the intersections of the given plane with the co-ordinate planes taken in pairs ; hence, combining

To find the

2.

we have

their equations,

A*

+ By + C* = D (4) -nL

y-Q

-D .

._ 00 COR.

If

the

= cept = OB

oo

.

.

.

.

.

(5)

(6)

plane is perpendicular to XZ its Y-inter0. Making B hence, equation (5), B

Ace

=

=

;

in the general equation,

we have

+O=D

.

.

.

(7)

a perpendic (7) and (2) are the same equations; hence, ular plane and its trace on the plane to ivhich it is perpendic ular have the same equation.

But

=

cos 7 177. If x cos a. -j- y cos ft -|p be the normal equa a plane, then x cos u -\- y cos (3 -j- z cos 7 p -^ d is the

=

tion of

equation of a parallel plane at the distance d from it. For the directional cosines of the perpendiculars are the

same hence, the perpendiculars are coincident hence, the The distance of the planes apart is equal planes are parallel. to the difference of the perpendiculars drawn to them from the origin but this difference is p -j- d p i.e., J- d. Hence, ;

;

;

;

the proposition. COR. If (V, ?/,

from the origin is p X COS _J_ d

=

)

be a point in the plane whose distance d then

-]-

f

-|-

;

if COS

(3

+

COS 7

p

.

.

.

(1)

SOLID ANALYTIC GEOMETRY.

232 is its

distance from the parallel plane whose distance from From equations (a), Art. 174, we have is p.

the origin

=-

cos

a hence

+ cos

cos 2

These values in

(1)

,

cos

+ cos

2

=

j8

cos 7

,

=

P;

6

=

2

/

c

^& + % v

-~ -

+

C

=

1-

2

give

for the expression of the distance of a point which is given in its symmetrical form.

from a plane

Let the student show that the expression for d becomes

_A^Vy

d

*

+ Cz:_-^ +B +C

m

(3)

2

2

of the plane is given in its general form. the significance of the double sign in (1), (2), and

when the equation

What

is

(3)?

178.

the equation of a plane which passes through

To find

three given points.

Let (V, y

,

z

),

(x",

y" ,

*"),

we seek

Since the equation

(*"

is

,

y",

which A, B,

C,

D

")

^ the given points.

that of a plane,

Ax + By + Cz = D

in

*

f

.

it

must be

(1)

are to be determined by the conditions

imposed. the co Since the plane is to contain the three given points, its equation hence, ordinates of each of these must satisfy the following equations of condition ;.

:

Ax Ax"

Ax

+ B// + C.~ = D =D + B/ + = D. C + By + C,~"

"

"

"

These three equations contain tlie/ow unknown quantities from the equations the values of A, A, B, C, D. If we find

THE PLANE. B,

C

D

in terms of

233

and the known quantities, and substitute

term of the resulting equation will (1), each Let as a factor.

these values in

D A=

contain

AT>,

B

Substituting in

= B D,

(1),

C

= CD be

the values found.

we have

A Daj + B D?/ + C Vz = D. A a + B y + C s = 1 ...

...

is

(2)

the required equation.

179.

The preceding discussion has

every equation of

the

first

elicited the

fact that

degree between three variables It remains to be shown that every

represents a plane surface. equation between three variables represents a surface of some kind.

Let

z

=/(,

y)

C

1)

Since be any equation between the three variables (x, y, z). x and y are independent, we may give them an infinite number For every pair of values thus assumed there is a of values. These values in (1) give the corre point on the XY plane. sponding value or values of z, which, laid off on the perpen dicular erected at the point in the XY plane, will locate one

But the number or more points on the locus of the equation. of values of z for any assumed pair of values of x and y are necessarily finite, Avhile the number of pairs of values which be given x and y are infinite a surface of some kind.

may

;

hence

(1)

must represent

If

-/(*,

z

=

q>

y)

(x, y)

.

.

.

(2)

j

be the equations of two surfaces, then they will represent their For these equa line of intersection if taken simultaneously. tions can only be satisfied at the

same time by the co-ordinates

of points common to both. Hence, in general, two equations between three variables determine the position of a line in space.

SOLID ANALYTIC GEOMETRY.

234

z

If

=f(x,y)

= =

<P

V

(^ y) (x, y}

be the equations of three surfaces, then they will represent simul their point or points of intersection when considered as Hence, in general, three equations variables determine the positions of space points.

taneous.

letiveen

three

EXAMPLES. Find

4.

11.

the traces

and intercepts

of the following planes:

2

The

from directional cosines of a perpendicular let fall

of the required the equation the origin on a plane are | o o o |; == 4. plane, the length of the perpendicular ,

\

,

An,

5+^ + f-l.

the plane whose intercepts are Required the equations of as follows

:

12.

1,2,3.

13.

2,

-

14-

1,

15.

3.

i,, -

1,

-

-2. "

,

4.

of the equation of the plane, the equations ? 4 z x 4 and 3 y -jwhose traces are x 4. z 3 ?/ a 16.

What

is

=

=

+ =

THE PLANE. The

17.

co-ordinates of the projection of a point in the 2ov. the plane are (2, 1) required 3y-}-2z

=

plane x

235

XY XY plane.

;

the distance of the point from the

Ans.

f.

Write the equations of the planes which contain the follow ing points

:

(-

-

18.

(1, 2, 3),

19.

(4, 1, 0), (2, 0, 0), (0, 1, 2).

20.

(0, 2, 0), (3, 2, 1),

21.

(2, 2, 2), (3, 3, 3),

1, 2,

1), (3, 2, 0).

((-

1, 0, 2).

!,-!,-

1).

Find the point of intersection of the planes 23. 2 x - y + z = 10. 22. a- + ^ _ z = 4. x + ?/-2 = 3. 2^-33 + ?/ = 10. 2 x - 4 y + 5 = 6. x -f T/ - 2 = 2. 24.

Find the distance of the point

(2, 1, 3),

from each of the

planes 25. .x cos

26.

x

27.

+ y cos

60

=

+ x

+ ^2 + 3 z = 4.

z

+z

60

3y

= 9.

cos 45

8.

--- + - =

325

28.

1.

Find the equation of the plane which contains the and is parallel to the plane x 2 ij -\- z 6. Reduce the following equations to their normal and sym metrical forms 29.

point

=

(3, 2, 2)

:

30.

2x

3y

+z= 2

qo

o<6.

4

31.

.

x

1 ,

~\-

y

-

#

=

cc

+2y

z

= ~.

o.

33. If s, s represent the sides of the triangle formed by the traces of a plane, and a, b. c represent the intercepts, 2 2 2 s 2 (a 2 -f b 2 s c ). show that s 2 s"

,

+

+

"

=

+

236

SOLID ANALYTIC GEOMETRY.

CHAPTEE

III.

THE STRAIGHT 180.

To deduce

LINE.

the equations of the straight line.

straight line in space is determined when two planes which intersect in that line are given. (See Art. 179.) The equations of any two planes, therefore, may be considered as

The

when taken simultaneously. Of the of pairs of planes which intersect in and de termine a space line, two of its projecting planes that is, two planes which pass through the line and are perpendicular representing a space line infinite

number

two of the co-ordinate planes give the simplest equations. For this reason two of these planes are usually selected.

to

THE STRAIGHT LINE. Let then

237

PBM

its

be the plane which projects a space line on XZ, equation will be of the form

x in which s = tan

= sz

a ( see Art. 176, Cor.) and a OA.

-j-

ZBP

=

M

Let P B be the plane which projects the line on YZ, then its equation will be

in

=

y=t*+b,

=

which t tan ZB P and b OA. But the two planes determine the line x

= y

;

hence

= sz + a

are the required equations. COR. 1. If a and b

x

.

=

= sz = tz

0,

then

)

j

which pass through the 0, we have

are the equations of a line and t COR. 2. If s

=

=

for the equation of a line

origin.

to the Z-axis. Since equations (1) express the relation existing between the co-ordinates of every point on the space line, if we eliminate Z from these equations we obtain the immediate

COR.

||

3.

relation existing between x and y for points of the line. But this relation is evidently the same for all points in the pro and therefore for jecting plane of the line which is 1 to

XY

trace on

its

on

XY

;

XY.

But the trace

hence, eliminating, tx

is

the projection of the line

we have

= bs

at (4) for the equation of the projection of the line

sy

.

.

on XY. have found, Art. 169, Schol., for the length of a joining two points the expression

181. line

.

We

T.-^-y _ COS a

y"

-j/

COS

(3

^

*"-:. COS 7

SOLID ANALYTIC GEOMETRY.

238

L and

Eliminating ordinates of

letting

cos

cos for the

182.

z"

x",

y",

(=

x, y, z)

any point on the line, we have x _ y x y _ z

To find where a

x-jx

cos 7

(3

SYMMETRICAL EQUATION

be the co

of a straight line.

line given by the equations

of

its

projections pierces the co-ordinate planes.

Let 1.

X

== __

f

? I be the equations of the fz-\-0 ) y To find where the line pierces the XY-plane.

The equation

of the

line.

"f

XY-plane

is

Since the point of intersection is common to both the line its co-ordinates must satisfy their equations.

and the plane,

Hence

= sz y =

x

t-Z

-j-

a

-\-

b

are simultaneous equations.

So treating them we find

(a, b, 0)

to be the required point. To find where the line pierces the XZ-plane. 2.

The equation

of the

XZ-plane

is

y-o. Combining

this with the equations of the line,

for the required point. To find where the line pierces the XZ-plane. 3.

x y

= sz = tz

-{-

-f-

hence <),

s is

the required point.

b y are

simultaneous

;

we have

THE STRAIGHT LINE. To find the equations of a

183.

pont. Let

r

(x

j

y

,

z

f

239 a given

line passing through

be the given point.

)

Since the line

is

straight its equations are

a)

= te + b

y in

ft

>

which the constants are unknown. Since

to pass through the point (# satisfied for the co-ordinates of

f

it is

must be

the equations of condition

,

y ,z

) its

equations

this point

;

hence

:

= sz + a = tz + b \ f

x

,n\

)

1

y

f

As the

three conditions imposed by these four equations in cannot, general, be fulfilled by a straight line, we must eliminate one of them. Subtracting the first equation in

group (2) from the first in group (1) and the second in group (2) from the second in group (1), we have x = s (z z - y = (Z - Z

x

)

t

y

,,

| }

)

for the general equations of a straight line passing through

a

point.

To find the equations of a

184.

line

passing through two

given points.

Let

f

(x

,

As the

y

,

z

fl

),

(x

,

y",

z")

be the given points.

line is straight its equations are

... in

which the constants are

As

it is

to be determined.

to pass through (x

x

f

= sz = tz

f

r

y

(i)

,

y

,

z

),

we must have

SOLID ANALYTIC GEOMETRY.

240

As

it is

to pass x"

y"

through

= =

sz"

tz"

(#",

+ +b

a

y",

z"),

I

we must have

also

(3)

}

As these six equations impose four conditions on the line, we must eliminate two of them. The conditions of the the line to pass through the hence we must eliminate the other two. Elimiting a and b from groups (1) and (2), by subtraction, we hav6 z ) x x = s (z proposition, however, require

two points

;

x

r

y

-y =

t

Now, eliminating a and X -X =

(z

b

-*

Eliminating

s

-y"

and

=

s (Z t

(z

and

(2)

_ -

4)

(3),

we have

-

z")

z")

between

t

(

from

"

,j

|

) j

t j

and

(4)

(5),

we have

,

for the required equations.

EXAMPLES. 1.

Given the

line

the projection on

x ^

= z 1 =4^ _ 3 *?

-4-

) r

required the equation of

XY. Ans.

2.

How

2x

y

=

5.

are the following lines situated with reference to

the axes ?

x=2\ y=0\

?/

= 0)

a;

=

)

Find the co-ordinates of the points in which the following lines pierce the co-ordinate planes

:

x=-z-l)

,

2x

=

THE STRAIGHT LINE. 6.

Given

(2, 1,

2), (3, 0, 2)

241

required

;

The length of the line joining the points. The equation of the line. The points in which the line pierces the

(a) (7>)

(c)

co-ordinate

planes.

Find the equations of the points

7. (2, 1, 3), (3, 8. (

-

1,

- 1).

9.

- 1, 2, 3), ( - 1,0, 2).

11.

of 45

which pass through the

lines

:

The

(2,

-

-

10. (1,

1,

1, 0), (3, 0, 0).

-

(-

2),

projections of a line on XZ and YZ make angles respectively with the Z-axis, and the line in (1, 2, 3)

;

required the equations of ~ ^^ w. -/

*v

t*/ /y>

y 12.

The

2, 4, 3) 13.

(-

15.

= -4- - V 3 + V3

vertices of a triangle are (2, 1, 3), required the equations of its sides.

Is the point (2,

1,

- 2) ?

3)

Assume two

(3,

0,

1),

on the line which passes through

Write the equations of a

NOTE.

them

is

;

1, 3, 2), (3, 2,

14.

- 3).

and 30

space contains the point the line.

(

-2,

1,

line

which

points in the plane

the plane

lies in

;

the line joining

will be a line of the plane.

Find the equation of a line through y + z = 4.

(1,

2,

2)

which

parallel to the plane x 16.

Find the point

pierces the plane 3 x 17.

-\-

in

2 y

which the z

= 4.

line

^ y

+2^ = 3 z + 2 = 0)

)

Required the equation of the plane which contains the

SOLID ANALYTIC GEOMETRY.

242

Find the point of intersection of the planes z = 4, x x + 3y y + z = 2, 2 x

18.

+ y = 3.

Find the equations of the projecting planes of the =4 a._2v

19.

line

+

20.

planes cross ? z 2y 2, x

Which angles do the following x -y. + z = 4:,2x +y -3z =

185.

To find the

equations.

Let

x

- =

-

1.

intersection of two lines given by their

= sz + a )

y-. + ft|

,

Mld

x

=

+

s z

y-^ +

a \ f

ft ; intersection is com of the Since be the given equations. point mon to both lines, its co-ordinates must satisfy their equations. Hence these equations are simultaneous. But we observe that there are four equations and only three unknown quantities the lines hence, in order that these equations may consist (and a certain relationship must exist between the con ;

intersect),

which enter into them. To find this relationship, we eliminate x between the first and third, y between the second and fourth, and z between the two equations which result. stants

We

thus obtain (s

-

f

s ) (b

-

&

)

-

-

(t

(a

-a = )

for the required equation of condition that the two lines shall If this condition is satisfied for any pair of as intersect. lines the lines will intersect, and we obtain the co-ordinates of this point by treating any three of the four which represent them as simultaneous. So treating

sumed

equations the first, second, and third

we

obtain a

a

s

s

/

,

;

i

^>

a s

for the co-ordinates of the required point. were prepared to expect NOTE.

We

a s

that our analysis

some conditional equation, for in assuming the it would be an exceptional case equations of two space lines

would lead

to

THE STRAIGHT LINE. if

we

243

assumed them that the

so

lines which they represent Lines may cross each other under any angle in space without intersecting. In a plane, however, all lines except parallel lines must intersect. Hence, no conditional

intersected.

equation arose in their discussion. 186.

To find the angle between

tiuo

lines,

equations, in terms of functions of the angles make with the axes.

x

Let

= sz

-f-

given by their

which the

lines

a

be the equations of the two lines. The angle under which two space lines cross each other is measured by the angle formed by two lines drawn through some point parallel to their directions.

FIG. .

Let

OB

and

OC

will be their equations.

angle sought.

.

be two lines drawn through the origin

parallel to the given lines.

Let

81.

cp

Then

The angle between these

(= BOG)

be this angle.

lines

is

the

SOLID ANALYTIC GEOMETRY.

244

which the line BO makes Let a y represent the angles the angle which and with X, Y, Z, respectively P (x y z } Take axes. same the with makes point CO any on CO and join them by on OB and any point ,

,

;

0",

",

y"

f

,

,

P"

Let

OF - I/,

From

OP"==

P

the triangle

But Art.

+ TL

IfO j

P P

OP".

I

T

ff>

168, equation (3)

and

(x"

2

2

2

,

//2

x"*+

=

cos

=L

=

L"

Substituting in cos

<f

(1),

(*"

- *r

we have ~

-

<p

Ov

T~^T"

169, (1)

x x"

,

y"*

Substituting these values in

But Art.

/-i\

(2)

=x +y +* = +^ - xj + (if - yj + L"

=

2

-Lr

2

2

L.

2L07-

L2

L

=

P"

we have

OP",

^

COS

and

L",

z")

y",

(x",

a right line forming the triangle

= cos

cos cos

cos

/?

,

=L = "

L"

(2),

cos

y"

+ cos

/3

cos

/3"

+ cos y

for the required relation. 90 COR. If v

= + cos

r

"

cos

cos

187.

To ymd ^e

^3

cos y cos

L"

y"

)3",

we have

"

cos

=L =

?/

,

",

cos

)8"

+ cos

awr/Ze ^//-ic/i

^<?o

y

cos

= y"

cos

.

.

.

y"

.

.

.

(3)

(4)

each space Zmes maA;e with

the projections other in terms of functions of the angles which axes. the lines make with the co-ordinate

of

be, as

of the lines in the preceding article, the equations

THE STRAIGHT LINE. drawn through the origin

P

(x, y

,

z

),

= sz =

x

y

*

I/ 2

and, Art. 168,

Eliminating,

we

=

parallel to the given lines.

a point on the

is

Fig. 81,

first line,

2 x"

+y +z\ 2

find

*

2

and since have

^

2

vi

r/ P"

(x

,

x"

y"

is

y",

")

=s = M, =x +

L"

+s + 2

1

_~

2

vi

+ *+T

a

a point on the second line,

;

we

z"

2

and, Art. 168,

Since

we have

,

"

vi + s +

245

1

2

"

y"

-f-

^ //2

.

Hence,

Vl +

s*

+

But, Art. 169,

L/

Substituting these values reducing,

we have _

Vl +

2 .9

for the required expression.

in

equation

Vl + s (3),

/2

+

Art. 186,

t

12

and

SOLID ANALYTIC GEOMETRY.

246 COB.

1.

If

<f>

= 0,

are parallel

the lines

and equation

(1)

becomes i

=

T= =4J^===.

Clearing of fractions and squaring,

we have

and Performing the operations indicated, transposing have we lecting,

of these quantities cannot be hence is equal to zero

But the sum of the squares

each separately equal to zero unless

j

=

<

s?

t

=

f

t,

col

st

;

=s

t

.

.

.

(2)

The first lines. are the conditions for parallelism of space in lines two space are two of these conditions show that if are their projections on the co-ordinate planes parallel, then conse mere a is s third condition t) parallel

also.

The

(sf

and quence of the other two,

may

=

be omitted in stating the

conditions for parallelism. to each 90, the lines are perpendicular COB. 2. If 9

=

other,

and equation

(1)

becomes 1

hence is

1

+ ss + =

+ ** +

it

(

3 >

the condition for perpendicularity in space. with any one ot Since the angle which a line makes

188. of the angle which the the co-ordinate axes is the complement to which that axis is line makes with the co-ordinate plane the complements of p, y if we let ft y be perpendicular ,

,

,

respectively,

we have

for the sines of the angles

co-ordinate planes.

which a space

line

makes with the

THE STRAIGHT LINE.

247

TRANSFORMATION OF CO-ORDINATES. 189. To find the equations of transformation from one system of co-ordinates to a parallel system, the origin being changed.

FIG.

82.

Let X, Y, Z be the old axes and X Y Z the new. Let P be any point on the locus CM. Draw PB, ,

OL

||

to

OZ and

meeting

XOY

in B,

,

A K, B and L. Draw BE Y draw LX to BE

and produce it to A BE will be to and LE to OX. Then (OA, AB, BP) ;

||

||

;

=

||

(

x

,

y,

z)

are the

old co-ordinates of the point P.

(O

A A B B P) = ,

,

(a;

,

y

,

z

)

are the

new

co-ordinates of

the point P.

(OX, XL, LO ) new origin From the figure .

=

(a, b, c)

are the old co-ordinates of the

SOLID ANALYTIC GEOMETRY.

248

OA = ON + O A AB ,

hence

x

=

a

-f-

x

==

NL + A B BP = LO + B P ,

,

y

=b

-\- ?/,

z

=c+z

;

are the required equations.

To find the equations of transformation from a rec tangular system in space to an oblique system, the origin being 190.

the same.

FIG.

Let OX, OY,

OZ

83.

be the old axes, and

OX OY OZ ,

,

the

new.

Let u

OZ

Let

OZ

f

be the angles which

OX

makes with OX, OY,

u",

fi",

f be

the angles which

OY

makes with OX, OY,

respectively.

Let

OZ

ft,

,

respectively.

"

,

$

",

f"

be the angles which

OZ makes

with OX, OY,

respectively.

Let

P

be any point on the locus

CM.

Draw PB and PB

THE STRAIGHT LINE. ||

249

OZ and OZ respectively, and let B and B be the points which these lines pierce the planes XOY and X OY

to

in

,

.

A

Draw B

OY

to

||

=

(OA, AB, BP) point P.

(OA

,

then

;

A B B P) =

f

(x

,

are the old co-ordinates of the

y, z)

(x,

!

y

,

)

,

new

are the

co-ordinates of the

point P.

From

B

P,

on the X-axis

A

and

the perpendiculars PA,

let fall

then from the

;

figure,

BT>,

AL

we have

OA = OL + LD + DA and DA are the projections of OA A B and PB respectively on the X-axis, and each, therefore, is equal to the line whose projection it is into the cosine of the angle which that line makes with the X-axis. (See Art. 169 (1) .)

LD

But OL,

,

,

,

OL = OA =x i.e., OL

cos

.-.

cos

LD = A B cos DA = B P cos LD = y cos DA = z cos ,

"

,

",

hence, substituting, we have r X x COS -f- y COS

Similarly

= = x y z = x

cos

"

</

,

p+y

n"

-f z COS a

p + -f if cos f + z

cos y

;

cos

cos f

cos

"

p"

1

...

I

(1)

Y"

Of the nine angles involved in these equations, six only are independent, for since the old axes are rectangular, we must have (See Art. 169, equation cos 2

cos 2

"

Con.

1.

we must have cos

cos

cos

cos

"

cos

"

"

2

I

(

2)

the

new axes

j

to be rectangular also

in addition to equation (2) the following condi See Art. 186, Cor. :

-f cos /? cos cos cos

"

y"

1

2

"

we suppose

tional equations

cos

2

"

If

).

2 -f cos / cos 2

$"

cos 2

(2)

=1 + p cos =1 + + cos + cos + f=1 cos 2

+ +

p cos p

/3"

+ cos f cos = + cos f cos = + cos f cos y = 7"

p"

cos

/"

"

/?

I

(3)

"

j

Hence, in this case, only three of the nine angles involved in equation (1) are independent.

SOLID ANALYTIC GEOMETRY.

250

THE CONIC SECTIONS. 191..

The CONIC SECTIONS, or, more simply, THE CONICS, from the surface of a right circular cone by

are the curves cut

a plane. We wish to show that every such section is an ellipse, a Art. or one of their limiting cases. parabola, an hyperbola, 146.

To deduce the equation of the

192.

FIG.

Let

any element

Draw DP

PK

to ||

84.

CAEA C be the conic surface, CA about OZ as an axis.

the element

OZ,

as ||

KB

meet the base

generated by revolving Let P be any point on

= c and OEC = 6. and intersecting OZ in D; draw XY-plane and K producing it to to OY, and join

CE

to

conic surface.

;

let

OC

||

circle in E.

Then (OB, BK, KP) = (a?, y, z) are the co-ordinates From the similar triangles COE, CDP, we have

50.-00_tan.. DP

OE

.(1)

of P.

THE STRAIGHT LINE. DC = 00 - PK = c -

But

z,

and

251

DP = OK = Vz + 2

y

z

hence, z

c

i.e.,

is

(c

-

2

z)

= tan =

2

(x

;

+ if)

tan

2 .

.

.

(2)

the required equation.

193.

To find the equation of the and a plane.

intersection of

a right

cir

cular cone

GALA

X

be the cone and OY the cutting plane. Let angle which the cutting plane makes with the plane of the cone s base, Let P (x, y, z) be any point on the curve of intersection

Let

X OX, BPB

.

the

=

We

<p.

wish to find the equation of this curve when

OX as axes. Draw PD to OY PL and DK

referred to

r

OY, ||

(OK, KL, LP)

=

;

(x, y, z)

||

to

OZ

;

then

are the space co-ordinates of P,

\

SOLID ANALYTIC GEOMETRY.

252

= (x

and (OD, DP) to

OX

f

y) are the co-ordinates of

,

P when referred

OY.

,

From

OD cos

KL =

the figure

PL = KD = OD sin

DP,

qp,

OK =

go.

=

V

i.e.,

= x s m Vi

z V>

x

=x

cos V-

But these values of x, y, & must subsist together with the curve of equation of the conic surface for every point on the and intersection; hence substituting in (2), Art. 192, reducing 2 2 2 we tan. cos sin that have, cp ?, dropping y remembering

=

accents, 2 ?/

2

tan

l9

+x

2

2

cos 2

+2 casing

tan

<p(tan

c

2

=

.

.

.

(1)

for the equation of the intersection. By giving every value to y from

value from

to

equation

oo,

(1)

to 90 and to c every can be made to represent every

section cut from a cone by a plane except sections planes that are parallel to the co-ordinate planes.

COR.

1.

Comparing

(1)

a

= tan.

c

= cos

with

(1),

Art. 138.

we

made by

find

2

2

2 (JP

(tan

9

tan

2 g>)

Hence, equation (1) represents an ellipse, a parabola, an Art. hyperbola or one of their limiting cases according as, 146. b

2

4 ac

<

IP =.

b

Case

and

c

1.

>

;

>

If 6

b

hence,

and

>

We

c

=

<p

this (0, 0)

4

>

<p.

ellipse.

4 ac

2

hence

it is

find this supposition in (2) gives a 4 ac, i.e., the intersection is

>

an

<

the equation resulting from introducing the point (1) can only be satisfied by

0,

supposition in ;

2

ac.

the equation of two imaginary lines inter

secting at the origin.

THE STRAIGHT LINE. ]f

= 0,

cp

equation

that

CASE c

=

2.

.-.

qp

and

2 ?/

which

is

2

tan

6>

=c

2 ,

a circle.

is

Hence the

=

c

2

This supposition in

g>.

= 4c.

Z>

=

=

(9

2

If

+x

2

the intersection

is,

becomes

(1)

if tan

From

0.

tan

2 <9

253

-

is

a parabola,

=

y

i.e.,

the equation of the X-axis a straight line. 90and<?= oo,then the cone becomes a cp cylinder,

= =

If

and the cutting plane is perpendicular to its base. section is therefore two parallel lines. CASE 3. 9 This supposition makes a cp.

The

<

b

2

4

>

If 6

Hence the

ac.

<

q>

and

=

c if-

intersection

then

tan

(1)

=x

2

2

is

c

0,

>

.-.

and

>

we have

(1),

;

gives a

(2)

intersection

inter

<

an hyperbola.

becomes

cos 2

2

(tan

9)

tan

<p

2

0)

which is the equation of two intersecting lines. CASE 4. Planes to the co-ordinate planes. to XY-plane. Let z = m be the (a) Plane \\

\\

such a plane. Combining surface, we have

it

equation of with the equation of the conic

tan

which (b)

is

the equation of a circle for

Plane

||

such a plane.

2

or

7/

tan

2 which, since b values of n.

(c)

Plane

a plane. tion a;

\\

all values of m. Let x = n be the equation of Combining with (2), Art. 192, we have to

YZ-plane.

2

>

to

z?

4

+ 2 cz + n

ac, is

tan

2

XZ-plane. Let y

z2

*

tan

2

c

2

=

.

.

.

(4)

the equation of an hyperbola for

Combining with 2

...

2

= p be

(2), Art. 192,

+ 2 c +^

2

tan

2

all

the equation of such after reduc

we have c

2

=

.

.

.

(5)

SOLID ANALYTIC GEOMETRY.

254 which, since values of p 2

b

2 >

4

ac, is

the equation of an hyperbola for

all

.

Hence, in

of the cutting plane, the possible positions an ellipse, a parabola, an hyperbola, or one of

all

intersection is

their limiting cases.

NOTE.

Equations

(3),

(4),

(5)

of

case

4 are the equa

on the tions of the projections of the curves of intersection the But are projection of any parallel. planes to which they to the given curve a is a on equal parallel plane plane curve the conclusions of case 4 are true for the curve

;

hence

curves themselves.

We have defined the conies, Art. 191, as the curves the surface of a right circular cone by a plane, and from cut found and discussed their assuming this definition we have 194.

general equation, Art. 193. A conic, however, may be otherwise defined as the locus in a plane that the ratio of its generated by a point so moving is always constant. distance from a fixed point and a fixed line

195.

To deduce the general equation of a

conic.

Y

D

FIG.

the basis of the Let us assume the definition of Art. 194 as the fixed line. Let F be the fixed point and operation. of its path. Let P be the generating point in any position

OY

THE STRAIGHT Draw FO Draw PL

OY, and take OY and OX as co-ordinate axes. OY, PD J_ to OY, and join P and F. Let OF

to

||

e

2

FP =

definition

From but

FL

-

=

e

DP = 2

=

e*x

2

2

=

2 .

These values

in (1) give e

or, after

a constant.

FPL, FP == FL + PL 2 ;...(!) 2 (OL - OF) = (x - p)\ LP 2 = f- and FP 2

triangle 2

255

J_ to

= p. By

LINE.

=

x2

2

reduction,

+

y-

-p)

(x

-e

(1

2

x2

)

+ if

2

- 2px + p* =

.

.

.

(2)

for the required equation.

COR.

Comparing a

hence

6

CASE

1.

2

(2)

=

4 ac

with

1, b

=

(1),

= 0, 4

c 2

e )

(1

we

Art. 138,

and

The fixed point not on

= (1 = 4 (e~

find

2

e ),

1)

.

the fixed line

.

;

.

(3)

i.e.,

p

not

zero.

If e

an

<

1, b

2 <

ellipse.

= 1,

If e

6

2

4

ac.;

= 4 ac

hence equation

(2) is the

hence equation

;

equation of

the equation of a

(2) is

parabola. If e

>

1, 6

2 >

4 ac

;

hence equation

(2) is the

equation of

an hyperbola.

CASE

The fixed point (2) becomes

2.

is

on the fixed

2

^2

line, i.e.,

p

= 0.

In this case

f+ If e

<

1,

(1

-

e )

=

.

.

.

(4)

equation (4) represents two imaginary lines inter

secting at origin. If

e

= l,

equation

(4)

represents

one

straight line (the

X-axis). If e

>

1,

equation

(4) represents

two straight

lines inter

secting at the origin.

Hence, equation iting cases.

(2) represents the conies or

one of their lim

SOLID ANALYTIC GEOMETRY.

256

GENERAL EXAMPLES. Find the point of intersection of the 2 x = 1 x = 2

1.

lines

+

+

and the cosine of the angle between them. ^%s.

Required the equation of the to 2, 3) and is parallel

2.

(1

_92

}

(3, 5, 1)

line

y~l-V}What

3.

g>

^|

x

=2z

?/=

the angle between the lines

is

What

5

-.+

Ans. 4.

5

=

which passes through

^

_]_ 1 )

cos

;

is

the distance of the point

(

3, 2,

V

-

=

90.

from

1)

the line

y 5.

A

= 4z

-\-

3

i

makes equal angles with the co-ordinate axes the angles which it makes with the co-ordinate

line

required

;

planes.

4y 2x The equation of a surface is x -f y* + _ 2 what does the equation become when the surface 2

-

6.

Qz

is

;

at

referred to a parallel system of axes, the origin being 2 16. z Ans. x 2 if ?

+

(1, 2, 3)

7.

Given the

the line on

XY

line

x ?

+ ^

^

2 ,

j

8.

Ans.

Required the distance cut I

the projections of the line

j~_

9

=

of required the projection

and the point on which the

co-ordinate planes.

+

off

__

line pierces the

in part, 2 y

on the Z and

+ = 4. a-

Y

axes by

A

^ x == 9

on Y^s

Ans. ?/

"

= -

_3o

6 .

THE STRAIGHT LINE. How

9.

257

are the following pair of lines related ?

= 2z

x

x

2

= 2z-

10. What are the equations of the line which passes through the origin and the point of intersection of the lines

What is origin ? What 11.

on

the distance of the point (3. 2, angle does this line make with

= 3z\

x

A

from the

4) its

projection

XY ? 12. A

straight line makes an angle of 60 with the X-axis and an angle of 45 with the Y-axis what angle does it make Ans. 60. with the Z-axis ? ;

What

13.

Q

"1

are the cosines of the angles

make with 14.

A

which the

lines

^

through the point

line passes

angles with X, Y,

the co-ordinate axes ?

Z whose cosines

V2

are

and makes

(1, 2, 3)

1

2

1 ?

res P ect

2

"

ively; required (a) the

equation of the line, the equation of the plane J_ to the line at the point, to the (V) to show that the projections of the line are J_ traces of the plane. (b)

15.

The

directional cosines of

two

lines are

212 and

o

Xfrj L

1,1. What

is

,

o

,

o

the cosine of the angle which they

2i

make with each other

?

Ans.

Cos

y>

=

3 -

+2 V2 b

16.

x

The projecting planes

=2y

-\-

2.

jects the line

What on

YZ ?

is

of a line are x

=3z

1

and

the equation of the plane which pro 3. 2y Ans. 3 z

=

SOLID ANALYTIC GEOMETRY.

258

and YZ each form with The projections of a line on the 45 of an equation of the line Z-axis the required angle which passes through (2, 1, 4) parallel to the line.

XZ

17.

;

Find the equation of the

18.

(3,.

2, 1)

and meets the

/

Given the

19.

line

x

lines

quired of (a) the value

*

line

which contains the point

* Z

^ _~3

=

l

at right angles. [

and

**+_l\ }

x y

=

+

*

\ J

|

;

re-

order that the lines may be parallel order that the lines may be perpen in of s (b) the value dicular the value of s in order that the lines may intersect. s in

;

;

(c)

212 - -

directional cosines of a line are

required o o the co-ordi the sines of the angles which the line makes with nate planes. 20.

The

21.

Find the equations of the

o

the origin and -

and y 22.

x

=

2,

i

Z

Find

is

1 )

line

,

;

,

which passes through

_

two lines perpendicular to the ^ Ans. x

r

11 &

)

the

angle

+ By + C* = D

and

included

between

Ax + By +

C

If

=

z

_^_

3z

3

j-

)

2 z\

the two planes

=D

.

AA +

+B + a

2

23.

j-

two planes are parallel show that the

C"

coefficients of

the variables in their equations are proportional. 24.

Find the condition for perpendicularity of the two

planes given in

Example

22.

Ans.

AA + BB + CO - 0.

SURFACES OF THE SECOND ORDER.

CHAPTER

259

IV.

A DISCUSSION OF THE SURFACES OF THE SECOND ORDER. By

A. L. Nelson, M.A., Professor of Mathematics in ton and Lee University, Va.

EVERY equation

Washing

involving three variables represents a sur

If the equation be of the first degree the surface will be a plane. If the equation be of a higher degree the surface will be curved. It is proposed in this to determine face.

chapter the nature of the surfaces represented by equations of the second degree involving three variables. The most general 2 2 form of the equation of the second degree is Ax 2 By/ -f- Cz

+ Day + Vxz + -Fyz +

Gx

+ Hy +

Iz

+

+K=

.

.

.

(1)

where

the coefficients A, B, C, etc., may be of either sign and of any magnitude. Let us suppose the co-ordinate axes to be rectan

The form of equation (1) may be simplified gular. by a transformation of axes. Let us turn the axes without chan ging the origin. The formulae

of transformation are (Art. 190)

=X y = x z =x X

f

f

z COS a + y COS cos p + / cos $ + z cos cos f + if cos / + cos f

COS

"

a"

-f-

"

ft

Substituting these values, equation (1) becomes

AV* + By + C V + DVv/ + E + TV + K = (2). 2

2

.

.

.

W+

F yV

+ GV + Hy

SOLID ANALYTIC GEOMETRY.

260

Since the original axes were supposed rectangular the nine are connected by the three relations angles ft f etc.,

+ cos

cos 2 cos 2 COS 2 a If

2

ft

2 4- cos 2 4- COS

"

"

ft"

=

f

2 4- cos

/J"

we take the new axes

able, the

2 4- cos

y"

2 4- COS

I.

= 1. =

1.

/"

also rectangular,

which

desir

is

nine angles will be connected by the three additional

relations

cos a cos

"

-j-

cos a

cos cos

"

cos

u"

cos

cos

ft

+ cos

"

/?

4- cos

cos

ft"

4- cos

ft"

"

ft

cos

f cos

f=

4- cos y cos

"

/?

-4-

f"

f cos

cos

y"

0.

= 0. = 0.

This will leave three of the nine angles to be assumed arbi Let us give to them such values as to render the co trarily. efficients D E and F each equal to zero in equation (2). The general equation will thus be reduced to the form ,

AV

or,

2

4-

,

By

2

4-

CV

2

4-

GV + Hy 4- IV 4- K = 0,

omitting accents,

Ax 2 In order

By 4 Cs to make a 2

4-

+ GJB + Hy + I

2

Equation

A

H

a 4- #

(3) will

(b

+ //)

.

.

(3)

>

=b +y

y

r ,

z

=

f-

4- %

become

Y +B

(a 4- x

.

move the origin without changing The formulae of transformation

will be (Art. 189)

=

K=

further reduction in the form of the

equation let us endeavor to the direction of the axes.

x

4-

(b

+ y Y + C (c + * )* + G +K=

4- I (c 4- *

(a

+x 4 )

0.

)

O

2 2 2 4- B& Developing, omitting accents, and placing A HZ Ic 4- Ga L, the equation takes the form Ax 2 4- B// 2 C,? 2 (2 Att 4- G) x 4- (2 B^ +H) y 4- (2 Cc +1 )z

+

+

+ + L = 0.

In order

and

c,

now

+

+K= +

to give definite values to the quantities a,

which were entirely

2A

,

arbitrary, let us

= --1L_ 2B

c

assume

= _ _JL or

b,

SURFACES OF THE SECOND ORDER.

= If these values of a, reduces to the form

b,

and

c

Cc

,

+I=

.

.

(4)

be Unite, the general equation

+ By + C* + L = 2

.

261

2

.

.

[A], a form which will be set aside for further examination. It may be remarked that equations (4) are of the first degree, and will give only one value to each of the quantities a, b, and c, and there is therefore only one position for the

new

.

origin.

If,

C

however, either A, B, or

become distance.

be zero, then

a, b,

and the origin will be removed This, must be avoided.

infinite,

Let us suppose then assume 2 B6 assume 2 Aa G

+

A=

0,

B and C are finite. We may and 2 Cc + I = 0, but we cannot

while

+ H = 0,

= 0.

or c will

to an infinite

Having assumed the values of assume the entire constant term

b

and

c as

indicated, let us This will give

equal to zero.

+ Cc + Ga + Hb + I C + K = 0, a = - g + Co + Kb + Ic + K

B6 2

2

2

or

2

G and the general equation will be reduced to the form 2 By + C* a + G* = .

.

.

(B),

a second form set aside for examination. must observe that this last proposed transformation will also fail when G 0, that is, when the first power of x as well as the second power of x is wanting in the general equation.

We

=

,

And

without making the second transformation we have a examination, viz.

third form for

:

IV*

+ C* + Hy + I* + K = a

.

.

(C) Lastly, two of the terms involving the second powers of the variables may be wanting, and the equation then .

(1)

becomes

Cs 2

+

Gx

+ Hy 4. Iz + K =

.

.

.

(D)

262

SOLID ANALYTIC GEOMETRY.

It is apparent, therefore, that

every equation of the second degree involving three variables can be reduced to one or another of the four forms

A* 2 + By 2 2 By + C* B?/

C

We

will

with the

+ C.? + Gx

2

2

2

+ Cz* + L = + Gx = (B) + H// + I* + K + Hy + Iz + K =

2

.

.

examine each of these forms form

first

.

(A)

.

.

.

.

.

.

.

.

.

in order,

(C)

(D)

beginning

:

Ax 2

+ B?/ + 2

C* 2

+

L

=

.

.

.

(A)

This equation admits of several varieties of form according to the signs of the coefficients.

C

L

1.

A, B, and

2.

A, B, C, and L positive. Two of the coefficients as

3.

positive,

and

negative in the

A

B

and

first

member.

positive,

C and L

C

negative,

negative.

Two

4.

and L

Ko

of the coefficients as

A

and

B

positive,

positive.

other cases will occur.

CASE 1. Ax 2 + B?/ 2 + Cs 2 = L, in which form all of the coefficients are

positive.

In order to determine the nature of the surface represented by this equation, let it be intersected by systems of planes

The equations parallel respectively to the co-ordinate planes. of these intersecting planes will be x c. Com a, y b, z bining the equations of these planes with that of the surface,

=

=

=

we

find the equations of the projections on the co-ordinate planes of the curves of intersection.

When "

"

= y = z = x

Thus we

2

a, b, c,

By Ax Ax

2 2

+ C.? = L - Aa + Cz = L - IW + By = L - Cc 2

2

2

2

2

an an an

ellipse.

ellipse.

ellipse.

see that the sections parallel to each of the co

ordinate planes are ellipses.

SURFACES OF THE SECOND ORDER. The

L

made

section

Aa

2

or a

>

<

by the

-j-

y

plane

=a

x

-~ and imaginary ,

is

real

263

when

in the contrary

case.

The ft <

=i=

<

Thus we

>

and imaginary when

see that the surface

lar parallelepiped

When sections

When

a

ft

>

is

c

-J-

y

=b

is

y =c

-[-

s

>

real

when

real

when

.

is

i/

.

enclosed within a rectangu

whose dimensions are

=

become a

plane

made by the plane

section

i y ~7T

by the

an d imaginary when

V/~fT~>

The c

made

section

or 4 points.

= 0, = 0,

and by the co-ordinate planes ft

c

By + 2

Ax Ax

2 2

=

or

= 0, C,-

2

we

=

L.

+ C,v = L. By = L. 2

2

-f~

E

c

=

!

Vc

find the sections

the

made

SOLID ANALYTIC GEOMETRY.

264

These are called the principal sections of the surface. The principal sections are larger ellipses than the sections parallel to them, as is indicated by the magnitude of the absolute term.

The surface

called the Ellipsoid.

is

may be generated by the motion of an ellipse of variable dimensions whose centre remains on a fixed line, and whose It

plane remains always perpendicular to that line, and whose semi-axes are the ordinates of two ellipses which have the

same transverse

but unequal conjugate axes placed at

axis,

The axes of the principal sections right angles to each other. are called the axes of the ellipsoid. c,

If

we

we

shall

represent the semi-axes of the ellipsoid by

a, b,

and

have

and the equation of the surface Ax 2 + B?/ 2 -j- Cz 2 a2

b

2

b c

2

x

2

-f-

2

c

a

2 2

c

y

=L

becomes

2

2 -f-

a2b 2z 2

=

a 2b 2 c 2

.

These are the forms in which the equation of the ellipsoid is

usually given. If

we suppose B

= A, then b = a, and the z x + y _ 2

2

equation becomes

2

1

2~

2

and the surface

is

the Ellipsoid of Revolution about the axis

of Z. If

x

A=B 2

2

-j_ ?/

If

L

-(-

z

2

= 0,

then a

== C,

=a

2 ,

=b=

c,

and the surface

the axes 2

and the equation becomes is

a sphere.

2 i/-^-,

reduce to zero, and the ellipsoid becomes a point.

SURFACES OF THE SECOND ORDER.

265

L

be negative in the second member, the equa 2 L will represent an imaginary B?/ -j- Cz will be no there and geometrical locus. surface, Hence the varieties of the ellipsoid are

CASE

tion

If

2.

Ax 2

+

=

2

The ellipsoid proper with three unequal axes. of revolution with two equal axes. (2) The ellipsoid The sphere. (3) (4) The point. (5) The imaginary surface. CASE 3. In this case the equation takes the form (1)

Ax 2 + By 2 - Cz 2 = L, C, and L are essentially

which A, B, positive. Cutting the surface by planes as before, the sections will be, 2 L Aa 2 a hyperbola, having its when x Cz 2 a, ~By in

=

=

,

when a

transverse axis parallel to the Y-axis parallel to the Z-axis

v/

,

when a

-j-

t/ V

but

,

A.

And when

y

>

<

=

a

-J-

the intersection becomes two straight lines whose pro-

.A.

jections on the plane of

When

y

=

b,

Ax 2

YZ pass through

Cz 2

=L

the origin.

B& 2 a hyperbola, with simi ,

lar conditions as above.

When values of

z

=

c,

Ax 2

+ B?/ = 2

L

+ Cc

2 ,

an ellipse real for

all

c.

Since the elliptical sections are all real, the surface it consists of a single sheet.

is

con

tinuous, or

The

principal sections are found by

= 0, b = 0, c = 0, a

which gives By 2

Ax 2 Ax 2

"

-

Cz 2 Cz 2

+%

2

making successively

= L, a hyperbola. = L, = L, an ellipse.

The surface is called the elliptical hyperboloid of one The equation may be reduced to the form v?_ 2

.y*_ 2

^L *

=

i

sheet.

SOLID ANALYTIC GEOMETRY.

266

may be generated by an ellipse centre remains constantly on whose dimensions is whose and perpendicular to that axis, plane semi-axes are the ordinates of two hyperbolas same conjugate axis coinciding with the Z-axis, This surface

of variable

the Z-axis,

and whose

having the but having different transverse axes placed at right angles to each other.

FIG. B.

If

we suppose

A = B,

then will a

=

b,

and the equation of

the surface becomes

the hyperboloid of revolution of one sheet. If

A=B=

C,

we have x 2

+ if - ^ = a

z ,

the equilateral

one sheet. hyperboloid of revolution of a right cone having an If L 0, the equation represents B this base becomes a circle. base and if

=

elliptical

;

A=

SURFACES OF THE SECOND ORDER. Hence we have the following

267

varieties of the hyperboloid

of one sheet.

The liyperboloid proper, with three unequal axes. The hyperboloid of revolution. 3. The equilateral hyperboloid of revolution. 4. The cone. 2 = - L, where A, B, C, and CASE 4. Ax 2 + B?/ 2 1.

2.

O

are essentially positive.

L

=

a, Intersecting the surface as before we have, when x Aa 2 a hyperbola having its transverse L QgZ axis parallel to the axis of Z.

__

_

_

,

j>^2

When its

ing

When c

-j-

>

y

=

b,

Ax 2

Cz"

=

z

=

I/V C

,

c,

Ax 2

+ B// =

L

2

B6 2 -f

and imaginary when

=

sections between the limits real

L

A

.

hyperbola hav

transverse axis parallel to the axis of Z.

beyond those

c

-j-

Cc 2 an ellipse real when ,

i/ V

/

-J- t

<

Since the

.

V C

G

are imaginary, but

two

limits, it follows that there are

distinct

sheets entirely separated from each other. The surface is called the hyperboloid of two sheets. The principal sections are found by making successively 2 2 a 0, which gives By L, a hyperbola with its transverse axis parallel to the Z-axis.

O O

=

b

= 0,

which gives Az 2

=

2

=

L, a hyperbola with its

transverse axis parallel to the Z-axis. c

= 0,

which gives Ax 2

The semi-axes

+ By = 2

of the

L, an imaginary ellipse.

section

first

Those of the second section are I/

V A

of

the imaginary section

The distances 2

i/-^_X I

,

are

i/ V

A

and

(

,

Jj

V/ B i

and

t

And !/_. V

V

1)

A

and

i /

V

B

/

V

.

C

those

C

2 4/1*1. and 2 \/ T

are

(

1).

are called the axes

SOLID ANALYTIC GEOMETRY.

268

FIG. C.

and Eepreseiiting the semi-axes by a, b, form the to reduced the equation of the surface may be of the surface.

L

5 + -? If

we suppose

A=B

?

then a

c,

=

&,

and the equation

duces to

the hyperboloid of revolution of two sheets.

re

SURFACES OF THE SECOND ORDER. If

A=B=

C, the equation

becomes

+f_^= _

x*

269

a*

9

which represents the surface generated by the revolution of an equilateral hyperbola about

its transverse axis. the surface becomes a cone having an Finally, elliptical base, and the base becomes a circle when B. We have, therefore, the following varieties of the if

L

= 0,

A=

hyper

boloid of two sheets: 1.

2.

3. 4.

The hyperboloid proper having three unequal The hyperboloid of revolution. The equilateral hyperboloid of revolution. The cone.

We

will

now examine the second 2 2 By + C* + G x =

axes.

form, .

.

.

(B)

Three cases apparently different present themselves for examination.

B and C

positive

(2).

B, C, and

G

(3).

B

(1).

G

and

negative in the

first

mem

ber.

CASE

positive.

C and G negative. The equation may be written

positive and

1.

B?/

in

which B,

2

+ O. = Gx 2

G

and

are essentially positive. C, Let the surface be intersected as usual by planes parallel respectively to the co-ordinate planes.

When

x

=

,

By

+O =

2

2

Ga, an ellipse real when a

and imaginary when a 0. 2 When y = b, = Gx

>

0,

<

O

B6 2

,

a

parabola with

its

axis

its

axis

parallel to the axis of X.

When

z

=

c,

By

2

= Gx -

O

2

a parabola

with

parallel to the axis of X.

The and

principal sections are found by

c

When When origin.

making a

= 0, b = 0,

0.

a b

= 0, By + C,? = = 0. C.s = Ga;, a 2

2

2

0,

a point, the origin. its vertex at the

parabola with

SOLID ANALYTIC GEOMETRY.

270

When c, =

=

2 G#, a parabola with vertex at the origin. 0, B?/ Since every positive value of x gives a real section, and every negative value of x an imaginary section, the surface consists of a single sheet extending indefinitely and contin

uously in the direction of positive abscissas, but having no from the origin. points in the opposite direction The surface is called the elliptical paraboloid. It may be

dimensions generated by the motion of an ellipse of variable the same on remains whose centre straight line, constantly

and whose plane continues perpendicular to that line, and whose semi-axes are the ordinates of two parabolas having a common transverse axis and the same vertex, but different to each parameters placed with their planes perpendicular other.

FIG. D.

in the 2. If we suppose G to be positive form the take will the so that equation

CASE

By

2

+C = 2

Go;,

first

member

SURFACES OF THE SECOND ORDER.

271

the sections perpendicular to the X-axis will become imaginary

when x

>

0,

and

real

when x

<

0.

In other respects the results are similar to those deduced case

in

1.

will represent a surface of the same form but turned in the opposite direction from the

Thus the equation as in case

1,

co-ordinate plane of If B = C, the

CASE

surface becomes the paraboloid of revolution.

By

3.

YZ.

2

-C = a

Gx.

Intersect the surface

by planes as before. Gz 2 = Ga, a hyperbola with transverse and in the axis in the direction of the Y-axis when a direction of the Z-axis when a 0.

When

x

= a,

2

By

>

;

<

When

y

=

b.

in the direction of

=

=

Gx

B6 2 a parabola having its the X-axis and extending to the left.

Cz 2

-\-

=

,

axis

Gx -j- Cc a parabola having its axis in When z c B?/ the direction of the X-axis and extending to the right. 2

}

2

,

Since every value of x, either positive or negative, gives a real section, the surface consists of a single sheet extending This indefinitely to the right and left of the plane of YZ.

surface

is

called the Hyperbolic Paraboloid. To find its princi make x, y, and z alternately equal to zero.

pal sections

When x = 0, B^/ = Cz two straight lines. When y = 0, Cz = Gx, a parabola with axis to the left. When = 0, B?/ = Gx, a parabola with axis to the right. 2

2

,

2

2

The hyperbolic paraboloid admits of no variety. Now taking up form (C), By 2 + Cs a + Hy + Iz + K = 0, we see that it is the equation of a cylinder whose elements are perpendicular to the plane of YZ, and whose base in the plane of YZ will be an ellipse or hyperbola according to the signs of B and C.

+

+

+K=

The fourth form (D), Cz Ga; Hy -f Is sents a cylinder having its bases in the planes 2

XZ

repre

and

YZ

SOLID ANALYTIC GEOMETRY.

272

\

VA

A

/

\

/

\ /

FIG. E.

parabolas, and having its right-lined elements parallel to the and to each other, but oblique to the axes of and Y.

plane

X

XY

The preceding discussion shows that every equation

of the

second degree between three variables represents one or an other of the following surfaces the ellipsoid 1. The ellipsoid with its varieties, viz. :

;

the

of

proper, ellipsoid and the imaginary surface.

revolution, the

sphere, the

point,

SURFACES OF THE SECOND ORDER. The hyperboloid

2.

of

one

or

two

sheets,

273

with

their

the hyperboloid proper of one or two sheets, the hyperboloid of revolution of one or two sheets, the varieties, viz.

:

equilateral hyperboloid of revolution of one or two sheets, the cone with an elliptical or circular base.

The

3.

paraboloid, either elliptical or hyperbolic, with the

variety, the paraboloid of revolution.

The

4.

cylinder, having its base either an ellipse,

hyper

bola, or parabola.

The general equation of surfaces be deduced by a direct method, as follows

Surfaces of Revolution. of revolution

may

:

Ri

FIG. F.

Let the Z-axis be the axis of revolution, and tion

x2

of

= fz. Let

P

AB,

the

let

the equa

generating curve in the plane of XZ, be

be the point in this curve which generates the circle

SOLID ANALYTIC GEOMETRY.

274

PQR, and

be the radius of the

let r

The value

circle.

We

will

have

of r2

may also be expressed in terms of z from the equation of the generatrix in the plane of XZas follows :

= CP = 2

r2

2

(JD =fz.

Equating these two values of r

we have

as the general equation of surfaces of revolution. It will be observed that the second value of r 2

is the value of x 2 in the equation of the generatrix. Hence, to find the equation of the surface of revolution we have only to substi 2 tute x 2 if of the surface for x in the generatrix.

+

2 Surface of a Sphere. Equation of generatrix x Hence the equation of the surface of the sphere is

+

z2

=

K,.

2

Ellipsoid of Revolution.

Generatrix

Surface

JL_

x*

-f-

~^~

J!L

^

= 1, 1

-4-

a2

c

2

Similarly, the equation of the hyperboloid of revolution

***-*-* Paraboloid of Revolution. x2 4:pz, the generatrix.

= + IT = 4=pz, the Cone of revolution, z = mx x2

surface of revolution.

-+- j3

~T Hence or

x

x

the generatrix,

~

~~i^~

2 -

m

2

2

(x

+

if)

=

(z

-

2

ft)

.

is

SURFACES OF THE SECOND ORDER.

275

EXAMPLES. 1.

9z 7/2

(^ 3.

2

Of ^ 2

+ if = r- ?

Qf

2

if

Determine the nature of the surfaces x 2

+^

_

= 36 ? + =r ?

the locus in space of 4 x 2 -f 9 y-

is

- 16 y = 144 ? + 8x = o?

2.

7

What

2

4

.2

=

-\-

2

,-

y"-

-(-

4 z2

Of Of

= 25,

79>

Find the equation of the surface of revolution about the Z whose generatrix is z = 3 x -\- 5.

axis of 4.

Find the equation of the cone of revolution whose inter 2 = 9, and whose vertex -f- y

section with the plane of XYisx 2 is

(0, 0, 5.)

5.

Determine the surfaces represented by

+ 4 y + 9 s = 36. + 4 y _ 9 # = 36 - 36. x + 4y = 9 4 y + 9 2 = 36 x. 9z* = 36x. 2

2

x*

2

a;

2

a

2

2

2

2

14

DAY USE

RETURN TO DESK FROM WHICH 4S This book

.OWED

^Sic

dbl below.or date stamped is cfil on tfie last on the date to which renewed. recall

Renewed books

are subject to immediate

7

LD

21-40m-5, 65

(F4308slO)476

General Library University of California Berkeley

^ vo

\* t

View more...

Comments

Copyright ©2017 KUPDF Inc.
SUPPORT KUPDF